You are on page 1of 163

Chemical Engineering Department

University of Technology

Mathematical
Modeling and
Numerical Analysis

Post graduate M. Sc.

Assoc. Prof. Dr. Zaidoon Mohsin shakor


Chapter 1 : Fundamentals

1.1 Introduction
The objective of mathematical modeling is the development of sets of quantitative
(mathematical) expressions that capture the essentials aspects of an existing system.
A mathematical model can assist in understanding the complex physical interactions
in the system and the causes and effects between the system variables. Mathematical
models are valuable tools since they are abstract equations that can be solved and
analyzed using computer calculations. It is therefore safer and cheaper to perform test
on the model using computer simulations rather than to carry out repetitive
experimentations and observations on the real system. This becomes vital if the real
system is new, hazardous, or expensive to operate. Modeling thus prevails the field of
science, engineering and business. It is used to assist in the design of equipment, to
predict behavior, to interpret data, to optimize resources and to communicate
information.

1.2 Incentives for process modeling


In the chemical engineering field, models can be useful in all the phases, from
research and development to plant operation. Models and their simulation are tools
utilized by the chemical engineer to help him analyze the process in the following
ways:
• Better understanding of the process
Models can be used to study and investigate the effects of various process
parameters and operating conditions on the process behavior. It can also be used
to evaluate the interactions of different parts of the process. This analysis can be
carried out easily on a computer simulation without interrupting the actual
process, thus avoiding any delay or upsets for the process.
• Process synthesis and design
Model simulation can be utilized in the evaluation of equipment's size and
arrangements and in the study of alternative process flow-sheeting and

1
strategies. Furthermore, to verify the reliability and safety of the process design
tests can be carried out even prior to plant commissioning.
• Plant operators training
Models can be used to train plant personnel to simulate startup and shutdown
procedures, to operate complex processes and to handle emergency situations
and procedures.
• Controller design and tuning
Models help in developing and evaluating better controller structure and
configuration. Dynamic simulation of models is usually employed for testing
and assessing the effectiveness of various controller algorithms. It is worthwhile
to mention that models play a vital part in designing advanced model-based
control algorithms such as model predictive and internal model controller.
Moreover it is a common practice of many control engineers to determine the
optimum values of the controller settings through dynamic simulation .
• Process optimization
It is desirable from economic standpoint to conduct process optimization before
plant operation to determine the optimum values of the process key parameters
or/and operating conditions that maximizes profit and reduces cost. Process
optimization is also performed during process operation to account for
variations in the feed-stock and utilities market and for changing environmental
regulations.

It is worth mentioning that despite all their usefulness, models at their best are no
more than approximation of the real process since they do not necessarily incorporate
all the features of the real system. Therefore modeling can not eliminate completely
the need for some plant tests, especially to validate developed models or when some
poorly known parameters in the process need to be experimentally evaluated.

Models can be classified in a number of ways. But since mathematical models are
developed from applying the fundamental physical and chemical laws on a specific

2
system, we review first the classification of systems since their nature affect the
modeling approach and the resulting model.

1.3 Systems
A system is a whole consisting of elements or subsystems. The system has
boundaries that distinguish it from the surrounding environment (external world) as
shown in Figure 1.1. The system may exchange matter and/or energy with the
surrounding through its boundary. Consequently, the state of a system can be defined
or understood via the interactions of its elements with the external world. A system
may be classified in different ways, some of which are as follows:

1.3.1 Classification based on thermodynamic principles


• Isolated system
This type of system does not exchange matter nor energy with the surrounding.
Adiabatic batch reactor is an example of such systems.
• Closed system
This type of system does not exchange matter with the surrounding but it does
exchange energy. Non-adiabatic batch reactor is an example of such systems .
• Open system
This system exchanges both matter and energy with the external environment.
An example of this system is the continuous stirred tank reactor (CSTR).

Boundary

System
Suroundings

Figure 1.1: system and its boundary

3
1.3.2 Classification based on number of phases
• Homogeneous system
This is a system that involves only one phase such as gas-phase or liquid-phase
chemical reaction processes.
• Heterogeneous system
This is a system that involves more than one phase. This kind of systems exists
in multi-phase reaction processes and in phase-based separation processes.

1.4 Classification of Models


Models can be classified according to how they are derived :
• Theoretical models.
These are models that are obtained from fundamental principles, such as the
laws of conservation of mass, energy, and momentum along with other
chemical principles such as chemical reaction kinetics and thermodynamic
equilibrium, etc. Theoretical models are, however, generally difficult to obtain
and sometimes hard to solve.
• Empirical models
These models are based on experimental plant data. These models are
developed using data fitting techniques such as linear and non-linear regression.
Such models do not provide detailed description of the underlying physics of
the process. However, they do provide a description of the dynamic relationship
between inputs and outputs. Thus they are sometimes more adequate for control
design and implementation.
• Semi-empirical models
These models are somehow between the two previous models where uncertain
or poorly known process parameters are determined from plant data.

1.5 State variables and state equations


Once the system has been classified, developing a theoretical model for it amounts
to characterizing its behavior at any time and at any spatial position. For most

4
processing systems a number fundamental quantities are used to describe the natural
state of the system. These quantities are the mass, energy and momentum. Most often
these fundamental quantities can not be measured directly thus they are usually
represented by other variables that can be measured directly and conveniently. The
most common variables are density, concentration, temperature, pressure and flow
rate. There are conveniently called 'state variables' since they characterize the state of
the processing system.
In order to describe the behavior of the system with time and position, the state
(dependent) variables should be linked to the independent variables (time, spatial
position) through sets of equations that are derived from writing mass, energy and
momentum balances. The set of equations describing these variables are called 'state
equations.'

1.6 Classification of theoretical models


Theoretical models may be further classified in more practical ways as discussed in
the following.

1.6.1 Steady state vs. unsteady state


When the physical state of the processing system remains constant with the time,
the system is said to be at steady state. Models that describe steady state situations are
also called static, time-invariant or stationary models. Basically, almost all chemical
process unit designs are carried out on static models. On the other hand unsteady state
processes represent the situation when the process state (dependent variables)
changes with time. Models that describe unsteady-state situations are also called
dynamic and transient models. Such models are useful for process control design and
development. Process dynamics are encountered in practice during startup, shutdown,
and upsets (disturbances).

1.6.2 Lumped vs. distributed parameters


Lumped parameters models are those in which the state variables and other
parameters have/or assumed to have no spatial dependence, i.e. they are considered to
5
be uniform over the entire system. In this case the time (for unsteady state models) is
the only independent variable. The chemical engineering examples for this case
include the perfectly mixed CSTR, distillation columns etc. Conceptually, these
models are obtained through carrying out a macroscopic balance for the process . On
the other hand, distributed parameters models are those in which states and other
variables are function of both time and spatial position. In this case, modeling takes
into account the variation of these variables with time and from point to point
throughout the entire system. Some examples of such systems include plug flow
reactor, heat exchangers, and packed columns. These models are essentially obtained
through writing microscopic balances equations for the process.

1.6.3 Linear vs non-linear


Linear models have the important property of superposition whereas nonlinear
models do not. Superposition means that the response of the system to a sum of
inputs is the same as the sum of responses to the individual inputs. In linear models
all the dependant variables or their derivatives appear in the model equations only to
the first power. These properties do not hold for nonlinear models. In this respect, it
is important to recognize the fact that most physical and chemical systems are
nonlinear. Linear models are commonly obtained through linearization of the
nonlinear model around a certain steady state.

1.7 Building steps for a mathematical model


Building a theoretical mathematical model of a processing system requires the
knowledge of the physical and chemical interactions taking place within the
boundaries of the system. With a given degree of fundamental knowledge of the
system at a certain stage one can build different models with different degree of
complexity depending on the purpose of the model building and the level of rigor and
accuracy required. The choice of the level of rigor and degree of sophistication is in
itself an art that requires much experience.
Consequently a modeler should practice careful utilization of the simplifying
assumptions based on engineering sense and experience. Failure to do so, the modeler
6
may fall into one of the two extremes, i.e. creating rigorous but over complicated
model or creating oversimplified model that does not capture all the critical features
of the true process.
The general procedure for building up a mathematical model includes the
following steps:
• Identification of the system configuration, its surrounding environment and the
modes of interaction between them, the identification of the relevant state
variables that describe the system and identification of the process taking place
within the boundaries of the system.
• Introduction of the necessary simplifying assumptions.
• Formulation of the model equations based on principles of mass, energy and
momentum balances appropriate to the type of the system. This also requires the
determination of the fundamental quantitative laws (chemical kinetics,
thermodynamic relations…) that govern the rates of the process in terms of the
state variables .
• Determination of the solvability of the model using degree of freedom analysis.
• Development of the necessary numerical algorithms for the solution of the
model equations.
• Validation of the model against experimental results to ensure its reliability and
to re-evaluate the simplifying assumptions which may result in imposing new
simplifying assumptions or relaxing others.

1.8 Conservation Laws


As mentioned in previous section, the state equation forming the mathematical
model defines a relationship between the state variables (dependent variables) and the
independent variables, i.e., time and spatial variables of the system. These equations
are derived, from applying the conservation law for a specific fundamental quantity
say S, on a specific system with defined boundaries (see Figure 1.2) as follows:

7
S

Sin
System Sout

Figure 1.2

Total flow Generation Total flow rate Accumulation amount of (S)


rate of (S) + rate of (S) = of (S) out of + rate of (S) ± exchanged (1.1)
into the within the system within system with the
system system surrounding

The quantity S can be any one of the following quantities:


• Total Mass
• Component Mass (Mole)
• Total Energy
• Momentum

1.8.1 Total mass balance


Since mass is always conserved, the balance equation for the total mass (m) of a
given system is:
Rate of mass in = rate of mass out + rate of mass accumulation (1.2)

The mass balance equation has the SI unit of kg/s.

1.8.2 Component balance


The mass balance for a component A is generally written in terms of number of moles
of A. Thus the component balance is :

8
Rate of Flow of Rate of
Flow of
Generation of moles of (A) Accumulation of (1.3)
moles (A) in + = +
moles of (A) out moles of (A)

The component balance has the unit of moles A/s. It should be noted that unlike the
total mass, the number of moles of species A is not conserved. The species A can be
generated or consumed by chemical reaction.

1.8.3 Momentum balance


The linear momentum (π) of a mass (m) moving with velocity (v) is defined as:
π = mv (1.4)

Since the velocity v is a vector, the momentum, unlike the mass is also a vector. The
momentum balance equation using (Eq 1.1) is:

Rate of Rate of Rate of Rate of


momentum in + Generation of = momentum + Accumulation of (1.5)
momentum out momentum

The momentum balance has the unit of kg.m/s2. The momentum balance equation is
P P

usually written using the Newton's second law. The law states that the time rate of
change of momentum of a system is equal to the sum of all forces F acting on the
system,
d (mv)
= ∑F
(1.6)
dt

1.8.4 Energy balance


The energy balance for a given system is:

9
Rate of Rate of Rate of Rate of ± Amount of
energy + Generation = energy + accumulation energy (1.7)
in of energy out of energy exchanged
with the
surrounding

The energy generated within a system includes the rate of heat and the rate of work.
The rate of heat includes the heat of reaction (if a reaction occurs in the system), and
the heat exchanged with the surroundings. For the rate of work we will distinguish
between the work done against pressure forces (flow work) and the other work such
as the work done against the gravity force, against viscous forces and shaft work.

For a given system the general conservation law (Eq. 1.1) can be carried out either on
microscopic scale or macroscopic scale.

1.9 Microscopic balance


In the microscopic case, the balance equation is written over a differential element
within the system to account for the variation of the state variables from point to
point in the system, besides its variation with time. If we choose for example
cartesian coordinates, the differential element is a cube as shown in Figure 1.3. Each
state variable V of the system is assumed to depend on the three coordinates x,y and z
plus the time. i.e. V = V(x,y,z,t). The microscopic balance can be also written in
cylindrical coordinates (Figure 1.4) and in spherical coordinates (Figure 1.5). The
selection of the appropriate coordinates depends on the geometry of the system under
study.

10
z

y
∆z
∆y

∆x

Figure 1.3: Cartesian coordinates

y
r
θ

Figure 1.4: Cylindrical coordinates

θ
r z

Figure 1.5: Spherical coordinates

11
1.10 Macroscopic balance
In some cases the process state variables are uniform over the entire system, that is
each state variable does not depend on the spatial variables, i.e. x,y and z in cartesian
coordinates but only on time t. In this case the balance equation is written over the
whole system using macroscopic modeling. When modeling the process on
microscopic scale the resulting models consists usually of partial differential equation
(PDE) where time and one or more spatial position are the independent variables. At
steady state, the PDE becomes independent of t and the spatial positions are the only
independent variables. On the other hand, when the modeling is based on
macroscopic scale the resulting model consists of sets of ordinary differential
equations (ODE) .
The fundamental balance equations of mass, momentum and energy already
discussed are usually supplemented with a number of equations associated with
transport rates and thermodynamic relationships. In the following we present an
overview of some of these relations.

1.11 Transport rates


Transport of the fundamental quantities, mass, energy and momentum occur by
two mechanisms:
• Transport due to convection or bulk flow.
• Transport due to molecular diffusion or potential difference

In many cases the two transport mechanism occurs together. Therefore, the flux due
to the transport of any fundamental quantity is the sum of a flux due to convection
and a flux due to diffusion.

1.11.1 Mass Transport


The total flux n Au (kg/m2s) of species A of density ρ A (kg/m3) flowing with velocity
R R P P R R P P

v u (m/s) in the u-direction is the sum of the two terms:


R R

n Au = j Au + ρ A v u
R R R R R R R
(1.8)

12
total flux = diffusive flux + bulk flux (1.9)

The diffusive flux j Au (kg/m2s) for a binary mixture A-B is given by Fick's law:
R R P P

dw A (1.10)
j Au = − ρD AB
du

where w A = ρ A /ρ is the mass fraction of species Α, ρ (kg/m3) the density of the


R R R R P P

mixture and D AB (m2/s) is the diffusivity coefficient of A in the mixture. In molar unit
R R P P

the flux J Au (mol A/m2s) is given by:


R R P P

dx A (1.11)
J Au = −CD AB
du

where C is the total concentration of A and B (Kg(A+B)/m3) and x A = C A /C is the P P R R R R

mole fraction of A in the mixture. For constant density the flux Eqs. ( 1.10 and Eq.
1.11) become:
dρ A (1.12)
j Au = − DAB
du
dC A (1.13)
J Au = − D AB
du

1.11.2 Momentum transport


Momentum is also transported by convection and diffusion. But unlike the mass,
the linear momentum π = mv is a vector. We have to consider then its transport in all
directions (x,y,z) of a given system. Let consider for instant the transport of the x-
component of the momentum. Similar analysis can be carried out for the transport of
the y and z component.
The flux due to convection of the x-component of the momentum in the y-direction,
for instant, is:
(ρv x )v y
R R R R (kg.m/s2)
P P
(1.14)

To determine the diffusion flux denoted by τ yx of the x-component of the momentum R R

in the y-direction, consider a fluid flowing between two infinite parallel plates as
13
shown in Figure 1.6. At a certain time the lower plate is moved by applying a
constant force F x while the upper plate is maintained constant. The force F x is called
R R R R

a shear force since it is tangential to the area A y on which it is applied (Fig. 1.6). R R

y
∆y F, force
∆ Vx

Figure 1.6: Momentum transfer between two parallel plates

The force per unit area


Fx
(kg.m / s 2 )
(1.15)
Ay

is called a stress and denoted τyx (kg.m/s). It is also a shear stress since it is tangential.
R R

The force F x imparts a constant velocity V = v x (y = 0 ) to the layer adjacent to the


R R R R

plate. Because of molecular transport, the layer above it has a slightly slower velocity
v x (y) and so on as shown in Figure 1.6. Therefore, there is a transport by diffusion of
R R

the x-component of the momentum in the y-direction. The flux of this diffusive
transport is in fact the shear stress τyx . R R

Therefore, the total flux πyx of the x-component in the y-direction is the sum of the
R R

convection term (Eq. 1.14) and diffusive term τ yx R

π yx = τ yx + (ρv x )v y
R R R R R R R
(1.16)
momentum flux = diffusive flux + bulk flux (1.17)

For a Newtonian fluid the shear stress τyx is proportional to the velocity gradient: R R

∂v x (1.18)
τ yx = − µ
∂y

where µ (kg/m.s) is the viscosity of the fluid.


14
1.11.3 Energy transport
The total energy flux e u (J/s.m2) of a fluid at constant pressure flowing with a
R R P P

velocity v u in the u-direction can be expressed as:


R R

e u = q u + (ρC p T)v u
R R R R R R R
(1.19)
energy flux = diffusive flux + bulk flux (1.20)

The heat flux by molecular diffusion, i.e. conduction in the u-direction is given by
Fourier's law:
∂T (1.21)
qu = −k
∂u

where k (J/s.m.K) is the thermal conductivity.

The three relations (Eq. 1.10, 1.18, 1.21) show the analogy that exists between mass,
momentum and energy transport. The diffusive flux in each case is given by the
following form:
Flux = − transport property × potential difference (gradient) (1.22)

The flux represents the rate of transfer per area, the potential difference indicates the
driving force and the transport property is the proportionality constant. Table 1.1
summarizes the transport laws for molecular diffusion.

Table 1.1: One-dimensional Transport laws for molecular diffusion


Transport Law Flux Transport gradient
Type property
dC A
Mass Fick's J Au
R D
du
dT
Heat Fourrier qu R K
du
dv x
Momentum Newton τ ux R µ
du

15
When modeling a process on macroscopic level we can also express the flux by a
relation equivalent to (Eq. 1.22). In this case the gradient is the difference between
the bulk properties, i.e. concentration or temperature in two medium in contact, while
the transport property represents an overall transfer coefficient. For example for mass
transfer problems, the molar flux can be expressed as follows:
J A = K × ∆C A (1.23)
where K an overall mass transfer coefficient.

The heat flux on the other hand is expressed as


q = U × ∆T (1.24)

where U is an overall heat transfer coefficient.

As for the momentum balance, the macroscopic description generally uses the
pressure drop as the gradient while the friction coefficient is used instead of the flux.
The following relation is for instance commonly used to describe the momentum
laminar transport in a pipe.

f =
D
∆P
(1.25)
2v 2 Lρ

f is the Fanning friction factor, ∆P is the pressure drop due to friction, D is the
diameter of the pipe, L the length and v is the velocity of the fluid.

1.12 Thermodynamic relations


An equation that relates the volume (V) of a fluid to its temperature (T) and
pressure (P) is called an equation of state. Such equations are used to determine fluid
densities and enthalpies .
• Densities :
The simplest equation of state is the ideal gas law:
PV = nRT (1.26)
Which can be used to determine the vapor (gas) density
16
ρ = M w P/RT R R
(1.27)
where M w is the molecular weight. As for liquids, tabulated values of density
R R

can be used and can be considered invariant unless large changes in


composition or temperature occur.

• Enthalpies :
Liquid and vapor enthalpies for pure component can be computed from simple
formulas, based on neglecting the pressure effect as follows:
~
h = C p (T − Tref ) (1.28)
~
H = C p (T − Tref ) + λ (1.29)
~
Where h is the liquid specific enthalpy, H~ is its vapor specific enthalpy, C p

the average liquid heat capacity and λ is the latent heat of vaporization

Note that the reference condition is taken to be liquid at temperature T ref . If


R R

heat of mixing is negligible, the enthalpy of a mixture can be taken as the sum
of the specific enthalpies of the pure components multiplied by their
corresponding mole fractions Note that the above enthalpy functions are valid
for small temperature variation and/or when the heat capacities of fluids are
weak function of temperature. In general cases the heat capacity C p can be
R R

taken as function of temperature such as:


C p = a + bT + cT2 + dT3
R R P P P (1.30)
The specific enthalpies for vapors and liquids are computed as integrals as
follows:
~
T (1.31)
h= ∫ C p dT
Tref

~
T (1.32)
H= ∫ C p dT + λ
Tref

• Internal energy :

17
The internal energy U is a fundamental quantity that appears in energy balance
equation. For liquids and solids the internal energy can be approximated by
enthalpy. This can be also a good approximation for gases if the pressure
change is small.

1.13 Phase Equilibrium


A large number of chemical processes involve more than one phase. In many cases
the phases are brought in direct contact with each other such as in packed or tray
towers. When the transfer of either mass or energy occurs from a fluid phase to
another phase the interface between fluid phases is usually at equilibrium. In the case
of heat transfer between phase I and Phase II, the equilibrium dictates that the
temperature is the same at the interface. This is not the case for mass transfer. Figure
1.7 shows for instant the mass transfer of species A from a liquid to a gas phase. The
concentration of the bulk gas phase y AG decreases at the interface. The liquid
R R

concentration increases, on the other hand, from x AL to x Ai . At the interface, an


R R R R

equilibrium exists and y Ai and x Ai are related by a relation of the form of


R R R R

y Ai = F(x Ai )
R R R R (1.33)

The equilibrium relations are in general nonlinear. However, quite satisfactory results
can be obtained through the use of simpler relations that are derived form
assumptions of ideal behavior of the two phases:

Gas-phase mixture Liquid -phase solution


of A in gas G of A in liquid L

yAG yAi
xAi
xAL
NA

Interface

Figure 1.7: Equlibrium at the interface

18
• For vapor phases at low concentration, Henry's law provides the following
simple equilibrium relation:
p A = Hx A
R R R (1.34)
Where p A (atm) is the partial pressure of species A in the vapor, x A is the mole
R R R R

fraction of A in the liquid and H is the Henry's constant (atm/mol fraction)

Dividing by the total Pressure (P) we get another form of Henry's law:
~
yA = H xA
R R R
(1.35)
~
The constant ( H ) depends on temperature and pressure.

• Raoult's law provides a suitable equilibrium law for ideal vapor-liquid


mixtures
y A P = x A PAS (1.36)

Where x A is the liquid-phase mole fraction, y A is the vapor-phase mole


fraction, P A s is the vapor pressure of pure A at the temperature of the system,
R RP P

and P is the total pressure on gas-phase side.

The dependence of vapor pressure P A s on temperature can be approximated


R RP P

by the Antoine equation

ln( PAS ) = A −
B (1.37)
C +T
where A, B and C are characteristic parameters of the fluid.

• Raoult’s law can be modified to account for non-ideal liquid and vapor
behavior using the activity coefficients γ i and φi of component (i) for liquid R R R R

and vapor phase respectively. The following equilibrium relation, also known
as the Gamma/Phi formulation, can be used:
y iφi P = xi γ i Pi S (1.38)

19
The activity coefficients can be determined using correlations found in standard
thermodynamics text books. Equation (1.38) is reduced to Raoult law for ideal
mixture i.e. ( φi = γ i =1).

• In some cases the transfer occurs in liquid-liquid phases (such as liquid


extraction) or liquid-solid (such as ion exchange). In such cases an equilibrium
relation similar to Henry's law can be defined:
y A = Kx A
R R R (1.39)
where K is the equilibrium distribution coefficient that depends on pressure,
temperature and concentration.

Phase equilibrium relations are commonly used in the following calculations:


• Bubble point calculations :
For a given molar liquid compositions (x i ) and either T or P, the bubble point
R R

calculations consist in finding the molar vapor composition (y i ) and either P or


R R

T.
• Dew point calculations :
For a given molar vapor compositions (y i ) and either T or P, the dew point
R R

calculations consist in finding the molar liquid compositions (x i ) and either P


R R

or T.
• Flash calculation :
For known mixture compositions (z i ) and known (T) and (P), the flash
R R

calculations consist in finding the liquid and vapor compositions via


simultaneous solution of component balance and energy balance equations.

1.14 Chemical kinetics


The overall rate R in moles/m3s of a chemical reaction is defined by:
P P

R=
1 dni (1.40)
viV dt

20
where
ni
R R the number of moles
νi
R R the stochiometric coefficient of component i
V is the volume due to the chemical reaction.
The rate expression R is generally a complex relation of the concentrations (or partial
pressures) of the reactants and products in addition to pressure and temperature.

For a general irreversible reaction


ν r1 R 1 + ν r2 R 2 + …+ ν rr R rr  ν p1 P 1 + ν p2 P 2 + …+ ν pp R pp
R R R R R R R R R R R R R R R R R R R R R R R
(1.41)

The law of mass action stipulates that the reaction rate is a power law function of
temperature and concentration of reactants, i.e.
R = kCRζ1 C Rζ 2 C Rζ r (1.42)
1 2 r

The powers ξ i are determined experimentally and their values are not necessarily
R R

integers. The temperature dependence comes from the reaction rate constant k given
by Arrhenius law
−E (1.43)
k = ko e RT

where k o is the pre-exponential factor, E the activation energy, T the absolute


R R

temperature and R is the ideal gas constant

1.15 Degrees of Freedom


A key step in the model development and solution is checking its consistency or
solvability, i.e. the existence of exact solution. This is done by checking the degrees
of freedom of the model after the equations have written and before attempting to
solve them.
For a processing system described by a set of N e independent equations and N v R R R R

variables, the degree of freedom f is


F = Nv − Ne R R R (1.44)

21
Depending on the value of f three cases can be distinguished:
• f = 0. The system is exactly determined (specified) system. Thus, the set of
balance equation has a finite number of solutions (one solution for linear
systems).
• f < 0. The system is over-determined (over-specified) by f equations. f
equations have to be removed for the system to have a solution.
• f > 0. The system is under-determined (under-specified) by f equations. The set
of equation, hence, has infinite number of solution.

To avoid the situations of over-specified or under-specified systems it is advised to


follow the following steps while checking the consistency of the model.
1. Determine known quantities of the model that can be fixed such as equipment
dimensions, constant physical properties, etc.
2. Determine other variables that can specified by the external world, for
example, variables that are the outcome of an upstream processing units,
and/or variables that can be used as forcing function or manipulated variables.

1.16 Model solution


After the solvability of the model has been checked, the next step is to solve the
model. The purpose of the solution of the model is be able to obtain the variations of
the state variables with the model independent variables (time, spatial positions..).
The solution of the model permits also a parametric investigation of the model, that is
a study of the effects of the changing the value of some parameters. It would be ideal
to be able to solve the model analytically, which is to get closed forms of the state
variables in term of the independent variables. Unfortunately this seldom occurs for
chemical processes. The reason is that the vast majority of chemical processes are
nonlinear. They may be a sets of nonlinear partial differential equations (PDE) as it is
the case for distributed parameter models, or sets of nonlinear ordinary differential
equations (ODE) or nonlinear algebraic equations as it is the case for lumped
parameter models. However most non linear problems can not be solved analytically.

22
In fact the only class of differential equations for which there is a well-developed
framework are linear ODE. However linearizing the original nonlinear model and
solving it is not always recommended (expect for control purposes) since the
behavior of the linearized model matches the original nonlinear model only around
the state chosen for linearization. For these reasons the solution of process models is
usually carried out numerically, most often through a computer programs.

1.18 Model validation


Model verification (validation) is the last and the most important step of model
building. Reliability of the obtained model depends heavily on faithfully passing this
test. Implementation of the model without validation may lead to erroneous and
misleading results. So, it is essential, as it is saves a lot of effort, time and frustration,
to verify the model against plant operating data, experimental data, or at least
published correlations. If the model failed in the test, then it might be necessary to
adjust some of the model parameters, which is believed to be poorly known, in order
to minimize the mismatch between the model and the true plant. In worst cases, a
modeler may need to reconsider some of the simplifying assumptions used or the
neglected modeling parts. However, it should be kept in mind that the model is no
more than approximation of the real world, thus some degree of mismatch will
remain and could be overlooked.

23
Chapter 2: Examples of Mathematical Models for Chemical
Processes

In this chapter we develop mathematical models for a number of elementary


chemical processes that are commonly encountered in practice. We will apply the
methodology discussed in the previous chapter to guide the reader through various
examples. The goal is to give the reader a methodology to tackle more complicated
processes that are not covered in this chapter and that can be found in books listed in
the reference. The organization of this chapter includes examples of systems that can
be described by ordinary differential equations (ODE), i.e. lumped parameter systems
followed by examples of distributed parameters systems, i.e those described by
partial differential equations (PDE). The examples cover both homogeneous and
heterogeneous systems. Ordinary differential equations (ODE) are easier to solve and
are reduced to simple algebraic equations at steady state. The solution of partial
differential equations (PDE) on the other hand is a more difficult task. But we will be
interested in the cases were PDE's are reduced to ODE's. The distinction between
lumped and distributed parameter models depends sometimes on the assumptions put
forward by the modeler. Systems that are normally distributed parameter can be
modeled under appropriate assumptions as lumped parameter systems. This chapter
includes some examples of this situation.

2.1 Examples of Lumped Parameter Systems

2.1.1 Liquid Storage Tank

Consider the perfectly mixed storage tank shown in figure 2.1. Liquid stream with
volumetric rate F f (m3/s) and density ρ f flow into the tank. The outlet stream has
R R P P R R

volumetric rate F o and density ρ ο . Our objective is to develop a model for the
R R R R

variations of the tank holdup, i.e. volume of the tank. The system is therefore the
liquid in the tank. We will assume that it is perfectly mixed and that the density of the
effluent is the same as that of tank content. We will also assume that the tank is
24
isothermal, i.e. no variations in the temperature. To model the tank we need only to
write a mass balance equation.

Ff
ρf V
Fo
ρo

Figure 2.1 Liquid Storage Tank

Since the system is perfectly mixed, the system properties do not vary with position
inside the tank. The only variations are with time. The mass balance equation can be
written then on the whole system and not only on a differential element of it. This
leads to therefore to a macroscopic model.
We apply the general balance equation (Eq. 1.2), to the total mass m = ρV. This
yields:
Mass flow in:
ρf FfR R R
(2.1)
Mass flow out:
ρo Fo
R R R
(2.2)
Accumulation:
dm d (ρV )
=
dt dt (2.3)
The generation term is zero since the mass is conserved. The balance equation yields:
d (ρV )
ρ f F f = ρo Fo +
dt (2.4)
For consistency we can check that all the terms in the equation have the SI unit of
kg/s. The resulting model (Eq. 2.4) is an ordinary differential equation (ODE) of first
order where time (t) is the only independent variable. This is therefore a lumped
parameter model. To solve it we need one initial condition that gives the value of the
volume at initial time t i , i.e.
R R

25
V(t i ) = V i
R R R (2.5)
Under isothermal conditions we can further assume that the density of the liquid is
constant i.e. ρ f = ρ o =ρ. In this case Eq. 2.4 is reduced to:
R R R R

dV
= F f − Fo
dt (2.6)
The volume V is related to the height of the tank L and to the cross sectional area A
by:
V = AL (2.7)
Since (A) is constant then we obtain the equation in terms of the state variable L:
dL
A = F f − Fo
dt (2.8)
with initial condition:
L(t i ) = L i
R R R (2.9)

Degree of freedom analysis


For the system described by Eq. 2.8 we have the following information:
• Parameter of constant values: A
• Variables which values can be externally fixed (Forced variable): F f R

• Remaining variables: L and F o R

• Number of equations: 1 (Eq. 2.8)

Therefore the degree of freedom is:


Number of remaining variables – Number of equations = 2 – 1 = 1

For the system to be exactly specified we need therefore one more equations. This
extra relation is obtained from practical engineering considerations. If the system is
operated without control (at open loop) then the outlet flow rate F o is a function of
R R

the liquid level L. Generally a relation of the form:


Fo = α L (2.10)

Could be used, where α is the discharge coefficient.

26
Note that at steady state, the accumulation term is zero (height does not change with
time), i.e., dL/dt = 0. The model of the tank is reduced to the simple algebraic
equation:
F0 = Ff
R R R R (2.11)

2.1.2 Stirred Tank Heater

We consider the liquid tank of the last example but at non-isothermal conditions.
The liquid enters the tank with a flow rate F f (m3/s), density ρ f (kg/m3) and
R R P P R R P P

temperature T f (K). It is heated with an external heat supply of temperature T st (K),


R R R R

assumed constant. The effluent stream is of flow rate F o (m3/s), density ρ o (kg/m3) R R P P R R P P

and temperature T(K) (Fig. 2.2). Our objective is to model both the variation of liquid
level and its temperature. As in the previous example we carry out a macroscopic
model over the whole system. Assuming that the variations of temperature are not as
large as to affect the density then the mass balance of Eq. 2.8 remains valid.
To describe the variations of the temperature we need to write an energy balance
equation. In the following we develop the energy balance for any macroscopic system
(Fig. 2.3) and then we apply it to our example of stirred tank heater.
Consequently, the flow of energy into the system is:
~
ρf Ff h f R R R R
(2.12)

Where the ( ~• ) denotes the specific enthalpy (J/kg).


Ff , Tf , ρf

Q
L

Fo , To , ρo

Tst
Heat supply

Figure 2.2 Stirred Tank Heater

27
Figure 2.3 General Macroscopic System

The flow of energy out of the system is:


~
ρ o F o ho
R R R R
(2.13)

The rate of accumulation of energy is:


(
d ρVh
~
) (2.14)
dt
As for the rate of generation of energy, it was mentioned in Section 1.8.4, that the
energy exchanged between the system and the surroundings may include heat of
reaction Q r (J/s), heat exchanged with surroundings Q e (J/s).
R R R R

Substituting all these terms in the general balance equation (Eq. 1.7) yields:
(
d ρVh
~
) ~ ~
= ρ f F f h f − ρ o Fo ho + Qe + Qr (2.15)
dt
We can check that all terms of this equation have the SI unit of (J/s). We return now
to the liquid stirred tank heater. A simplifying assumption can be introduced, that
there is no reaction involved, i.e. Q r = 0.R R

The energy balance (Eq. 2.15) is reduced to:


d ρVh
~
( ) ~ ~
= ρ f F f h f − ρ o Fo ho + Qe (2.16)
dt
Here Q e is the heat (J/s) supplied by the external source. The enthalpy is generally a
R R

function of temperature, pressure and composition. However, it can be safely


estimated from heat capacity relations as follows:

28
~ ~
h = Cp(T − Tref ) (2.17)
~
Where Cp is the average heat capacity.

Furthermore since the tank is well mixed the effluent temperature T o is equal to R R

process temperature T. The energy balance equation can be written, assuming


constant density ρ f = ρ o = ρ , as follows:
R R R R

ρC p
(
~ d V (T − Tref ) ~ ) ~
= ρF f C p (T f − Tref ) − ρFoC p (T − Tref ) + Qe (2.18)
dt
Taking T ref = 0 for simplicity and since V = AL result in:
R R

~ d (LT ) ~ ~
ρC p A = ρF f C pT f − ρFoC pT + Qe (2.19)
dt
or equivalently:
d (LT ) Q
A = F f T f − FoT + ~e (2.20)
dt ρC p

Since
d (LT ) d (L ) d (T )
A = AT + AL (2.21)
dt dt dt
And using the mass balance (Eq. 2.8) we get:
dT Q
AL + T ( F f − Fo ) = F f T f − FoT + ~e (2.22)
dt ρC p

or equivalently:
dT Q
AL = F f (T f − T ) + ~e (2.23)
dt ρC p

The stirred tank heater is modeled, then by the following coupled ODE's:

A
dL
= F f − Fo
(2.24)
dt

AL
dT Q
= F f (T f − T ) + ~e
(2.25)
dt ρC p

This system of ODE's can be solved if it is exactly specified and if conditions at


initial time are known,
L(t i ) = L i
R R R R and T(t i ) = T i
R R R (2.26)

29
Degree of freedoms analysis
For this system we can make the following simple analysis:
• Parameter of constant values: A, ρ and C p R

• (Forced variable): F f and T fR R R

• Remaining variables: L, F o , T, Q e R R R

• Number of equations: 2 (Eq. 2.24 and Eq. 2.25)

The degree of freedom is therefore, 4− 2 = 2. We s till need two relations for our
problem to be exactly specified. Similarly to the previous example, if the system is
operated without control then F o is related to L through (Eq. 2.10). One additional
R R

relation is obtained from the heat transfer relation that specifies the amount of heat
supplied:
Q e = UA H (T st −T )
R R R R R R (2.27)
U and A H are heat transfer coefficient and heat transfer area. The source temperature
R R

T st was assumed to be known.


R R

2.1.3 Isothermal CSTR

We revisit the perfectly mixed tank of the first example but where a liquid phase
chemical reactions taking place:
k
A
→ B (2.28)
The reaction is assumed to be irreversible and of first order. As shown in figure 2.4,
the feed enters the reactor with volumetric rate F f (m3/s), density ρ f (kg/m3) and R R P P R R P P

concentration C Af (mole/m3). The output comes out of the reactor at volumetric rate
R R P P

F o , density ρ 0 and concentration C Ao (mole/m3) and C Bo (mole/m3). We assume


R R R R R R P P R R P P

isothermal conditions.
Our objective is to develop a model for the variation of the volume of the reactor
and the concentration of species A and B. The assumptions of example 2.1.1 still hold
and the total mass balance equation (Eq. 2.6) is therefore unchanged

30
Ff
ρf
CAf V Fo
CBf ρo
CAo
CBo
Figure 2.4 Isothermal CSTR

The component balance on species A is obtained by the application of (Eq. 1.3) to


the number of moles (n A = C A V ). Since the system is well mixed the effluent
R R R R

concentration C Ao and C Bo are equal to the process concentration C A and C B .


R R R R R R R R

Flow of moles of A in:


F f C AfR R R (2.29)
Flow of moles of A out:
F o C Ao
R R R (2.30)
Rate of accumulation:
dn d (VC A )
= (2.31)
dt dt
Rate of generation: -rV
where r (moles/m3s) is the rate of reaction.
P P

Substituting these terms in the general equation (Eq. 1.3) yields:


d (VC A )
= F f C Af − Fo C A − rV (2.32)
dt
We can check that all terms in the equation have the unit (mole/s).

We could write a similar component balance on species B but it is not needed since
it will not represent an independent equation. In fact, as a general rule, a system of n
species is exactly specified by n independent equations. We can write either the total
mass balance along with (n −1) component balance equations, or we can write n
component balance equations.

31
Using the differential principles, equation (2.32) can be written as follows:
d (VC A )
=V
d (C A )
+ CA
d (V )
= F f C Af − Fo C A − rV
(2.33)
dt dt dt
Substituting Equation (2.6) into (2.33) and with some algebraic manipulations we
obtain:
d (C A )
V + C A ( F f − Fo ) = F f C Af − Fo C A − rV (2.34)
dt
or equivalently:
d (C A )
V = F f ( C Af − C A ) − rV (2.35)
dt
In order to fully define the model, we need to define the reaction rate which is for a
first-order irreversible reaction:
r = k CA R R (2.36)
Equations 2.6 and 2.35 define the dynamic behavior of the reactor. They can be
solved if the system is exactly specified and if the initial conditions are given:
V(t i ) = V i and C A (t i ) = C Ai
R R R R R R R R R (2.37)

Degrees of freedom analysis


• Parameter of constant values: A
• (Forced variable): F f and C Af
R R R

• Remaining variables: V, F o , and C A R R R

• Number of equations: 2 (Eq. 2.6 and Eq. 2.35)

The degree of freedom is therefore 3− 2 =1. The extra relation is obtained by the
relation between the effluent flow F o and the level in open loop operation (Eq. 2.10).
R R

The steady state behavior can be simply obtained by setting the accumulation terms
to zero. Equation 2.6 and 2.35 become:
F0 = Ff
R R R (2.38)
F f ( C Af − C A ) = rV (2.39)

32
2.1.4 Isothermal CSTR of Two feed streams
More complex situations can also be modeled in the same fashion. Consider the
catalytic hydrogenation of ethylene:
A+BP (2.40)
Where A represents hydrogen, B represents ethylene and P is the product (ethane).

The reaction takes place in the CSTR shown in figure 2.5. Two streams are feeding
the reactor. One concentrated feed with flow rate F 1 (m3/s) and concentration C B1 R R P P R R

(mole/m3) and another dilute stream with flow rate F 2 (m3/s) and concentration C B2
P P R R P P R R

(mole/m3). The effluent has flow rate F o (m3/s) and concentration C B (mole/m3). The
P P R R P P R R P P

reactant A is assumed to be in excess.


F1, CB1 F2, CB2

V
Fo, CB

Figure 2.5 Reaction in a CSTR

The reaction rate is assumed to be:

r=
k1CB
( mole / m 3 .s )
(2.41)
(1 + k2CB ) 2

Where k 1 is the reaction rate constant and k 2 is the adsorption equilibrium constant.
R R R R

Assuming the operation to be isothermal and the density is constant, and following
the same procedure of the previous example we get the following model:
Total mass balance:
dL
A = F1 + F2 − Fo (2.42)
dt
Component B balance:
d (C B )
V = F1 ( C B1 − C B ) + F2 ( C B 2 − C B ) − rV (2.43)
dt

33
Degrees of freedom analysis
• Parameter of constant values: A, k 1 and k 2 R R R

• (Forced variable): F 1 F 2 C B1 and C B2


R R R R R R R

• Remaining variables: V, F o , and C B R R R

• Number of equations: 2 (Eq. 2.42 and Eq. 2.43)


The degree of freedom is therefore−3 2 =1. The extra relation is between the
effluent flow F o and the level L as in the previous example.
R R

2.1.5 Non-Isothermal CSTR

We reconsider the previous CSTR example (Sec 2.1.3), but for non-isothermal
conditions. The reaction A  B is exothermic and the heat generated in the reactor is
removed via a cooling system as shown in figure 2.6. The effluent temperature is
different from the inlet temperature due to heat generation by the exothermic
reaction.

Ff , CAf , Tf

Qe V Fo, CA, T

Figure 2.6 Non-isothermal CSTR

Assuming constant density, the macroscopic total mass balance (Eq. 2.6) and mass
component balance (Eq. 2.35) remain the same as before. However, one more ODE
will be produced from the applying the conservation law (equation 2.20) for total
energy balance. The dependence of the rate constant on the temperature:
k = k o e-E/RT R R P (2.44)
The general energy balance (Eq. 2.20) for macroscopic systems applied to the CSTR
yields, assuming constant density and average heat capacity:

34
ρC p
(
~ d V (T − Tref ) ~ ) ~
= ρF f C p (T f − Tref ) − ρFoC p (T − Tref ) + Qr − Qe (2.45)
dt
Where Q r (J/s) is the heat generated by the reaction, and Q e (J/s) the rate of heat
R R R R

removed by the cooling system. Assuming T ref = 0 for simplicity and using the R R

differentiation principles, equation 2.45 can be written as follows:


~ dT
ρC pV
~ dV
+ ρC pT
~ ~
= ρF f C pT f − ρFoC pT + Qr − Qe
(2.46)
dt dt
Substituting Equation 2.6 into the last equation and rearranging yields:
~ dT
ρC pV
~
= ρF f C p (T f − T ) + Qr − Qe
(2.47)
dt
The rate of heat exchanged Q r due to reaction is given by: R R

Q r = −(∆H r )Vr
R R R R
(2.48)
Where ∆H r (J/mole) is the heat of reaction (has negative value for exothermic
R R

reaction and positive value for endothermic reaction). The non-isothermal CSTR is
therefore modeled by three ODE's:
dV
= F f − Fo
(2.49)
dt

V
d (C A )
= F f ( C Af − C A ) − rV
(2.50)
dt
~ dT
ρC pV
~
= ρF f C p (T f − T ) + (−∆H r )Vr − Qe
(2.51)
dt
Where the rate (r) is given by:
r = k o e-E/RTC A R R P P R (2.52)
The system can be solved if the system is exactly specified and if the initial
conditions are given:
V(t i ) = V i
R R R R T(t i ) = T i R R R R and C A (t i ) = C Ai
R R R R R (2.53)

Degrees of freedom analysis


• Parameter of constant values: ρ, E, R, C p , ∆H r and k o R R R R R

• (Forced variable): F f , C Af and T f R R R R R

• Remaining variables: V, F o , T, C A and Q e R R R R R

• Number of equations: 3 (Eq. 2.49, 2.50 and 2.51)

35
The degree of freedom is 5−3 = 2. Following the analysis of example 2.1.3, the two
extra relations are between the effluent stream (F o ) and the volume (V) on one hand
R R

and between the rate of heat exchanged (Q e ) and temperature (T) on the other hand,
R R

in either open loop or closed loop operations.

2.1.6 Heat Exchanger

Consider the shell and tube heat exchanger shown in figure 2.7. Liquid A of
density ρ A is flowing through the inner tube and is being heated from temperature
R R

T A1 to T A2 by liquid B of density ρ B flowing counter-currently around the tube.


R R R R R R

Liquid B sees its temperature decreasing from T B1 to T B2 . Clearly the temperature of


R R R R

both liquids varies not only with time but also along the tubes (i.e. axial direction)
and possibly with the radial direction too. Tubular heat exchangers are therefore
typical examples of distributed parameters systems. A rigorous model would require
writing a microscopic balance around a differential element of the system. This
would lead to a set of partial differential equations. However, in many practical
situations we would like to model the tubular heat exchanger using simple ordinary
differential equations. This can be possible if we think about the heat exchanger
within the unit as being an exchanger between two perfect mixed tanks. Each one of
them contains a liquid.
Tw Liquid, B
TB1

Liquid, A
TA1 TA2

Liquid, B
TB2

Figure 2.7 Heat Exchanger

36
For the time being we neglect the thermal capacity of the metal wall separating the
two liquids. This means that the dynamics of the metal wall are not included in the
model. We will also assume constant densities and constant average heat capacities.
One way to model the heat exchanger is to take as state variable the exit temperatures
T A 2 and T B 2 of each liquid. A better way would be to take as state variable not the
R RR R R RR R

exit temperature but the average temperature between the inlet and outlet:
TA1 + TA2
TA = (2.54)
2
TB1 + TB 2
TB = (2.55)
2
For liquid A, a macroscopic energy balance yields:
dTA
ρ AC p VA = ρ A FAC p A (TA1 − TA2 ) + Q (2.56)
A
dt
where Q (J/s) is the rate of heat gained by liquid A. Similarly for liquid B:
dTB
ρ BC p VB = ρ B FBC p B (TB1 − TB 2 ) − Q (2.57)
B
dt
The amount of heat Q exchanged is:
Q = UA H (T B – T A )
R R R R R R (2.58)
Or using the log mean temperature difference:
Q = UA H ∆T lm R R R
(2.59)
Where
(TA2 − TB1 ) − (TA1 − TB 2 )
∆Tlm =
(T − TB1 )
ln A2 (2.60)
(TA1 − TB 2 )

with U (J/m2s) and A H (m2) being respectively the overall heat transfer coefficient and
P P R R P P

heat transfer area. The heat exchanger is therefore describe by the two simple ODE's
(Eq. 2.56) and (Eq. 2.57) and the algebraic equation (Eq. 2.58).

Degrees of freedom analysis


• Parameter of constant values: ρ Α , Cp A , V A , ρ Β , Cp B , V B , U, A H R R R R R R R R R R R R R

• (Forced variable): T A1, T B1 , F A , F B R R R R R R R

37
• Remaining variables: T A2 , T B2 , Q R R R R

• Number of equations: 3 (Eq. 2.56, 2.57, 2.58)

The degree of freedom is 5− 3 = 2. The two extra relations are obtained by noting
that the flows F A and F B are generally regulated through valves to avoid fluctuations
R R R R

in their values.
So far we have neglected the thermal capacity of the metal wall separating the two
liquids. A more elaborated model would include the energy balance on the metal wall
as well. We assume that the metal wall is of volume V w , density ρ w and constant heat R R R R

capacity Cp w . We also assume that the wall is at constant temperature T w , not a bad
R R R R

assumption if the metal is assumed to have large conductivity and if the metal is not
very thick. The heat transfer depends on the heat transfer coefficient h o,t on the R R

outside and on the heat transfer coefficient h i,t on the inside. Writing the energy R R

balance for liquid B yields:

ρ BC p VB
dTB
= ρ B FBC p B (TB1 − TB 2 ) − ho , t Ao , t (TB − TW )
(2.61)
B
dt
Where A o,t is the outside heat transfer area. The energy balance for the metal yields:
R R

ρ wC p Vw
dTw
= ho , t Ao , t (TB − Tw ) − hi , t Ai , t (Tw − TA )
(2.62)
w
dt
Where A i,t is the inside heat transfer area. The energy balance for liquid A yields:
R R

ρ AC p VA
dTA
= ρ A FAC p A (TA1 − TA2 ) + hi , t Ai , t (Tw − TA )
(2.63)
A
dt
Note that the introduction of equation (Eq. 2.62) does not change the degree of
freedom of the system.

2.7 Heat Exchanger with Steam


A common case in heat exchange is when a liquid L is heated with steam (Figure
2.8). If the pressure of the steam changes then we need to write both mass and energy
balance equations on the steam side.

38
Tw Steam
Ts(t)

Liquid, L
TL1 TL2

condensate, Ts

Figure 2.8 Heat Exchanger with Heating Steam

The energy balance on the tube side gives:


dTL
ρ LC p L VL = ρ L FLC p L (TL1 − TL 2 ) + Qs (2.64)
dt
where
TL1 + TL 2
TL = (2.65)
2
Q s = UA s (T s – T L )
R R R R R R R R (2.66)
The steam saturated temperature T s is also related to the pressure P s : R R R R

T s = T s (P)
R R R R (2.67)
Assuming ideal gas law, then the mass flow of steam is:
M s PsVs
ms = (2.68)
RTs

Where M s is the molecular weight and R is the ideal gas constant. The mass balance
R R

for the steam yields:


M sVs dP
= ρ s Fs − ρc Fc (2.69)
RTs dt

Where F c and ρ c are the condensate flow rate and density. The heat losses at the
R R R R

steam side are related to the flow of the condensate by:


Qs = Fc λs
R R R R R (2.70)
Where λ s is the latent heat.
R R

Degrees of freedom analysis


• Parameter of constant values: ρ L , Cp L , M s , A s , U , M s , R R R R R R R R R R R

39
• (Forced variable): T L1 R

• Remaining variables: T L2 , F L , T s , F s , P s , Q s , F c
R R R R R R R R R R R R R R

• Number of equations: 5 (Eq. 2.64, 2.66, 2.67, 2.69, 2.70)

The degrees of freedom is therefore 7 – 5 = 2. The extra relations are given by the
relation between the steam flow rate F s with the pressure P s either in open-loop or
R R R R

closed-loop operations. The liquid flow rate F 1 is usually regulated by a valve. R R

2.1.8 Single Stage Heterogeneous Systems: Multi-component flash drum


The previous treated examples have discussed processes that occur in one single
phase. There are several chemical unit operations that are characterized with more
than one phase. These processes are known as heterogeneous systems. In the
following we cover some examples of these processes. Under suitable simplifying
assumptions, each phase can be modeled individually by a macroscopic balance.
A multi-component liquid-vapor separator is shown in figure 2.9. The feed consists
of N c components with the molar fraction z i (i=1,2… N c ). The feed at high
R R R R R R

temperature and pressure passes through a throttling valve where its pressure is
reduced substantially. As a result, part of the liquid feed vaporizes. The two phases
are assumed to be in phase equilibrium. x i and y i represent the mole fraction of
R R R R

component i in the liquid and vapor phase respectively. The formed vapor is drawn
off the top of the vessel while the liquid comes off the bottom of the tank. Taking the
whole tank as our system of interest, a model of the system would consist in writing
separate balances for vapor and liquid phase. However since the vapor volume is
generally small we could neglect the dynamics of the vapor phase and concentrate
only on the liquid phase.

40
Fv
yi
P, T, Vv
Fo
zi
To
Po VL ρL FL
xi

Figure 2.9 Multicomponent Flash Drum

For liquid phase:


Total mass balance:
d ( ρ LVL )
= ρ f F f − ρ L FL − ρ v Fv (2.71)
dt
Component balance:
d ( ρ LV L x i )
= ρ f F f z i − ρ L FL x i − ρ v Fv y i (i=1,2,….,N c -1)
R R
(2.72)
dt
Energy balance:
~
d ( ρ LVL h ) ~ ~ ~
= ρ f Ff h f − ρ L FL h − ρv Fv H
dt (2.73)
~ ~
where h and H are the specific enthalpies of liquid and vapor phase respectively.
In addition to the balance equations, the following supporting thermodynamic
relations can be written:

• Liquid-vapor Equilibrium:
Raoult's law can be assumed for the phase equilibrium
xi Pi s
yi = (i=1,2,….,N c )
(2.74)
R R

P
Together with the consistency relationships:
Nc

∑y
i =1
i =1
(2.75)
Nc

∑x
i =1
i =1
(2.76)

41
• Physical Properties:
The densities and enthalpies are related to the mole fractions, temperature and
pressure through the following relations:
ρL R R = f(x i ,T,P)
R R (2.77)
ρv R R = f(y i ,T,P) ≈ M v aveP/R T
R R R RP P (2.78)
Nc
(2.79)
M v ave =
R RP P

∑y M
i =1
i i

Nc
(2.80)
h = f(x i ,T) ≈
R R

∑ x Cp (T − T
i =1
i i ref )

Nc
(2.81)
H = f(y i ,T) ≈
R R

∑ y Cp (T − T
i =1
i i ref ) + λm

Nc
(2.82)
λm R R = ∑yλ
i =1
i i

Degrees of freedom analysis:


• Forcing variables: F f , T f , P f , z i (i=1,2..N c ), R R R R R R R R R R

• Remaining variables:2N c +5: V L , F L , F V , P, T, x i (i=1,2..N c ), y i (i=1,2,…N c ) R R R R R R R R R R R R R R R R

• Number of equations: 2N c +3: (Eq. 2.71, 2.72, 2.73, 2.74, 2.75, 2.76) R R

Note that physical properties are not included in the degrees of freedom since they
are specified through given relations. The degrees of freedom is therefore (2N c +5)- R R

(2N c +3)=2. Generally the liquid holdup (V L ) is controlled by the liquid outlet flow
R R R R

rate (F L ) while the pressure is controlled by F V . In this case, the problem becomes
R R R R

well defined for a solution.

2.1.9 Multistage Heterogeneous Systems: Liquid-liquid extraction

There are many chemical processes which consist in a number of consecutive


stages in series. In each stage two streams are brought in contact for separating
materials due to mass transfer. The two streams could be flow in co-current or
counter current patterns. Counter-current flow pattern is known to have higher
separation performance Examples of these processes are distillation columns,

42
absorption towers, extraction towers and multi-stage flash evaporator where distillate
water is produced from brine by evaporation.
The same modeling approach used for single stage processes will be used for the
staged processes, where the conservation law will be written for one stage and then
repeated for the next stage and so on. This procedure will result in large number of
state equation depending on the number of stages and number of components.
The separation process generally takes place in plate, packed or spray-type towers.
In tray or spay-type columns the contact and the transfer between phases occur at the
plates. Generally, we can always assume good mixing of phases at the plates, and
therefore macroscopic balances can be carried out to model these type of towers.
Packed towers on the other hand are used for continuous contacting of the two phases
along the packing. The concentrations of the species in the phases vary obviously
along the tower. Packed towers are therefore typical examples of distributed
parameters systems that need to be modeled by microscopic balances.
In the following we present some examples of mass separation units that can be
modeled by simple ODE's, and we start with liquid-liquid extraction process.

Liquid-liquid extraction is used to move a solute from one liquid phase to another.
Consider the single stage countercurrent extractor, shown in Figure 2.10, where it is
desired to separate a solute (A) from a mixture (W) using a solvent (S). The stream
mixture with flow rate W (kg/s) enters the stage containing X Af weight fraction of
R R

solute (A). The solvent with a flow rate (S) (kg/s) enters the stage containing Y AfR R

weight fraction of species (A). As the solvent flows through the stage it retains more
of (A) thus extracting (A) from the stream (W). Our objective is to model the
variations of the concentration of the solute. A number of simplifying assumptions
can be used:
• The solvent is immiscible in the other phase.
• The concentration X A and Y A are so small that they do not affect the mass flow
R R R R

rates. Therefore, we can assume that the flow rates W and S are constant. A total
mass balance is therefore not needed.

43
• An equilibrium relationship exists between the weight fraction Y A of the solute R R

in the solvent (S) and its weight fraction X A in the mixture (W). The relationship R R

can be of the form:


Y A = K XA
R R R (2.83)
Here K is assumed constant. Since both phases are assumed perfectly mixed a
macroscopic balance can be carried out on the solute in each phase. A component
balance on the solute in the solvent-free phase of volume V 1 and density ρ 1 gives: R R R R

dX A
ρ1V1 = WX Af − WX A − N A (2.84)
dt
whereas N A (kg/s) is the flow rate due to transfer flow between the two phases. A
R R

similar component balance on the solvent phase of volume V 2 and density ρ 2 gives: R R R R

dYA
ρ2V2 = SYAf − SYA + N A (2.85)
dt
Since Y A = K X A and K is constant, the last equation is equivalent to:
R R R R

dX A
ρ 2V2 K = SY Af − SKX A + N A
dt (2.86)
Adding Eq. 2.84 and 2.86 yields:
dX A
( ρ1V1 + ρ 2V2 K ) = WX Af + SY Af − (W + KS ) X A
dt (2.87)
The latter is a simple linear ODE with unknown X A . With the volume V 1 , V 2 and flow R R R R R R

rates W, S known the system is exactly specified and it can be solved if the initial
concentration is known:
X A (t i ) = X Ai
R R R R R (2.88)
Note that we did not have to express explicitly the transferred flux N A . R R

S, YA S, YAf

W, XAf W, xA

Figure 2.10 Single Stage Liquid-Liquid Extraction Unit

The same analysis can be extended to the multistage liquid/liquid extraction units
as shown in Figure 2.11. The assumptions of the previous example are kept and we

44
also assume that all the units are identical, i.e. have the same volume. They are also
assumed to operate at the same temperature.

V1, y1 V2, y2 V3, y3 Vi, yi Vi+1, yi+1 VN, yN VN+1, yf

1 2 i N

L0, xf L1, x1 L2, x2 Li-1, xi-1 Li, xi LN-1, xN-1 LN, xN

Figure 2.11 Multi-Stage Liquid-Liquid Extraction Unit

A component balance in the ith stge (excluding the first and last stage) gives:
P P

• Solvent-free phase, of volume V 1 i and density ρ 1 i R RR R R RR

ρ1iV1i
dX Ai
= WX Ai −1 − WX Ai − N Ai (i=2…,N-1)
(2.89)
dt
where N Ai is the flow rate due to transfer between the two phases at stage i.
R R

• Solvent phase of volume V 2 i and density ρ 2 i : R RR R R RR R

ρ 2iV2i
dY Ai
= SY Ai +1 − SY Ai + N Ai … (i = 2…,N-1)
(2.90)
dt
Writing the equilibrium equation (Eq. 2.83) for each component Y Ai = K X Ai R R R

R (i=1,.,.N) and adding the last two equations yield:

(ρ1iV1i + ρ2iV2i K )
dX Ai
= WX Ai −1 + SYAi +1 − (W + KS ) X Ai (i=2…,N-1)
(2.91)
dt
Since the volume and densities are equal, i.e.:
V 1i = V 1 and V 2i = V
R R R R R R (2.92)
ρ 1i = ρ 1 and ρ 2i = ρ 2
R R R R R R R
(2.93)
Equation 2.91 is therefore equivalent to:
dX Ai
(ρ1V1 + ρ2V2 K ) = WX Ai −1 + SYAi +1 − (W + KS ) X Ai (i=2…,N-1)
dt (2.94)
The component balance in the first stage is:
dX A1
(ρ1V1 + ρ2V2 K ) = WX Af + SYA1 − (W + KS ) X A1
dt (2.95)
And that for the last stage is:

45
dX AN
(ρ1V1 + ρ2V2 K ) = WX AN −1 + SYAf − (W + KS ) X AN
dt (2.96)
The model is thus formed by a system of linear ODE's (Eq. 2.94, 2.95, 2.96) which
can be integrated if the initial conditions are known:
X A (t i ) = X Ai (i=1,2…,N)
R R R R R R (2.97)

Degrees of freedom analysis


• Parameter of constant values: ρ 1 , ρ 2, K, V 1 , V 2 , W and S R R R R R R R R

• (Forced variable): X Af, Y Af


R R R

• Remaining variables: X Ai (2N variables): (i=1,2…,N) and Y Ai (i=1,2…,N)


R R R R

• Number of equations: 2N [2.83 (N equations, one for each component), Eq.


2.94 (N-2 eqs), 2.95(1 eq), 2.96(1 eq)].
The problem is therefore is exactly specified.

2.10 Binary Absorption Column


Consider a N stages binary absorption tower as shown in figure 2.12. A Liquid
stream flows downward with molar flow rate (L) and feed composition (x f ). A Vapor R R

stream flows upward with molar flow rate (G) and feed composition (y f ). We are R R

interested in deriving an unsteady state model for the absorber. A simple vapor-liquid
equilibrium relation of the form of:
yi = a xi + b
R R R R (2.98)
Can be used for each stage i (i=1,2,…,N).

Assumptions:
• Isothermal Operation
• Negligible vapor holdup
• Constant liquid holdup in each stage
• Perfect mixing in each stage
According to the second and third assumptions, the molar rates can be considered
constants, i.e. not changing from one stage to another, thus, total mass balance need

46
not be written. The last assumption allows us writing a macroscopic balance on each
stage as follows:
Component balance on stage i:
dxi
H = G ( yi −1 − yi ) + L( xi +1 − xi ) (i=2…,N-1) (2.99)
dt
where H is the liquid holdup, i.e., the mass of liquid in each stage. The last equation
is repeated for each stage with the following exceptions for the last and the first
stages:
L, xf G, yN

stage N

xN yN-1

xi+1 yi

stage i
xi yi-1

x3 y2

stage 2

x2 y1

stage 1

L, x1 G, yf

Figure 2.12 N-stages Absorbtion Tower

In the last stage, x i+1 is replaced by x f


R R R

In the first stage, y i-1 is replaced by y f


R R R

Degrees of freedom analysis


• Parameter of constant values: Η, a, b
• (Forced variable): G, L, x f , y f R R R

• Remaining variables: x i (i=1,2…,N), y i (i=1,2…,N)


R R R R

• Number of equations:2N (Eqs.2.98, 2.99)


The problem is therefore is exactly specified.
47
2.11 Multi-component Distillation Column
Distillation columns are important units in petrochemical industries. These units
process their feed, which is a mixture of many components, into two valuable
fractions namely the top product which rich in the light components and bottom
product which is rich in the heavier components. A typical distillation column is
shown in Figure 2.13. The column consists of n trays excluding the reboiler and the
total condenser. The convention is to number the stages from the bottom upward
starting with the reboiler as the 0 stage and the condenser as the n+1 stage.

Description of the process:


The feed containing nc components is fed at specific location known as the feed tray
(labeled f) where it mixes with the vapor and liquid in that tray. The vapor produced
from the reboiler flows upward. While flowing up, the vapor gains more fraction of
the light component and loses fraction of the heavy components. The vapor leaves the
column at the top where it condenses and is split into the product (distillate) and
reflux which returned into the column as liquid. The liquid flows down gaining more
fraction of the heavy component and loses fraction of the light components. The
liquid leaves the column at the bottom where it is evaporated in the reboiler. Part of
the liquid is drawn as bottom product and the rest is recycled to the column. The loss
and gain of materials occur at each stage where the two phases are brought into
intimate phase equilibrium.

48
Cw

D
xd

F
z

steam

B
xb

Figure 2.13 Distillation Column

Modeling the unit:


We are interested in developing the unsteady state model for the unit using the
flowing assumptions:
• 100% tray efficiency
• Well mixed condenser drum and reboiler.
• Liquids are well mixed in each tray.
• Negligible vapor holdups.
• liquid-vapor thermal equilibrium

Since the vapor-phase has negligible holdups, then conservation laws will only be
written for the liquid phase as follows:

Stage n+1 (Condenser), Figure 2.14a:


Total mass balance:
dM D
= Vn − ( R + D)
dt (2.100)
Component balance:
49
d ( M D xD , j ) (2.101)
= Vn yn , j − ( R + D ) x D , j j = 1, nc − 1
dt
Energy balance:
d ( M D hD )
= Vn hn − ( R + D )hD − Qc
(2.102)
dt
Note that R = L n+1 and the subscript D denotes n+1
R R

Stage n, Figure fig2.19b


Total Mass balance:
dM n
= Vn −1 − Vn + R − Ln
(2.103)
dt
Component balance:
d (M n xn, j ) (2.104)
= Vn −1 y n −1, j − Vn y n , j + Rx D , j − Ln x n , j j = 1, nc − 1
dt
Energy balance:
d ( M n hn )
= Vn −1H n −1 − Vn H n + RhD − Ln hn
(2.105)
dt

Stage i, Figure 2.14c


Total Mass balance:
dM i
= Vi −1 − Vi + Li +1 − Li
(2.106)
dt
Component balance:
d ( M i xi , j ) (2.107)
= Vi −1 yi −1, j − Vi yi , j + Li +1 xi +1, j − Li xi , j j = 1, nc − 1
dt
Energy balance:
d ( M i hi )
= Vi −1H i −1 − Vi H i + Li +1hi +1 − Li hi
(2.108)
dt

Stage f (Feed stage), Figure 2.14d


Total Mass balance:
dM f (2.109)
= V f −1 − (V f + (1 − q ) F ) + L f +1 − ( L f + qF )
dt
Component balance:
50
d (M f x f , j ) (2.110)
= V f −1 y f −1, j − (V f y f , j + (1 − q) Fz j ) + L f +1 x f +1, j − ( L f x f , j + qFz j )
dt
j = 1, nc − 1

Energy balance:
d (M f hf ) (2.111)
= V f −1H f −1 − (V f H f + (1 − q) Fh f ) + L f +1h f +1 − ( L f h f + qFh f )
dt

Stage 1, Figure 2.14e


Total Mass balance:
dM 1
= VB − V1 + L2 − L1 (2.112)
dt
Component balance:
d ( M 1 x1, j )
= VB y B , j − V1 y1, j + L2 x2, j − L1 x1, j j = 1, nc − 1 (2.113)
dt

Energy balance:
d ( M 1h1 )
= VB H B − V1H1 + L2 h2 − L1h1 (2.114)
dt

Stage 0 (Reboiler), Figure 2.14f


Total Mass balance:
dM B
= −VB + L1 − B (2.115)
dt
Component balance:
d ( M B xB , j )
= −VB y B , j + L1 x1, j − Bx B , j j = 1, nc − 1 (2.116)
dt
Energy balance:
d ( M B hB )
= −VB H B + L1h1 − BhB + Qr (2.117)
dt
Note that L 0 = B and B denotes the subscript 0
R R

Additional given relations:


Phase equilibrium: y j = f (x j , T,P)
R R R R

51
Liquid holdup: M i = f (L i ) R R R R

Enthalpies: H i = f (T i , y i,j ), h i = f (T i , x i,j ) R R R R R R R R R R R R

Vapor rates: V i = f (P) R R

Notation:
Li, Vi
R R R R Liquid and vapor molar rates
Hi, hi R R R R Vapor and liquid specific enthalpies
xi, yi
R R R R Liquid and vapor molar fractions
Mi R R Liquid holdup
Q Liquid fraction of the feed
Z Molar fractions of the feed
F Feed molar rate

Degrees of freedom analysis


Variables

Mi R R n
MB, MD R R R R 2
Li R R n
B,R,D 3
x i,j
R R n(nc − 1)
x B,j ,x D,j
R R R 2(nc − 1)
y i,j
R R n(nc − 1)
y B,j
R R nc − 1
hi R R n
hB, hDR R R R 2
Hi R R n
HB R R 1
Vi R R n
VB R R 1
Ti R R n
52
TD, TB R R R R 2

Total 11+6n+2n(nc−1)+3(nc−1)

Equations:
Total Mass n+2
Energy n+2
Component (n + 2)(nc − 1)
Equilibrium n(nc − 1)
Liquid holdup n
Enthalpies 2n+2
Vapor rate n
hB = h1 R R R 1
yB = xB
R R R (nc − 1)
Total 7+6n+2n(nc-1)+3(nc-1)

Constants: P, F, Z

Therefore; the degree of freedom is 4

To well define the model for solution we include four relations imported from
inclusion of four feedback control loops as follows:
• Use B, and D to control the liquid level in the condenser drum and in the re-
boiler.
• Use V B and R to control the end compositions i.e., x B , x D
R R R R R

53
R, xd Vn, yn
Vn, yn

stage n
(a)
Qc (b)

R, xd D, xd
Ln, xn Vn-1, yn-1

Li+1, xi+1 Vi, yi Lf+1, xf+1 Vf , yf

stage i stage f

(c) (d)

Li, xi Vi-1, yi-1 Lf, xf Vf-1, yf-1

L2, x2 V1, y1 VB, yB


Qr

stage 1
(f)
B, xB
(e)

L1, x1 V B , yB
L1, x1

Figure 2.14 Distillation Column Stages

2.12 Multi-component Batch Distillation Column


For batch distillation column of constant trays holdup, all equation applied for
conventional distillation column could be used to model batch distillation except the
rebolier equations. Figure (2.15) show the reboiler of batch distillation column.

Section 3

Section 2
L2 Figure (2.15) Reboiler of batch distillation column
V1
Reboiler M1

54
Total mass balance.
dM B
= L1 − VB (2.118)
dt
Component balance
d ( M B ⋅ x B ,i )
= L1 x1,i − VB y B ,i (2.119)
dt
Energy balance
d ( M B hB )
= L1h1 − VB H B + Qr (2.120)
dt
Where Q r is heat supplied by reboiler
R R

2.13 Continuous Reactive Distillation Column


Figure (2.16) represents the continuous packed reactive distillation column. Reactive
distillation column can be divided into three different zones: rectifying, reactive and
stripping. The rectifying section and the stripping section serve as the reactant
recovery from product. However, they may not be needed depending on the reaction
and desired separation. In the reactive zone, chemical reaction and distillation occur
simultaneously. Reactive zone also is the place where catalysts, either homogeneous
or heterogeneous are packed. The feed input can be either single or double. Double
feed input can improve the contact between the reactants, resulting more uniformly
distributed reaction in the column.

55
Figure (2.16) continuous packed reactive distillation column.

2.13.1 Model Assumptions


The packed reactive distillation column is vertically divided into a number of
segments. The reboiler and condenser stages are numbered 1 and N, respectively. The
following assumptions were made to simplify the model of continuous reactive
distillation column:-
1. Neglect of vapor holdup and assume total condensation.
2. Perfect mixing on all stages and in all vessels (condenser and reboiler), and the
condenser and the reboiler are treated as equilibrium stages.
3. Ideal vapor phase for all components in mixture.
4. Liquid and vapor phases in thermodynamic phase equilibrium.

1.13.2 Model Equations


All equations applied for conventional distillation could be used to model reactive
distillation except component and heat balance equations in reaction stages. Figure
(2.17) represent reactive stage.

56
Lm+1 Vm

Stage Figure (2.17) Reaction Stage

Lm Vm-1

Total mass balance


dM m
= Lm+1 + Vm−1 − Lm − Vm (2.121)
dt
Component balance
d ( M m x m ,i )
= Lm+1 xm+1,i + Vm−1 y m−1,i − Lm xm ,i − Vm y m ,i + ε iWm Re m ,i (2.122)
dt
Energy balance
d ( M m hm )
= Lm+1hm+1 + Vm−1 H m−1 − Lm hm − Vm H m + Wm Re m ,i ∆H R (2.123)
dt
Where
M : Molar holdup mol
Re : Forward and reverse reaction rate mol/(s. gm catalyst)
W : Catalyst weight gm
ε : Void fraction of the packing -
ΔHR : Heat of reaction j/mol

2.2 Examples of Distributed Parameter Systems

2.2.1 Liquid Flow in a Pipe

Consider a fluid flowing inside a pipe of constant cross sectional area (A) as shown
in Figure 2.18. We would like to develop a mathematical model for the change in the
fluid mass inside the pipe. Let v be the velocity of the fluid. Clearly the velocity
changes with time (t), along the pipe length (z) and also with the radial direction (r).
In order to simplify the problem, we assume that there are no changes in the radial
direction. We also assume isothermal conditions, so only the mass balance is needed.
Since the velocity changes with both time and space, the mass balance is to be carried

57
out on microscopic scale. We consider therefore a shell element of width ∆z and
constant cross section area (A) as shown in Fig. 2.18.
ρ(t,z)
∆z

vo v(t,z)

z=0 z z+∆z z=L

Figure 2.18 Liquid flow in a pipe

Mass into the shell:


ρvA∆t| z R
(2.124)
Where the subscript ( .| z ) indicates that the quantity (.) is evaluated at the distance z.
R R

Mass out of the shell:


ρvA∆t| z+∆z R
(2.125)
Accumulation:
ρA∆z| t+∆t − ρA∆z| t
R R R
(2.126)
Similarly the subscript (.| t ) indicates that the quantity (.) is evaluated at the time t.
R R

The mass balance equation is therefore:


ρvA∆t| z = ρvA∆t| z+∆z + ρA∆z| t+∆t − ρA∆z| t
R R R R R R R
(2.127)

We can check for consistency that the units in each term are in (kg). Dividing Eq.
2.127 by ∆t ∆z and rearranging yields:
(ρA) − (ρA) (ρvA) − (ρvA) (2.128)
t + ∆t t
= z z + ∆z
∆t ∆z

Taking the limit as ∆t  0 and ∆z  0 gives:


∂ (ρA) ∂ (ρvA) (2.129)
=−
∂t ∂z
Since the cross section area (A) is constant, Eq. 2.129 yields:
∂ρ ∂ ( ρv ) (2.130)
=−
∂t ∂z
or

58
∂ρ ∂ ( ρv ) (2.131)
+ =0
∂t ∂z

The equation (2.131) is a partial differential equation (PDE) that defines the variation
of ρ and v with the two independent variables t and z. This equation is known as the
one-dimensional continuity equation. For incompressible fluids for which the density
is constant, the last equation can also be written as:
∂v (2.132)
=0
∂z
This indicates that the velocity is independent of axial direction for one dimensional
incompressible flow.

2.2.2 Velocity profile inside a pipe

We reconsider the flow inside the pipe of the previous example. Our objective is to
find the velocity profile in the pipe at steady state. For this purpose a momentum
balance is needed. To simplify the problem we also assume that the fluid is
incompressible. We will carry out a microscopic momentum balance on a shell with
radius r, thickness ∆r and length ∆z as shown in Fig 2.19.

r
∆r
R
r
∆r z
∆z
P|z P|z+∆z

Figure 2.19 Velocity profile for a laminar flow in a pipe


Momentum in:
(τrz 2πr∆z)| r
R R R
(2.133)
where τ rz is the shear stress acting in the z-direction and perpendicular to the radius r.
R R

Momentum out:
59
(τrz 2πr∆z)| r+∆r
R R R
(2.134)
As for the momentum generation we have mentioned earlier in section 1.8.3 that the
generation term corresponds to the sum of forces acting on the volume which in this
example are the pressure forces, i.e.
(PA)| z – (PA)| z+∆z = P(2πr∆r)| z − P(2πr∆r)| z+∆z
R R R R R R R
(2.135)
There is no accumulation term since the system is assumed at steady state.
Substituting this term in the balance equation (Eq. 1.6) and rearranging yields,
rτrz |r + ∆r −rτrz |r r ( P |z − P |z + ∆z ) (2.136)
=
∆r ∆z
We can check for consistency that all the terms in this equation have the SI unit of
(N/m2). Taking the limit of (Eq. 2.136) as ∆z and ∆r go to zero yields:
P P

d (rτrz ) dP (2.137)
= −r
dr dz
Here we will make the assumption that the flow is fully developed, i.e. it is not
influenced by the entrance effects. In this case the term dP/dz is constant and we
have:
dP P2 − P1 ∆P (2.138)
= =
dz L L
where L is the length of the tube. Note that equation (2.138) is a function of the shear
stress τrz , but shear stress is a function of velocity. We make here the assumption that
R R

the fluid is Newtonian, that is the shear stress is proportional to the velocity gradient:

τ rz = − µ
dv z (2.139)
dr
Substituting this relation in Eq. 2.137 yields:
d rdv z ∆P (2.140)
µ ( )=r
dr dr L
or by expanding the derivative:
d 2 v z 1 dv z ∆P (2.141)
µ( 2
+ )=
dr r dr L
The system is described by the second order ODE (Eq. 2.141). This ODE can be
integrated with the following conditions:
• The velocity is zero at the wall of the tube

60
vz = 0
R R at r = R (2.142)
• Due to symmetry, the velocity profile reaches a maximum at the center of the
tube:
dvz
=0 at r = 0 (2.143)
dr
Note that the one-dimensional distributed parameter system has been reduced to a
lumped parameter system at steady state.

2.2.3 Diffusion with chemical reaction in a slab catalyst


We consider the diffusion of a component A coupled with the following chemical
reaction A  B in a slab of catalyst shown in figure 2.20. Our objective is to
determine the variation of the concentration at steady state. The concentration inside
the slab varies with both the position z and time t. The differential element is a shell
element of thickness ∆z.

Flow of moles A in:


(SN A )| z R R R (2.144)
where S (m2) is the surface area and N A (moles A/s m2) is the molar flux.
P P R R P P

Flow of moles A out:


(SN A )| z+∆zR R R
(2.145)
Rate of generation of A:
−(S∆z)r (2.146)
where r = kC A is the rate of reaction, assumed to be of first order. There is no
R R

accumulation term since the system is assumed at steady state. The mass balance
equation is therefore,
(SN A )| z − (SN A )| z+∆z − (S∆z)kC A =0
R R R R R R R R R R
(2.147)

61
Porous
catalyst
particle
Exterior
surface

∆z
z=0 z=L
Figure 2.20 diffusion with chemical reaction inside a slab catalyst

Dividing equation (2.147) by S∆z results in:


( N A ) | z − ( N A ) | z + ∆z (2.148)
− kC A = 0
∆z

Taking the limit when ∆z  0, the last equation becomes:


dN A
− kC A = 0
(2.149)
dz
The molar flux is given by Fick's law as follows:
dC A (2.150)
N A = −D A
dz
where D A is diffusivity coefficient of (A) inside the catalyst particle. Equation (2.149)
R R

can be then written as follows:


d 2C A (2.151)
DA 2
− kC A = 0
dz
This is also another example where a one-dimensional distributed system is reduced
to a lumped parameter system at steady state. In order to solve this second-order
ODE, the following boundary conditions could be used:
at z = L, C A = C Ao
R R R R (2.152)
at z = 0, dC A /dr = 0 R R (2.153)
The first condition imposes the bulk flow concentration C Ao at the end length of the R R

slab. The second condition implies that the concentration is finite at the center of the
slab.

62
2.2.4 Temperature profile in a heated cylindrical Rod

Consider a cylindrical metallic rod of radius R and length L, initially at a uniform


temperature of T o . Suppose that one end of the rod is brought to contact with a hot
R R

fluid of temperature T m while the surface area of the rod is exposed to ambient
R R

temperature of T a . We are interested in developing the mathematical equation that


R R

describes the variation of the rod temperature with the position. The metal has high
thermal conductivity that makes the heat transfer by conduction significant. In
addition, the rod diameter is assumed to be large enough such that thermal
distribution in radial direction is not to be neglected. The system is depicted by figure
2.21. For modeling we take an annular ring of width ∆z and radius ∆r as shown in
the figure. The following transport equation can be written:

R
∆r
r
∆r

∆z

Figure 2.21 Temperature Distribution In a cylindrical rod

Heat flow in by conduction at z:


q z (2π r ∆r)∆t
R R
(2.154)
Heat flow in by conduction at r:
q r (2π r ∆z)∆t
R R
(2.155)
where q z and q r are the heat flux by conduction in the z and r directions.
R R R R

Heat flow out by conduction at z+∆z:


q z+∆z (2π r ∆r)∆t
R R
(2.156)
Heat flow out by conduction at r+∆r:

63
q r+∆r (2π (r+∆r)∆z)∆t
R R
(2.157)
Heat accumulation:
~ ~
ρ(2π r ∆r ∆z)( h t+∆t − h t ) R R R R
(2.158)
~
Where h is the specific enthalpy. Summing the above equation according to the
conservation law and dividing by (2π ∆r ∆z ∆t), considering constant density, gives:
~ ~
ht + ∆t − ht q − qz + ∆z rqr − rqr + ∆r (2.159)
ρr =r z +
∆t ∆z ∆r

Taking the limit of ∆t, ∆z, and ∆r go to zero yield:


~
∂h ∂q ∂ ( rqr ) (2.160)
ρr
= −r z −
∂t ∂z ∂r
~
Dividing by r and replacing h by C p (T − T ref ), where C p R R is the average heat
capacity, and substituting the heat fluxes q with their corresponding relations (Fourier
law):
∂T (2.161)
qz = −k z
∂z
and
∂T (2.162)
qr = −kr
∂r
Equation (2.160) is then equivalent to:
∂T ∂ 2T k ∂ ∂T (2.163)
ρC p = kz 2 + r (r )
∂t ∂z r ∂r ∂r
This is a PDE where the temperature depends on three variables: t, z, and r. If we
assume steady state conditions then the PDE becomes:
∂ 2T k r ∂ ∂T (2.164)
0 = kz + (r )
∂z 2 r ∂r ∂r
If in addition to steady state conditions, the radius of the rod is assumed to be small
so that the radial temperature gradient can be neglected then the PDE (Eq. 2.164) can
be further simplified. In this case, the differential element upon which the balance
equation is derived is a disk of thickness ∆z and radius R. The heat conduction in the
radial direction is omitted and replaced by the heat transfer through the surface area
which is defined as follows:
Q = U(2πRL)(T a – T) R R
(2.165)

64
Consequently, the above energy balance equation (Eq. 2.165) is reduced to the
following ODE
d 2T (2.166)
0 = kz − U ( 2πRL)(Ta − T )
dz 2

2.2.5 Isothermal Plug Flow Reactor


Let consider a first-order reaction occurring in an isothermal tubular reactor as
shown in figure 2.22. We assume plug flow conditions i.e. the density, concentration
and velocity change with the axial direction only. Our aim is to develop a model for
the reaction process in the tube.
∆z

v(t,z)
CAo CA

z z+∆z z=L

ρ(t,z), CA(t,z)

Figure 2.22 Isothermal Plug flow reactor

In the following we derive the microscopic component balance for species (A) around
differential slice of width ∆z and constant cross-section area (S).

Flow of moles of A in:


As has been indicated in section 1.11.1 mass transfer occurs by two mechanism;
convection and diffusion. The flow of moles of species A into the shell is therefore
the sum of two terms:
(vC A S ∆t) | z + (N A S ∆t)| z
R R R R R R R R
(2.167)
Where N A is the diffusive flux of A ( moles of A/m2 s).
R R P P

Flow of moles of A out:


(vC A S ∆t) | z+∆z + (N A S ∆t)| z+∆z
R R R R R R R
(2.168)
Accumulation:

65
(C A S ∆z) | t+∆t − (C A S ∆z) | t
R R R R R R R
(2.169)
Generation due to reaction inside the shell:
− r(S∆z∆t) (2.170)
Where r = k C A is the rate of reaction.
R R

Substituting all the terms in the mass balance equation (Eq. 1.3) and dividing by ∆t
and ∆z gives:
(C A S ) |t + ∆t −(C A S ) |t (vC A S + N A S ) |z −(vC A S + N A S ) |z + ∆z (2.171)
= − kC A S
∆t ∆z

Taking the limit of ∆t  0 and ∆z  0 and omitting S from both sides give the
following PDE:
∂C A ∂vC A ∂N A (2.172)
=− − − kC A
∂t ∂z ∂z

Where N A is the molar flux given by Fick’s law as follows:


R R

N A = − DAB
dC A (2.173)
dz
Where D AB is the binary diffusion coefficient. Equation 2.172 can be then written as
R R

follows:
∂C A ∂ ( vC A ) ∂ 2C A (2.174)
=− + DAb − kC A
∂t ∂z ∂z 2
Expanding the derivatives, the last equation can be reduced to:
∂C A ∂C ∂v ∂ 2C A (2.175)
= −v A − C A + DAb − kC A
∂t ∂z ∂z ∂z 2
This equation can be further simplified by using the mass balance equation for
incompressible fluids (Eq. 2.132). We get then:
∂C A ∂C ∂ 2C A (2.176)
= −v A + DAb − kC A
∂t ∂z ∂z 2
The equation is a PDE for which the state variable (C A ) depends on both t and z.R R

The PDE is reduced at steady state to the following second order ODE,
dC A d 2C A (2.177)
0 = −v + DAb − kC A
dz dz 2
The ODE can be solved with the following boundary conditions (BC):

66
BC1: at z = 0 C A (0) = C A0
R R R (2.178)
BC2: at z = L dC A ( z )
=0
(2.179)
dz
The first condition gives the concentration at the entrance of the reactor while the
second condition indicates that there is no flux at the exit length of the reactor.

2.2.6 Non-Isothermal Plug-Flow reactor


The tubular reactor discussed earlier is revisited here to investigate its behavior
under non-isothermal conditions. The heat of reaction is removed via a cooling jacket
surrounding the reactor as shown in figure 2.23. Our objective is to develop a model
for the temperature profile along the axial length of the tube. For this purpose we will
need to write an energy balance around an element of the tubular reactor, as shown in
Fig.2.23. The following assumptions are made for the energy balance:
Water in

To T
CAo v(t,z) v(t,z)
CA

Water out T(t,z)


CA(t,z)
ρ(t,z)

Figure 2.23 Non-isothermal plug flow reactor

Assumptions:
• Kinetic and potential energies are neglected.
• No Shaft work.
• Internal energy is approximated by enthalpy.
• Energy flow will be due to bulk flow (convection) and conduction.

Under these conditions, the microscopic balance around infinitesimal element of


width ∆z with fixed cross-section area is written as follows:
67
Energy flow into the shell:
As mentioned in Section 1.11.3 the flow of energy is composed of a term due to
convection and another term due to molecular conduction with a flux q z . R R

~
(q z A + v Aρ h )∆t| z
R R R R
(2.180)
Energy flow out of the shell:
~
(q z A + v Aρ h )∆t| z+∆z
R R R R
(2.181)
Accumulation of energy:
~ ~
(ρA h ∆z)| t+∆t − (ρA h ∆z)| t R R R R
(2.182)
Heat generation by reaction:
(−∆H r )kC A A ∆z ∆t R R R R
(2.183)
Heat transfer to the wall:
h t (πD∆z)(T − T w )∆t
R R R R
(2.184)
Where h t is film heat transfer coefficient.
R R

Substituting these equations in the conservation law (equation 1.7) and dividing by
Α∆t ∆z give:
~ ~ ~ ~
( ρh ) |t + ∆t −( ρh ) |t ( ρvh ) | z −( ρvh ) | z + ∆z q z | z −q z | z + ∆z πD
= + − ∆H r kC A − ht ( )(T − Tw ) (2.185)
∆t ∆z ∆z A

Taking the limit as ∆t and ∆z go to zero yields:


~ ~
∂ ( ρh ) ∂ ( ρvh ) ∂qz πD (2.186)
=− − − ∆H r kC A − ht ( )(T − Tw )
∂t ∂z ∂z A
The heat flux is defined by Fourier’s law as follows:
∂T (2.187)
q z = − kt
∂z
~
Where k t is the thermal conductivity. The specific enthalpy ( h ) can be approximated
R R

by:
~
h = C p(T − Tref ) (2.188)

Since the fluid is incompressible it satisfies the equation of continuity (Eq. 2.132).
Substituting these expressions in Eq. 2.186 and expanding gives:
∂T ∂T ∂ 2T πD
ρC p = − ρC pv + kt 2 − ∆H r ko e − E / RT C A − ht ( )(T − Tw ) (2.189)
∂t ∂z ∂z A

68
At steady state this PDE becomes the following ODE,
dT d 2T πD
0 = − ρC pv + kt 2 − ∆H r ko e − E / RT C A − ht ( )(T − Tw ) (2.190)
dz dz A
Similarly to Eq. 2.177 we could impose the following boundary conditions:
B.C1: at z = 0 T(z) = T oR (2.191)
B.C2: at z = L dT ( z )
=0
(2.192)
dz
The first condition gives the temperature at the entrance of the reactor and the second
condition indicates that there is no flux at the exit length of the reactor.

69
Chapter 3: Equations of Change

In the last chapter, we presented examples of microscopic balances in one or two


dimensions for various elementary examples. In this chapter we present the general
balance equations in multidimensional case. The balances, also called equations of
changes can be written in cartesian, cylindrical or spherical coordinates. We will
explicitly derive the balance equations in cartesian coordinates and present the
corresponding equations in cylindrical and spherical coordinates.

3.1 Total Mass balance


Our control volume is the elementary volume ∆x∆y∆z shown in Figure 3.1. The
volume is assumed to be fixed in space. To write the mass balance around the volume
we need to consider the mass entering in the three directions x,y, and z.
(ρvy)y+∆y
(ρvz)z+∆z

∆y

∆x
(ρvx)x+∆x
(ρvx)x

∆z

z (ρvy)y
y
(ρvz)z

Figure 3.1 Total Mass balance in Cartesian coordinates

Mass in:
The mass entering in the x-direction at the cross sectional area (∆y∆z) is
(ρv x )| x ∆y∆z∆t
R R R R (3.1)
The mass entering in the y-direction at the cross sectional area (∆x∆z) is
(ρv y )| y ∆x∆z∆t R R R R (3.2)
The mass entering in the z-direction at the cross sectional area (∆x∆y) is
(ρv z )| z ∆x ∆y ∆t
R R R R
(3.3)

Mass out:
The mass exiting in the x-direction is:
(ρv x )| x+∆x ∆y∆z∆t
R R R R
(3.4)
The mass exiting in the y-direction is:
(ρv y )| y+∆y ∆x∆z∆t
R R R R
(3.5)
The mass exiting in the z-direction is:
(ρv z )| z+∆z ∆x∆y ∆t
R R R R
(3.6)

Rate of accumulation:
The rate of accumulation of mass in the elementary volume is:
(ρ)| t+∆t ∆x ∆y ∆z - (ρ)| t ∆x ∆y ∆z
R R R R
(3.7)

Since there is no generation of mass, applying the general balance equation Eq. 1.2
and rearranging gives:
(ρ| t+∆t − ρ| t ) ∆x ∆y ∆z = (ρv x | x − ρv x | x+∆x )∆y∆z∆t + (ρv y | y
R R R R R R R R R R R R R R R R

(3.8)
− ρv y | y+∆y )∆x∆z∆t + (ρv z | z − ρv z | z+∆z ) ∆x ∆y ∆t
R R R R R R R R R R R R

Dividing the equation by ∆x ∆y ∆z∆t results in:


ρ |t +∆t −ρ |t ρvx | x −ρvx |x+∆x ρv y | y −ρv y | y +∆y ρvz | z −ρvz | z +∆z
= + + (3.9)
∆t ∆x ∆y ∆z

By taking the limits as ∆y,∆x,∆,z and ∆t goes to zero, we obtain the following
equation of change:
∂ρ ∂ρvx ∂ρv y ∂ρvz
=− − − (3.10)
∂t ∂x ∂y ∂z

Expanding the partial derivative of each term yields after some rearrangement:
∂ρ ∂ρ ∂ρ ∂ρ ∂v ∂v ∂v
+ vx + vy + vz = −ρ( x + y + z ) (3.11)
∂t ∂x ∂y ∂z ∂x ∂y ∂z
This is the general form of the mass balance in cartesian coordinates. The equation is
also known as the continuity equation. If the fluid is incompressible then the density
is assumed constant, both in time and position. That means the partial derivatives of ρ
are all zero. The total continuity equation (Eq. 3.11) is equivalent to:
∂vx ∂v y ∂vz
0 = −ρ( + + ) (3.12)
∂x ∂y ∂z

or simply:
∂vx ∂v y ∂vz
0= + + (3.13)
∂x ∂y ∂z

3.2 Component Balance Equation


We consider a fluid consisting of species A, B … , and where a chemical reaction
is generating the species A at a rate r A (kg/m3s). The fluid is in motion with mass-
R R P P

average velocity v = n t /ρ (m/s) where n t = n A + n B + … (kg/m2s) is the total mass flux


R R R R R R R R P P

and ρ (kg/m3s) is the density of the mixture. Our objective is to establish the
P P

component balance equation of A as it diffuses in all directions x,y,z (Figure 3.2).


z nAy|y+∆y
nAz|z

∆y

nAx|x+∆x
nAx|x
∆z
y

∆x

nAy|y nAz|z+∆z
x

Figure 3.2 Mass balance of component A

Mass of A in:
The mass of species A entering the x-direction at the cross sectional (∆y∆z) is:
(n Ax )| x ∆y∆z∆t
R R R R
(3.14)
where n Ax kg/m2 is the flux transferred in the x-direction
R R P P
Similarly the mass of A entering the y and z direction are respectively:
(n Ay )| y ∆x∆z∆t
R R R R
(3.15)
(n Az )| z ∆x∆y∆t
R R R R
(3.16)
Mass of A out:
The mass of species A exiting the x, y and z direction are respectively
(n Ax )| x+∆x ∆y∆z∆t
R R R R
(3.17)
(n Ay )| y+∆y ∆x∆z∆t
R R R R
(3.18)
(n Az )| z+∆z ∆x∆y∆t
R R R R
(3.19)
The rate of accumulation is:
ρ A | t+∆t ∆x ∆y ∆z − ρ A | t ∆x ∆y ∆z
R R R R R R R R
(3.20)

The rate of generation is:


-r A ∆x ∆y ∆z ∆t
R R
(3.21)

Applying the general balance equation (Eq. 1.3) yields:


(ρ A | t+∆t − ρ A | t ) ∆x ∆y ∆z = (n Ax | x+∆x − n Ax | x )∆y∆z∆t + (n Ay | y+∆y −
R R R R R R R R R R R R R R R R R R R R

(3.22)
n Ay | y )∆x∆z∆t + (n Az | z+∆z − n Az | z )∆x∆y∆t +r A ∆x ∆y ∆z∆t
R R R R R R R R R R R R R R

Dividing each term by ∆x∆y∆z∆t and letting each of these terms goes to zero yields:
∂ρ A ∂n Ax ∂n Ay ∂n Az
+ + + = rA (3.23)
∂t ∂t ∂t ∂t
We know from Section 1.11.1, that the flux n A is the sum of a term due to convection R R

(ρ A v) and a term due to diffusion j A (kg/m2s):


R R R R P P

n A = ρA v + j A
R R R R R (3.24)
Substituting the different flux in Eq. 3.23 gives:
∂ρ A ∂ ( ρ A v x ) ∂ ( ρ A v y ) ∂ ( ρ A v z ) ∂j Ax ∂j Ay ∂j Az
+ + + + + + = rA (3.25)
∂t ∂x ∂y ∂z ∂x ∂y ∂z

For a binary mixture (A,B), Fick’s law gives the flux in the u-direction as :
∂wA
j Au = −ρDAB (3.26)
∂u

where w A = ρ A /ρ. Expanding Eq. 3.25 and substituting for the fluxes yield:
R R R R
∂ρ A  ∂v ∂v y ∂v z   ∂ρ A ∂ρ ∂ρ 
+ ρ A  x + +  +  v x + v y A + v z A 
∂t  ∂x ∂y ∂z   ∂x ∂y ∂z 
(3.27)
 ∂ ∂ρD AB w A ∂ ∂ρD AB w A ∂ ∂ρD AB w A 
−  ( )+ ( )+ ( )  = rA
 ∂x ∂x ∂y ∂y ∂z ∂z 

This is the general component balance or equation of continuity for species A. This
equation can be further reduced according to the nature of properties of the fluid
involved. If the binary mixture is a dilute liquid and can be considered
incompressible, then density ρ and diffusivity D AB are constant. Substituting the
R R

continuity equation (Eq. 3.13) in the last equation gives:


∂ρ A  ∂ρ A ∂ρ ∂ρ   ∂ 2ρ A ∂ 2ρ A ∂ 2ρ A 
+  vx + v y A + vz A  − DAB  + + )  = rA (3.28)
∂t  ∂x ∂y ∂z   ∂x 2 ∂y 2 ∂z 2 

This equation can also be written in molar units by dividing it by the molecular
weight M A to yield:
R R

∂C A  ∂C A ∂C A ∂C A   ∂ 2C A ∂ 2C A ∂ 2C A 
+  v x + vy + vz  − D AB  + +  = RA
∂
t  ∂x ∂y ∂z   ∂x 2 ∂ 2
∂ 2   (3.29)
     y

z
 reaction
accumulation Convection Diffusion

The component balance equation is composed then of a transient term, a convective


term, a diffusive term and a reaction term.

3.3 Momentum Balance


We consider a fluid flowing with a velocity v(t,x,y,z) in the cube of Figure 3.3. The
flow is assumed laminar. We know from Section 1.11.2 that the momentum is
transferred through convection (bulk flow) and by molecular transfer (velocity
gradient).
−σyx |y+∆y σzx |z

−σxx |x+∆x
σxx |x

−σzx |z+∆z
σyx |y

x
z

Figure 3.3 Balance of the x-component of the momentum

Since, unlike the mass or the energy, the momentum is a vector that has three
components, we will present the derivation of the equation for the conservation of the
x-component of the momentum. The balance equations for the y-component and the
z-component are obtained in a similar way. To establish the momentum balance for
its x-component we need to consider its transfer in the x-direction, y-direction, and z-
direction.

Momentum in:
The x-component of momentum entering the boundary at x-direction, by
convection is:
(ρv x v x )| x ∆y∆z∆t
R R R R R R
(3.30)
The x-component of momentum entering the boundary at y-direction, by
convection is:
(ρv y v x )| y ∆x∆z∆t
R R R R R R
(3.31)
and it enters the z-direction by convection with a momentum:
(ρv z v x )| z ∆x∆y∆t
R R R R R R
(3.32)
The x-component of momentum entering the boundary at x-direction, by molecular
diffusion is:
(τxx )| x ∆y∆z∆t
R R R R
(3.33)
The x-component of momentum entering the boundary at y-direction, by molecular
diffusion is:
(τyx )| y ∆x∆z∆t R R R R
(3.34)
and it enters the z-direction by molecular diffusion with a momentum:
(τzx )| z ∆x∆y∆t R R R R
(3.35)

Momentum out:
The rate of momentum leaving the boundary at x+∆x, by convection is:
(ρv x v x )| x+∆x ∆y∆z∆t R R R R R R
(3.36)
and at boundary y+∆y,:
(ρv y v x )| y+∆y ∆x∆z∆t
R R R R R R
(3.37)
and at boundary z+∆z:
(ρv z v x )| z+∆z ∆x∆y∆t R R R R R R
(3.38)

The x-component of momentum exiting the boundary x+∆x, by molecular


diffusion is:
(τxx )| x+∆x ∆y∆z∆t
R R R R
(3.39)
and at boundary y+∆y:
(τyx )| y+∆y ∆x∆z∆t
R R R R
(3.40)
and at boundary z+∆z::
(τzx )| z+∆z ∆x∆y∆t
R R R R (3.41)

Forces acting on the volume:


The net fluid pressure force acting on the volume element in the x-direction is:
(P| x – P| x+∆x ) ∆y∆z∆t
R R R R (3.42)
The net gravitational force in the x-direction is:
ρg| x ∆x ∆y∆z ∆t R R (3.43)
Accumulation is:
(ρv x | t+∆t − ρv x | t ) ∆x ∆y∆z
R R R R R R R R (3.44)
Substituting all these equations in Eq. 1.5, dividing by ∆x ∆y ∆z ∆t and taking the
limit of each term goes zero gives:
∂ ( ρv x ) ∂ ( ρv x v x ) ∂ ( ρv x v y ) ∂ ( ρv x v z ) ∂τ ∂τ ∂τ ∂P
+ + + = −( xx + yx + zx ) − + ρg x (3.45)
∂t ∂x ∂y ∂z ∂x ∂y ∂z ∂x

Expanding the partial derivative and rearranging:


 ∂ρ ∂ρv x ∂ρv y ∂ρv z   ∂v ∂v x ∂v x ∂v x 
vx  + + +  + ρ  x + v x + vy + vz  =
 ∂t ∂x ∂y ∂z   ∂t ∂x ∂y ∂z 
∂τ ∂τ yx ∂τ zx ∂P (3.46)
− ( xx + + )− + ρg
∂x ∂y ∂z ∂x x

Using the equation of continuity (Eq. 3.10) for incompressible fluid, Equation (3.46)
is reduced to:
 ∂v x ∂v ∂v ∂v  ∂τ ∂τ ∂τ ∂P
ρ + v x x + v y x + v z x  = −( xx + yx + zx ) − + ρg x (3.47)
 ∂t ∂x ∂y ∂z  ∂x ∂y ∂z ∂x

Using the assumption of Newtonian fluid, i.e.


∂v x ∂v x ∂v x
τ xx = − µ , τ yx = − µ , τ zx = − µ (3.48)
∂x ∂y ∂z

Equation 3.47 yields:


∂v x  ∂v ∂v ∂v   ∂ 2v ∂ 2v ∂ 2 v  ∂P
ρ + ρ  v x x + v y x + v z x  = µ  2x + 2x + 2x  − + ρg x
∂ t
   ∂x ∂y ∂z   ∂ x ∂ y ∂ z  ∂x (3.49)
accumulation
       
generation
transport by bulk flow transport by viscous forces

The momentum balances in the y-direction and z-direction can be obtained in a


similar fashion:
∂v y  ∂v y ∂v y ∂v y   ∂ 2 v y ∂ 2 v y ∂ 2 v y  ∂P
ρ + ρ  v x + vy + vz  = µ  2 + 2 + 2  − + ρg y

 
t ∂x ∂y ∂ z   ∂x ∂y ∂z   ∂
y  
(3.50)
accumulation
    
transport by bulk flow transport by viscous forces generation

∂v z  ∂v ∂v ∂v   ∂ 2v ∂ 2v ∂ 2 v  ∂P
ρ + ρ  v x z + v y z + v z z  = µ  2z + 2z + 2z  − + ρg z
 ∂
t  ∂x ∂y ∂z   ∂x ∂y ∂z   ∂
z   (3.51)
accumulation
      generation
transport by bulk flow transport by viscous forces

These equations constitute the Navier-Stock’s equation.

3.4 Energy balance


In deriving the equation for energy balance we will be guided by the analogy that
exists between mass and energy transport mentioned in Section 1.11.3 We will
assume constant density, heat capacity and thermal conductivity for the
incompressible fluid. The fluid is assumed at constant pressure (Fig 3.4).
The total energy flux is the sum of heat flux and bulk flux:
e = q + ρCpTv (3.52)
Therefore, the energy coming by convection in the x-direction at boundary x is:
(q x + ρCpTv x )∆y∆z∆t
R R R R
(3.53)
Similarly the energy entering the y and z directions are
(q y + ρCpTv y )∆x∆z∆t
R R R R
(3.54)
(q z + ρCpTv z ) ∆x ∆y ∆t
R R R R
(3.55)
The energy leaving the x,y and z directions are:
(q x + ρCpTv x )| x+∆x ∆y∆z∆t
R R R R R R
(3.56)
(q y + ρCpTv y )| y+∆y ∆x∆z∆t
R R R R R R
(3.57)
(q z + ρCpTv z )| z+∆z ∆x ∆y ∆t
R R R R R R
(3.58)
The energy accumulated is approximated by:
(ρCpT| t+∆t − ρCpT| t ) ∆x ∆y∆z R R R R
(3.59)
qy | y
qz |z+∆z

∆y

∆x
qx |x+∆x
qx |x
∆z

qy |y+∆y
qz |z

Figure 3.4 Energy Balance in Cartesian coordinates

The rate of generation is Φ H where Φ H includes all the sources of heat generation, i.e.
R R R R

reaction, pressure forces, gravity forces, fluid friction, etc. Substituting all these terms
in the general energy equation (Eq. 1.7) and dividing the equation by the term
∆x∆y∆z∆t and letting each of these terms approach zero yield:
∂ ( ρCpT ) ∂ ( ρCpTv x ) ∂ ( ρCpTv y ) ∂ ( ρCpTvz ) ∂qx ∂q y ∂qz
+ + + + + + = ΦH (3.60)
∂t ∂x ∂y ∂z ∂x ∂y ∂z

Expanding the partial derivative yields:


 ∂T ∂T ∂T ∂T   ∂ρ ∂ρv x ∂ρv y ∂ρvz  ∂q x ∂q y ∂q z
ρCp + vx + vy + vz  + ρCpT  + + + +
 ∂x + ∂y + ∂z = Φ H (3.61)
 ∂t ∂x ∂y ∂z   ∂t ∂x ∂y ∂z 

Using the equation of continuity (Eq. 3.10) for incompressible fluids the equation is
reduced to:
 ∂T ∂T ∂T ∂T  ∂q x ∂q y ∂q z
ρCp + vx + vy + vz + + + = ΦH (3.62)
 ∂t ∂x ∂y ∂z  ∂x ∂y ∂z

Using Fourier's law:


dT
qu = −k (3.63)
du
into the last equation gives:
∂T  ∂T ∂T ∂T   ∂ 2T ∂ 2T ∂ 2T 
ρCp + ρCp vx + vy + vz  = k  2 + 2 + 2  + Φ H
∂ ∂x ∂y ∂z  
 t
   ∂x ∂y

∂z  generation

(3.64)
accumulation Transport by bulk flow Transport by thermal diffusion

The energy balance includes as before a transient term, a convection term, a diffusion
term, and generation term. For solids, the density is constant and with no velocity, i.e.
v = 0, the equation is reduced to:
∂T  ∂ 2T ∂ 2T ∂ 2T 
ρCp = k  2 + 2 + 2  + Φ 
∂ ∂ ∂y ∂z  generation
H
 t x (3.65)
accumulation
  
Transport by thermal diffusion
Introduction to Numerical Analysis

Analysis versus Numerical Analysis


The word analysis in mathematics usually means who to solve a problem through
equations. The solving procedures may include algebra, calculus, differential
equations, or the like.

Numerical analysis is similar in that problems solved, but the only procedures that
are used are arithmetic: add, subtract, multiply, divide and compare.

Differences between analytical solutions and numerical solutions:

1) An analytical solution is usually given in terms of mathematical functions. The


behavior and properties of the function are often apparent. However, a numerical
solution is always an approximation. It can be plotted to show some of the
behavior of the solution.

2) An analytical solution is not always meaningful by itself.

Example: 3 as one of the roots of x 3 − x 2 − 3x + 3 = 0 .

3) While the numerical solution is an approximation, it can usually be evaluated as


accurate as we need. Actually, evaluating an analytic solution numerically is
subject to the same errors.

Computers and Numerical Analysis

Numerical Methods+Programs Computers


Numerical Analysis

• As you will learn enough about many numerical methods, you will be able to
write programs to implement them.
• Programs can be written in any computer language. In this course all programs
will be written in Matlab environment.
• Actually, writing programs is not always necessary. Numerical analysis is so
important that extensive commercial software packages are available.

Numerical Analysis / M. Sc. -1- Written by Assoc. Prof.


Introduction to Numerical Analysis Dr. Zaidoon M. Shakoor
Types of Equations
The equations is divided into three main categories such as in below figure:-

Kinds of Errors in Numerical Procedures

The total error comprises of:

1) Model Error: due to the mismatch between the physical situation and the
mathematical model.

2) Data Error: due to the measurements of doubtful accuracy.

3) Human Error: due to human blunders.

4) Propagated Error: the error in the succeeding steps of a process due to an


occurrence of an earlier error.

5) Truncation Error: the notion of truncation error usually refers to errors introduced
when a more complicated mathematical expression is “replaced” with a more
elementary formula. This formula itself may only be approximated to the true values,
thus would not produce exact answers.

Numerical Analysis / M. Sc. -2- Written by Assoc. Prof.


Introduction to Numerical Analysis Dr. Zaidoon M. Shakoor
Example:
Truncation of an infinite series to a finite series to a finite number of terms leads to
the truncation error. For example, the Taylor series of exponential function
x 2 x3 xn
ex = 1+ x + + + ... +
2! 3! n!
If only four terms of the series are used, then
x2 x3
e ≈ 1+ x +
x
+
2! 3!
12 13
e ≈ 1+1+ +
1
= 2.66667
2! 3!

The truncation error would be the unused terms of the Taylor series, which then are
x4 x5 14 15
Et = + + = + + ≅ 0.0516152
4! 5! 4! 5!

Check a few Taylor series approximations of the number ex, for x = 1, n = 2, 3 and 4.
Given that e1 = 2.718281.

Approximation Percent relative


Order of n Absolute error
for ex error
2 2.500000 0.218281 8.030111%
3 2.666667 0.051614 1.898774%
4 2.708333 0.00995 0.365967%

6) Round-Off Error: A round-off error, also called rounding error, is the


difference between the calculated approximation of a number and its exact
mathematical value due to rounding

Example:

Numbers such as π, e, or 3 cannot be expressed by a fixed number of decimal

places. Therefore they cannot be represented exactly by the computer.


Consider the number π. It is irrational, i.e. it has infinitely many digits after the
period: π = 3.1415926535897932384626433832795.....
The round-off error computer representation of the number π depends on how many
digits are left out.

Numerical Analysis / M. Sc. -3- Written by Assoc. Prof.


Introduction to Numerical Analysis Dr. Zaidoon M. Shakoor
Let the true value for π is 3.141593.

Number of digits Approximation Percent relative


Absolute error
(Decimal digit) for π error
1 3.1 0.041593 1.3239%
2 3.14 0.001593 0.0507%
3 3.142 0.000407 0.0130%

1.5 Errors in Numerical Procedures


There are two common ways to express the size of the error in a computed result:
absolute error and relative error.

• Absolute error = | true value – approximate value |, which is usually used when the
magnitude of the true value is small.

| true value - approximate value |


• Relative error = , which is a desirable one.
| true value |

While
true value − approximate value
Percent relative error, ε t = × 100%
true value

Numerical Analysis / M. Sc. -4- Written by Assoc. Prof.


Introduction to Numerical Analysis Dr. Zaidoon M. Shakoor
Chapter 1: Solution System of Linear Equations

1.1 Linear Equation


y = mx is an equation, in which variable y is expressed in terms of x and the constant
m, is called Linear Equation. In Linear Equation exponents of the variable is always
‘one’.

1.2 Linear Equation in n variables:


a1 x1 + a2 x2 + a3 x3 + .... + an xn = b
Where x1 , x2 , x3 ,..., xn are variables and
a1 , a2 , a13 ,..., an and b are constants.

1.3 System of Linear Equations:


A Linear System of m linear equations and n unknowns is:
a11 x1 + a12 x2 + a13 x3 + .... + a1n xn = b1
a21 x1 + a22 x2 + a23 x3 + .... + a2 n xn = b2
a31 x1 + a32 x2 + a33 x3 + .... + a3n xn = b3
.......................................................
........................................................
am1 x1 + am 2 x2 + am3 x3 + .... + amn xn = bm
Where x1 , x2 , x3 ,..., xn are variables or unknowns and a’s and b’s are constants.

1.4 Augmented Matrix


System of linear equations:
a11 x1 + a12 x2 + a13 x31 = b1
a21 x1 + a22 x2 + a23 x31 = b2
a31 x1 + a32 x2 + a33 x31 = b3
Can be written in the form of matrices product
 a11 a12 a13   x1   b1 
a
 21 a22 a23   x2  = b2 
 a31 a32 a33   x3  b3 
Or we may write it in the form AX=b,
 a11 a12 a13   x1   b1 
Where A= a21 a22 a23  , X =  x2  , b = b2 
 a31 a32 a33   x3  b3 
 a11 a12 a13 b1 
Augmented matrix is [A : b] = a21 a22 a23 b2 
 a31 a32 a33 b3 
Numerical Analysis / M. Sc. -5- Written by Assoc. Prof.
Chap. 1 Dr. Zaidoon M. Shakoor
Example 1.1:
Write the matrix and augmented form of the system of linear equations
3x – y + 6z = 6
x+y+z=2
2x + y+4z = 3
Solution: Matrix form of the system is
 3 − 1 6   x  6 
1 1 1   y  =  2 
    
2 1 4  z  3
 3 − 1 6 6
Augmented form is [A : b] = 1 1 1 2 
2 1 4 3

1.5 Direct Methods for Solving System of Linear Equations


1.5.1 Gaussian Elimination.
Gaussian elimination is a general method of finding possible solutions to a linear
system of equations.
Gaussian Elimination Method
Step 1. By using elementary row operations
 a11 a12 a13 b1  1 A12 A13 B1 
a
 21 a22 a23 b2  → 0 1 A23 B2 
 a31 a32 a33 b3  0 0 1 B3 
Step 2. Find solution by back – substitutions.

Example 1.2:
Solve the system of linear equations by Gaussion- Elimination method
 x1 + x2 + x3 = 3

 2 x1 − x2 − 2 x3 = 6
4 x + 2 x + 3x =
 1 2 3 7

Solution:
Step 1.
Augmented matrix is
1 1 1 3 R2 = r2 − 2r1
  R3 = r3 − 4r1
 2 − 1 − 2 6 
4 2 3 7 

Numerical Analysis / M. Sc. -6- Written by Assoc. Prof.


Chap. 1 Dr. Zaidoon M. Shakoor
1 1 1 3
  R3 = 3r3 − 2r1
0 − 3 − 4 0 
0 − 2 − 1 − 5

1 1 1 3 
  R2 = −r2
 0 − 3 − 4 0 
r3
0 0 5 − 15 R3 =
5
1 1 1 3 
 
 0 3 4 0 
0 0 1 − 3
Equivalent system of equations form is:

Step 2. Back Substitution


x1 + x2 + x3 = 3 x3 = −3
3 x2 + 4 x3 = 0 ⇒ x2 = −4 x3 / 3 = 12 / 3 = 4
x3 = −3 x1 = 3 − x2 − x3 = 3 − 4 + 3 = 2
Solutions are x1 = 2, x2 = 4, x3 = −3

Example 1.3:
For the below figure calculate the values of the unknown flow rates F 1 , F 2 and F 3 by
using Gaussion- Elimination method

. F1=? F2=?
99% Benzene 5% Benzene
1% Toluene 92% Toluene
3 % Xylene
Tower 1

Tower 2

F=1000 kg/hr

40% Benzene
40% Toluene
20% Xylene
F3=?
10% Toluene
90% Xylene

Component material balance gives these three equations of three variables


F1 + F2 + F3 = 1000
0.99 F1 + 0.05 F2 + 0 F3 = 400
0.01F1 + 0.92 F2 + 0.1F3 = 400
Numerical Analysis / M. Sc. -7- Written by Assoc. Prof.
Chap. 1 Dr. Zaidoon M. Shakoor
Augmented matrix is
 1 1 1 1000 
0.99 0.05 0 400  R 2 =r 2 – (0.99/1)×r 1
 
 0.01 0.92 0.1 400 R 3 =r 3 – (0.01/1)×r 1
1 1 1 1000 
0 − 0.94 − 0.99 - 590
  R 3 =r 3 -(0.91/(-0.94))r 2
0 0.91 0.09 390 
1 1 1 1000 
0 − 0.94
 − 0.99 - 590  R 2 =r 2 /(-0.94)
0 0 - 0.8684 - 181.17  R 3 =r 3 /( -0.8684)
1 1 1 1000 
0 1 1.0532 627.6596
 
0 0 1 208.6253 

Equivalent system of equations form is:


F 1 + F 2 + F 3 = 1000
F 2 +1.0532F 3 = 627.6596
F 3 = 208.6253

Step 2. Back Substitution


F 3 = 208.6253
F 2 = 627.6596-1.0532F 3 =627.6596-1.0532×208.6253 =407.9354
F 1 =1000- F 2 -F 3 =1000 -208.6253 - 407.9354= 383.4393

1.5.2 Gauss - Jorden Elimination Method

Gauss - Jorden Method


By using elementary row operations
 a11 a12 a13 b1  1 0 0 B1 
a
 21 a22 a23 b2  → 0 1 0 B2 

 a31 a32 a33 b3  0 0 1 B3 

Example 1.4:
Solve the system of linear equations by Gauss-Jorden elimination method
x 1 + x 2 + 2x 3 = 8
- x 1 - 2x 2 + 3x 3 = 1
3x 1 -7x 2 + 4x 3 = 10

Numerical Analysis / M. Sc. -8- Written by Assoc. Prof.


Chap. 1 Dr. Zaidoon M. Shakoor
Solution:
Augmented matrix is
1 1 2 8
− 1 − 2 3 1  R 2 = r 2 +r 1
 
 3 − 7 4 10 R 3 = r 3 -3r 1

1 1 2 8 
0 − 1
 5 9  R 2 =-r 2
0 − 10 − 2 - 14 R 3 =r 3 -10r 2
1 1 2 8 
0 1 − 5
 - 9  R 3 =-r 3 /52
0 0 − 52 - 104

1 1 2 8 R 1 =r 1 -2r 3
0 1 − 5 − 9  R 2 =r 2 +5r 3
 
0 0 1 2 
1 1 0 4
0 1 0 1  R 1 =r 1 -r 2
 
0 0 1 2
1 0 0 3
0 1 0 1 
 
0 0 1 2
Equivalent system of equations form is:
x1 = 3
x2 = 1
x3 = 2
is the solution of the system.

Example 1.5:
Total and component material balance on a system of distillation columns gives the
flowing equations:-
F1 + F2 + F 3 + F 4 = 1690
0.4F 1 + 0.15F 2 + 0.25F 3 + 0.2F 4 = 412.5
0.25F 1 + 0.8F 2 + 0.3F 3 + 0.45F 4 = 701
0.08F 1 + 0.05F 2 + 0.45F 3 + 0.3F 4 = 487.3
Use Gauss - Jorden method to compute the four un-known's in above equations:-

Numerical Analysis / M. Sc. -9- Written by Assoc. Prof.


Chap. 1 Dr. Zaidoon M. Shakoor
Solution:
Augmented matrix is
 1 1 1 1 1690 
 0.4 0.15 0.25 0.2 412.5 R 2 = r 2 -(0.4/1)r 1
 
0.25 0.8 0.3 0.45 701  R 3 =r 3 -(0.25/1)r 1
  R 4 =r 4 -(0.08/1)r 1
0.08 0.05 0.45 0.3 487.3
1 1 1 1 1690 
0 − 0.25 − 0.15 − 0.2 − 263.5
 
0 0.55 0.05 0.2 278.5  R 3 =r 3 -(0.55/(-0.25))r 2
  R 4 =r 4 -((-0.03)/(-0.025))r 2
0 − 0.03 0.37 0.22 352.1 
1 1 1 1 1690 
0 − 0.25 − 0.15 − 0.2 − 263.5
 
0 0 − 0.28 − 0.24 − 301.2
  R 4 =r 4 -((0.0388)/(-0.028))r 3
0 0 0.388 0.244 383.72 
1 1 1 1 1690 
0 − 0.25 − 0.15 − 0.2 − 263.5 
 R 2 = r 2 /(-0.25)
0 0 − 0.28 − 0.24 − 301.2  R 3 =r 3 /(-0.028)
 
0 0 0 − 0.08857 − 33.657 R 4 =r 4 /(-0.0887)
1 1 1 1 1690  R 1 = r 1 -r 4
0 1 0.6 0.8 1054  R 2 =r 2 -(0.8/1)r 4

0 0 1 0.85714 1075.74 R 3 =r 3 -(0.85714/1)r 4
 
0 0 0 1 380 
1 1 1 0 1310 R 1 = r 1 -r 3
0 1 0.6 0 750  R 2 =r 2 -((-0.6)/1)r 3

0 0 1 0 750 
 
0 0 0 1 380 
1 1 0 0 560
0 1 0 0 300 R 1 =r 1 -r 2

0 0 1 0 750
 
0 0 0 1 380
1 0 0 0 260 Equivalent system of equations form:
0 1 0 0 300  F 1 = 260 , F 2 = 300 , F 3 = 750 and

0 0 1 0 750 F 4 = 380 is the solution of the system.
 
0 0 0 1 380 

Numerical Analysis / M. Sc. - 10 - Written by Assoc. Prof.


Chap. 1 Dr. Zaidoon M. Shakoor
1.6 In-direct (iterative) Methods for Solving System of Linear Equations
The sufficient condition for solving systems of linear equations using iterative
methods is:
a1 ≥ b1 + c1
b2 ≥ a2 + c2
c3 ≥ a3 + b3
The absolute value of the diagonal element in each row of the coefficient matrix must
be greater than the sum of the absolute values of the off-diagonal elements in the
same row.

1.6.1 Jacobi Method


Let the given equation be
a2 x + b1 y + c1 z = d1
a2 x + b2 y + c2 z = d 2
a3 x + b3 y + c3 z = d 3
If the given system of equation is diagonally dominant then
x (i +1) =
1
a1
(
d1 − b1 y (i ) − c1 z (i ) )
y (i +1) =
1
b2
(
d 2 − a2 x (i ) − c2 z (i ) )
z (i +1) =
1
c3
(
d 3 − a3 x (i ) − b3 y (i ) )

Example 1.6:
Use the Jacobi iteration method to obtain the solution of the following equations:
6x 1 -2 x 2 + x 3 = 11
x 1 +2x 2 -5x 3 = -1
-2x 1 +7 x 2 +2x 3 = 5

Solution
Step 1: Re-write the equations such that each equation has the unknown with largest
coefficient on the left hand side:
6x 1 = 11+2 x 2 -x 3
7x 2 = 5+2x 1 -2x 3
5x 3 =1+x 1 +2x 2
2 x2 − x3 + 11
x1 =
6

Numerical Analysis / M. Sc. - 11 - Written by Assoc. Prof.


Chap. 1 Dr. Zaidoon M. Shakoor
2 x1 − 2 x3 + 5
x2 =
7
x + 2 x2 + 1
x3 = 1
5
Step 2: Assume the initial guesses x1 = x2 = x3 = 0 then calculate x1 , x2 and x3 :
0 0 0 1 1 1

2( x20 ) − ( x30 ) + 11 2(0) − (0) + 11


x11 = = =1.833
6 6
2( x10 ) − 2( x30 ) + 5 2(0) − 2(0) + 5
x12 = = = 0.714
7 7
( x10 ) + 2( x20 ) + 1 (0) + 2(0) + 1
x31 = = = 0.200
5 5
Step 3: Use the values obtained in the first iteration, to calculate the values for the 2nd
iteration:
2( x12 ) − ( x3`1 ) + 11 2(0.714) − (0.200) + 11
x12 = = = 2.038
6 6
2( x11 ) − 2( x31 ) + 5 2(1.833) − 2(0.200) + 5
x22 = = =1.181
7 7
( x11 ) + 2( x12 ) + 1 (1.833) + 2(0.714) + 1
x32 = = = 0.852
5 5
and so on for the next iterations so that the next values are calculated using the
current values:
2( x2i ) − ( x3i ) + 11
x1i +1 =
6
2( x1i ) − 2( x3i ) + 5
x2i +1 =
7
( x1i ) + 2( x2i ) + 1
x3i +1 =
5
The results for 9 iterations are:
Unknowns
Iter. x1 x2 x3
1 1.833 0.714 0.200
2 2.038 1.181 0.852
3 2.085 1.053 1.080
4 2.004 1.001 1.038
. . . .
. . . .
9 2.000 1.000 1.000

Numerical Analysis / M. Sc. - 12 - Written by Assoc. Prof.


Chap. 1 Dr. Zaidoon M. Shakoor
Example 1.7:
Solve the equations by Jacobi method
20x 1 + x 2 – 2x 3 = 17
3x 1 + 20x 2 – x 3 = –18
2x 1 – 3x 2 + 20x 3 = 25
Solution
Rewrite the given equation in the form:
1
x1i +1 = (17 − x2i + 3x3i )
20
1
x2i +1 = (−18 − 3x1i + x3i )
20
1
x3i +1 = (25 − 2 x1i + 3x2i )
20
Using x=
0
1
x=
0
2
x 30 = 0, we obtain
17
x=
1
1
= 0.85
20
−18
x12 = = −0.90
20
25
x=
1
3
= 1.25
20
Putting these values on the right of equations to obtain
(17 − x12 − 2x=3)
1
x=
2
1
1
1.02
20
1
(
x 22 = −18 − 3x11 + x13 =
20
) −0.965

x=3
2 1
20
( 25 − 2x11 + 3x12 =) 1.1515
These and further iterates are listed in the table below:
i x1i x2i x3i
0 0 0 0
1 0.85 –0.90 1.25
2 1.02 –0.965 1.1515
3 1.0134 –0.9954 1.0032
4 1.0009 –1.0018 0.9993
5 1.0000 –1.0002 0.9996
6 1.0000 –1.0000 1.0000

The values in 5th and 6th iterations being practically the same, we can stop. Hence the
solutions are:
x 1 = 1, x 2 = –1 and x 3 = 1

Numerical Analysis / M. Sc. - 13 - Written by Assoc. Prof.


Chap. 1 Dr. Zaidoon M. Shakoor
1.6.2 Gauss-Seidel Method
If the given system of equation is diagonally dominant then
x (i +1) =
1
a1
(
d1 − b1 y (i ) − c1 z (i ) )
y (i +1) =
1
b2
(
d 2 − a2 x (i +1) − c2 z (i ) )
z (i +1) =
1
c3
(
d 3 − a3 x (i +1) − b3 y (i +1) )

Example 1.8:
Use the Gauss-Seidel method to obtain the solution of the following equations:
6x 1 -2 x 2 + x 3 = 11 (1)
x 1 +2 x 2 -5x 3 = -1 (2)
-2x 1 +7 x 2 +2x 3 = 5 (3)
Solution
Step 1: Re-write the equations such that each equation has the unknown with largest
`coefficient on the left hand side:
2 x − x + 11
x1 = 2 3 from eq. (1)
6
2 x − 2 x3 + 5
x2 = 1 from eq. (3)
7
x + 2 x2 + 1
x3 = 1 from eq. (2)
5
Step 2: Assume the initial guesses x20 = x30 = 0 , then calculate x11 :
2( x20 ) − ( x30 ) + 11 2(0) − (0) + 11
x11 = = =1.833
6 6
Use the updated value x11 = 1.833 and x30 = 0 to calculate x12
2( x11 ) − 2( x30 ) + 5 2(1.833) − 2(0) + 5
x12 = = =1.238
7 7
Similarly, use x11 = 1.833 and x12 = 1.238 to calculate x31
( x11 ) + 2( x12 ) + 1 (1.833) + 2(1.238) + 1
x31 = = =1.062
5 5

Step 3: Repeat the same procedure for the 2nd iteration


2( x12 ) − ( x31 ) + 11 2(1.238) − (1.062) + 11
x12 = = = 2.069
6 6
2( x12 ) − 2( x31 ) + 5 2(2.069) − 2(1.062) + 5
x22 = = =1.002
7 7

Numerical Analysis / M. Sc. - 14 - Written by Assoc. Prof.


Chap. 1 Dr. Zaidoon M. Shakoor
( x12 ) + 2( x22 ) + 1 (2.069) + 2(1.002) + 1
x32 = = =1.015
5 5
and so on for the next iterations so that the next values are calculated using the
current values:
2( xi2 ) − ( x3i ) + 11
x1i +1 =
6
2( x1i +1 ) − 2( x3i ) + 5
xi2+1 =
7
( x1i +1 ) + 2( xi2+1 ) + 1
x3i +1 =
5
and continue the above iterative procedure until [(x k )i+1 - (x k )i]/ (x k )i+1 < Є for i=1,2 and
3.
The procedure yields the exact solution after 5 iterations only:
Unknown
Iter. x1 x2 x3
1 1.833 1.238 1.062
2 2.069 1.002 1.015
3 1.998 0.995 0.998
4 1.999 1.000 1.000
5 2.000 1.000 1.000

Example 1.9:
For the below figure calculate the values of the unknown flow rates F 1 , F 2 and F 3 by
using Gauss-Seidel Method
. F1=? F2=?
99% Benzene 5% Benzene
1% Toluene 92% Toluene
3 % Xylene
Tower 1

Tower 2

F=1000 kg/hr

40% Benzene
40% Toluene
20% Xylene
F3=?
10% Toluene
90% Xylene
Component material balance gives these three equations of three variables
0.99 F1 + 0.05 F2 + 0 F3 = 400
0.01F1 + 0.92 F2 + 0.1F3 = 400
0 F1 + 0.03F2 + 0.9 F3 = 200

Numerical Analysis / M. Sc. - 15 - Written by Assoc. Prof.


Chap. 1 Dr. Zaidoon M. Shakoor
Re-arranging the above equations
F1 = (400 − 0.05F2 ) / 0.99
F2 = (400 − 0.01F1 − 0.1F3 ) / 0.92
F3 = (200 − 0.03F2 ) / 0.9
Starting with F1=F2=F3=1000/3
Iteration F1 F2 F3
333.3333 333.3333 333.3333
1.0000 387.2054 394.3420 209.0775
2.0000 384.1241 407.8815 208.6262
3.0000 383.4403 407.9380 208.6243
4.0000 383.4375 407.9383 208.6243
5.0000 383.4375 407.9383 208.6243

A Matlab program for solving the above equations using Gauss-Seidel method is
listed in Table 1.1
Table (1.1) Matlab code and results for solution example (1.9)
F1=333.33; F2=333.33; F3=333.33
for i=1:4
F1=(400-0.05*F2)/0.99;
Matlab F2=(400-0.01*F1-0.1*F3)/0.92;
Code F3=(200-0.03*F2)/0.9;
disp([ i, F1, F2, F3])
end
1.0000 387.2056 394.3423 209.0775
2.0000 384.1241 407.8815 208.6262
Results 3.0000 383.4403 407.9380 208.6243
4.0000 383.4375 407.9383 208.6243

1.7 Solving Linear Simultaneous Equations Using Gaussian Elimination Matlab


Command
There are a number of common situations in chemical engineering where systems
of linear equations appear. There are at least three ways in MATLAB to solve these
systems of equations. One of these methods is using matrix algebra commands (Also
called matrix inverse or Gaussian Elimination method).
Consider a set of n equations in which the unknowns are x 1 , x 2 ... x n .
The system of equations given above can be expressed in the matrix form as.
 a 11 a 12 a 13 : a 1n   x 1   b1 
a
 21 a 22 a 23 : :   x 2   b 2 
  
 : : : : : x 3  = b3 
     Or AX = b
 : : : : :  :   : 
a n1 a n2 a n3 : a nn   x n  b n 
Numerical Analysis / M. Sc. - 16 - Written by Assoc. Prof.
Chap. 1 Dr. Zaidoon M. Shakoor
Where:
 a 11 a 12 a 13 : a 1n 
 b1   x1 
a
 21 a 22 a 23 : a 2n  b  x 
 2  2
A= : : : : :  b = b3  X = x 3 
     
 : : : : : 
:   : 
a n1 a n2 a n3 : a nn  b n   x n 

To determine the variables contained in the column vector 'x', complete the
following steps.
(a) Create the coefficient matrix 'A'. Remember to include zeroes where an equation
does not contain a variable.
(b) Create the right-hand-side column vector 'b' containing the constant terms from
the equation. This must be a column vector, not a row.
(c) Calculate the values in the 'x' vector by left dividing 'b' by 'A', by typing x = A\b.
Note: this is different from x = b/A.

Example 1.10
Xylene, styrene, toluene and benzene are to be separated with the array of distillation
columns that is shown below. Write a program to calculate the amount of the streams
D, B, D1, B1, D2 and B2 also to calculate the composition of streams D and B.

Note: Solve the system of equations by using the matrix inverse method.
Solution
By making material balances on individual components on the overall

Numerical Analysis / M. Sc. - 17 - Written by Assoc. Prof.


Chap. 1 Dr. Zaidoon M. Shakoor
separation train yield the equation set
Xylene: 0.07D1+ 0.18B1+ 0.15D2+ 0.24B2= 0.15× 70
Styrene: 0.04D1+ 0.24B1+ 0.10D2+ 0.65B2= 0.25× 70
Toluene: 0.54D1+ 0.42B1+ 0.54D2+ 0.10B2= 0.40× 70
Benzene: 0.35D1+ 0.16B1+ 0.21D2+ 0.01B2= 0.20× 70
Overall material balances and individual component balances on column 2 can be
used to determine the molar flow rate and mole fractions from the equation of stream
D.
Molar Flow Rates: D = D1 + B1
Xylene: XDxD = 0.07D1 + 0.18B1
Styrene: XDsD = 0.04D1 + 0.24B1
Toluene: XDtD = 0.54D1 + 0.42B1
Benzene: XDbD = 0.35D1 + 0.16B1
Where:
XDx = mole fraction of Xylene.
XDs = mole fraction of Styrene.
XDt = mole fraction of Toluene.
XDb =mole fraction of Benzene.
Similarly, overall balances and individual component balances on column 3 can be
used to determine the molar flow rate and mole fractions of stream B from the
equation set.
Molar Flow Rates: B = D2 + B2
Xylene: XBxB = 0.15D2 + 0.24B2
Styrene: XBsB = 0.10D2 + 0.65B2
Toluene: XBtB = 0.54D2 + 0.10B2
Benzene: XBbB = 0.21D2 + 0.01B2
Where: F, D, B, D1, B1, D2 and B2 are the molar flow rates in mol/min.
A Matlab program for solving the above equations using Gaussian-Elimination
method is listed in Table 1.2
Table (1.2) Matlab code and results for solution example (1.10)
A=[0.07, 0.18, 0.15, 0.24; 0.04, 0.24, 0.10, 0.65; 0.54, 0.42, 0.54, 0.1;0.35,
0.16, 0.21, 0.01];
B=[0.15*70; 0.25*70; 0.4*70; 0.2*70];
X=A\B;
D1=X(1),B1=X(2),D2=X(3),B2=X(4)
Matlab D=D1+B1
Code B=D2+B2
XDx=(0.07*D1+0.18*B1)/D
XDs=(0.04*D1+0.24*B1)/D
XDt=(0.54*D1+0.42*B1)/D
XDb=(0.35*D1+0.16*B1)/D
Numerical Analysis / M. Sc. - 18 - Written by Assoc. Prof.
Chap. 1 Dr. Zaidoon M. Shakoor
XBx=(0.15*D2+0.24*B2)/B
XBs=(0.1*D2+0.65*B2)/B
XBt=(0.54*D2+0.1*B2)/B
XBb=(0.21*D2+0.01*B2)/B
D1 =
26.2500
B1 =
17.5000
D2 =
8.7500
B2 =
17.5000
D=
43.7500
B=
26.2500
XDx =
0.1140
Results XDs =
0.1200
XDt =
0.4920
XDb =
0.2740
XBx =
0.2100
XBs =
0.4667
XBt =
0.2467
XBb =
0.0767

Example 1.11
Balance the following chemical equation:
x 1 P 2 I 4 + x 2 P 4 + x 3 H2 O → x 4 PH 4 I + x 5 H 3 PO 4
R R R R R R R R R R R R R R R R R R R R R R R

USolution:
P balance: 2x 1 + 4x 2 =x 4 + x 5 R R R R R R R R

I balance: 4x 1 =x 4 + x 5 R R R R R R

H balance: 2x 3 =4x 4 +3x 5 R R R R R R

O balance: x 3 =4x 5 R R R R

Re-write these as homogeneous equations, each having zero on its right hand
side:

Numerical Analysis / M. Sc. - 19 - Written by Assoc. Prof.


Chap. 1 Dr. Zaidoon M. Shakoor
2x 1 + 4x 2 - x 4 - x 5 = 0
4x 1 - x 4 - x=0
2x 3 - 4x 4 - 3x 5 =0
x 3 - 4x 5 = 0
At this point, there are four equations in five unknowns. To complete the system,
we define an auxiliary equation by arbitrarily choosing a value for one of the
coefficients:
x1= 1
We can easily solve the above equations to balance this reaction using MATLAB
such in table 1.3

Table (1.3) Matlab code and results for solution example (1.11)
A = [2 4 0 -1 -1
4 0 0 -1 0
0 0 2 -4 -3
Matlab
0 0 1 0 -4
Code
1 0 0 0 0];
B= [0;0;0;0;1];
X = A\B
X=
1.0000
Results 1.3000
12.8000
4.0000
3.2000
This does not yield integral coefficients, but multiplying by 10 will do the trick:
The balanced equation will be:
10 P 2 I 4 + 13 P 4 + 128 H 2 O → 40 PH4 I + 32 H 3 PO 4

Example 1:12
Steady state mass balances on a flash tank.
Consider an isothermal flash tank:
Vi(y)
Fi(Z)
Isothermal
flash tank Li(x)

This unit takes a pressurized liquid, three-component feed stream and exposes it to
a low-pressure vessel maintained under isothermal conditions. The net result is that

Numerical Analysis / M. Sc. - 20 - Written by Assoc. Prof.


Chap. 1 Dr. Zaidoon M. Shakoor
some of the fluid is vaporized, while some fluid remains liquid. The compositions of
the liquid and vapor phase are determined by the combined analysis of mass balances
and Raoult’s Law for vapor-liquid equilibrium. The temperature in the flash tank is T
= 298 K and the pressure in the tank is P= 101 kPa. Raoult’s Law states that the
product of the liquid mole fraction of component i and the vapor pressure of
component i is equal to the partial pressure of component i in the vapor phase:
xi Pi vap = yi P
Use the following data for the temperature given above
PAVap =0.6 bar at 298 K
PBVap =1.0 bar at 298 K
PCVap =2.9 bar at 298 K
F=100 mol/hr V=44.738 mol/hr L=F-V mol/hr
ZA=0.4 y A =? x A =?
ZB=0.3 yB =? x B =?
ZC=0.3 yC =? x C =?
Using MATLAB solve for steady state values of the unknowns.

Solution:
Then you have six unknowns, the compositions of the liquid stream and the
composition of the vapor stream and you have eight available equations, which are.
A mole balance: Lx A +Vy A =Fz A
B mole balance: Lx B +VyB =Fz B (not used dependent)
C mole balance: Lx C +VyC =Fz C (not used dependent)
Liquid mole fraction constraint x A +x B +x C =1
Vapor mole fraction constraint y A +yB +y C =1
A equilibrium constraint x A PAVap -y A P=0
B equilibrium constraint x B PBVap -y B P=0
C equilibrium constraint x C PCVap -y C P=0
R R

Put equations in matrix form


Matrix of coefficients A (6 ×6)

eqn xAR xBR xCR yA


R yB
R yCR

1 L 0 0 V 0 0
2 1 1 1 0 0 0
3 0 0 0 1 1 1
4 PAVap 0 0 P 0 0
5 0 PBVap 0 0 -P 0
6 0 0 PCVap 0 0 -P

Numerical Analysis / M. Sc. - 21 - Written by Assoc. Prof.


Chap. 1 Dr. Zaidoon M. Shakoor
Vector of right hand side b(6×1)
Eqn b
1 Fz A R

2 1
3 1
4 0
5 0
6 0
A Matlab program for solving the above equations using Gaussian-Elimination
method is listed in Table 1.4

Table (1.4) Matlab code and results for solution example (1.12)
F = 100; V = 44.738; L = F - V; zA = 0.4; zB = 0.3; zC = 0.3; PvapA = 0.6;
PvapB = 1.0;PvapC = 2.0;P = 1.01325;
A = [L, 0,0,V,0,0;1,1,1,0,0,0;0,0,0,1,1,1;PvapA,0,0,-P,0,0;0,PvapB, 0,0,-P,0;
Matlab 0,0,PvapC,0,0,-P]
Code b =[F*zA; 1; 1; 0; 0; 0]
x = A\b;
xA = x(1),xB = x(2),xC = x(3),yA = x(4),yB = x(5),yC = x(6)
xA =
0.4893
xB =
0.3018
xC =
0.2090
Results yA =
0.2897
yB =
0.2978
yC =
0.4125

Example 1.13
Steady state mass balances on a single-stage liquid-liquid extractor.
Extract
E , (XEb, XEC, XEf) Solvent
S , (Xsb, XsC, Xsf)
Extractor

Feed Raffinate
F , (XFb, XFC, XFf) R , (XRb, XRC, XRf)

Numerical Analysis / M. Sc. - 22 - Written by Assoc. Prof.


Chap. 1 Dr. Zaidoon M. Shakoor
This unit removes uses a recycled furfural stream as the solvent to extract benzene
from a cyclohexane product stream. The data you are given is:
F=100 mol/hr S=150 mol/hr R=95 mol/hr E=155 mol/hr
X Fb =0.1
R R X Sb =0.0010
R R X Rb =? R R X Eb =? R R

X Fc =0.9
R R X Sc =0.0001
R R X Rc =? R R X Ec =? R R

X Ff =0.0
R R X Sf =0.9989
R R X Rf =? R R X Ef =? R R

The equilibrium constants are:


X Eb X Ec
Kb = = 20.0 and KC = = 0.05
X Rb X Rc
Using MATLAB solve for steady state values of the unknowns.
Solution:
You have six unknowns, the compositions of the raffinate and the compositions of
the extract stream. In addition, you have seven independent equations, which are
Benzene mole balance RX Rb +EX EB =FX Fb +SX Sb R R R R R R R

Cyclohexane mole balance RX Rc +EX Ec =FX Fc +SX S c R R R R R R R R

furfural mole balance RX Rf +EX Ef =FX Ff +SX Sf (not used R R R R R R R R

dependent)
raffinate mole fraction constraint X Rb +X Rc +X Rf =1 R R R R R R

Extract mole fraction constraint X Eb +X Ec +X Ef =1 R R R R R R

Benzene equilibrium constraint X Eb -X Rb K b =0 R R R R R R

Cyclohexane equilibrium constraint X Ec -X Rc K c =0 R R R R R R

Note: you have six variables and seven equations therefore; you have to choice any
six equations from these equations. In this case, we neglect mole balance equation.
Put the equations in matrix form.

Matrix of coefficients A (6 ×6)


Eqn X Rb R X Rc R X Ef R X Eb R X EcR X Ef
R

1 R 0 0 E 0 0
2 0 R 0 0 E 0
3 1 1 1 0 0 0
4 0 0 0 1 1 1
5 -Kb 0 0 1 0 0
6 0 -Kc 0 0 1 0

Vector of right hand side b (6×1)

Numerical Analysis / M. Sc. - 23 - Written by Assoc. Prof.


Chap. 1 Dr. Zaidoon M. Shakoor
Eqn. B
1 FX Fb +SX Sb
2 FX Fc +SX Sc
3 1
4 1
5 0
6 0
A Matlab program for solving the above equations using Gaussian-Elimination
method is listed in Table 1.5

Table (1.5) Matlab code and results for solution example (1.13)
F=100;S=150;R=95;E=155;
Kb=20;Kc=0.05;XFb=.1;XFc=.9;XFf=0;
XSb=.001;XSc=.0001;XSf=.9989;
Matlab A=[R,0,0,E,0,0;0,R,0,0,E,0;1,1,1,0,0,0;0,0,0,1,1,1;-Kb,0,0,1,0,0;0,-Kc,0,0,1,0];
Code B=[F*XFb+S*XSb;F*XFc+S*XSc;1;1;0;0];
X=A\B;
XRb=X(1),XRc=X(2),XRf=X(3),XEb=X(4),XEc=X(5),XEf=X(6)
XRb =
0.0032
XRc =
0.8761
XRf =
0.1208
Results XEb =
0.0635
XEc =
0.0438
XEf =
0.8927

Numerical Analysis / M. Sc. - 24 - Written by Assoc. Prof.


Chap. 1 Dr. Zaidoon M. Shakoor
Chapter 2: Solution of Non-Linear Equations

2.1 Solutions of Non-Linear Equations in One Variable (Root Finding)

2.1.1 Graphical Methods


A simple method for obtaining a root of the equation f(x) = 0 is to plot the function
and observe where it crosses the x axis. There is much available software that will
facilitate making a plot of a function. We will use Matlab exclusively for the course
notes; however you can use other software such as Excel or Matcad for your work.

Example 2.1
(1 − e−0.15x) =50 using the graphical method.
600
Solve
x
Solution
(1 − e−0.15x) − 50 can be plotted in Figure 2.1 using the
600
The function f(x) =
x
Matlab statements listed in table (2.1).

Table (2.1) Matlab code for solving example (2.1) using graphical method
x=4:0.1:20;
fx=600*(1-exp(-0.15*x))./x-50;
Matlab plot(x,fx,[0 20],[0 0])
Code xlabel('x');
ylabel('f(x)')
grid on; zoom on

20

15

10

0
f(x)

-5

-10

-15

-20

-25
0 2 4 6 8 10 12 14 16 18 20
x

Figure 2.1 The graphical method for roots finding.

Numerical Analysis / M. Sc. - 25 - Written by Assoc. Prof.


Chap. 2 Dr. Zaidoon M. Shakoor
The Matlab Zoom on statement allows the function to be zoomed in at the cursor
with left mouse click (right mouse click will zoom out). Each time you click the axes
limits will be changed by a factor of 2 (in or out). You can zoom in as many times as
necessary for the desired accuracy. Figure 2.2 shows the approximate root x to be
8.79

0.06

0.04

0.02

0
f(x)

-0.02

-0.04

-0.06

-0.08

8.75 8.76 8.77 8.78 8.79 8.8 8.81 8.82


x

Figure 2.2 The graphical method for roots finding with Matlab Zoom on.

The plot of a function between x 1 and x 2 is important for understanding its behavior
within this interval. More than one root can occur within the interval when f(x 1 ) and
f(x 2 ) are on opposite sides of the x axis. The roots can also occur within the interval
when f(x 1 ) and f(x 2 ) are on the same sides of the x axis. Since the functions that are
tangent to the x axis satisfy the requirement f(x) = 0 at this point, the tangent point is
called a multiple root.

2.1.2 The Bisection Method


The bisection method or interval halving can be used to determine the solution to f(x)
= 0 on an interval [x 1 = a, x 2 = b] if f(x) is real and continuous on the interval and
f(x 1 ) and f(x 2 ) have opposite signs. We assume for simplicity that the root in this
interval is unique. The location of the root is then calculated as lying at the midpoint
of the subinterval within which the functions have opposite signs. The process is
repeated to any specified accuracy.
The procedure can be summarized in the following steps

Numerical Analysis / M. Sc. - 26 - Written by Assoc. Prof.


Chap. 2 Dr. Zaidoon M. Shakoor
Let f(x 1 ) f(x 2 ) < 0 on an interval [x 1 = a, x 2 = b]
1
Step 1 Let x x = (x 1 + x 2 ); f 1 = f(x 1 ); f 2 = f(x 2 )
2
Step 2 Evaluate f x = f(x x )
If f x f 1 > 0 then
x1 = xx; f1 = fx
else
x2 = xx; f2 = fx
end
Step 3 If abs(x 2 − x 1 ) > an error tolerance, go back to Step 1

Figure 2.3 shows first three iterations x 3 , x 4 , and x 5 of the bisection method.

15

10

x5 x3 x2=b
f(x)

0
x1 = a x4

-5

-10

-15
4 6 8 10 12 14 16
x

Figure 2.3 The first three iterations x 3 , x 4 , and x 5 of the bisection method.

x 1 =6 ⇒ f(x 1 ) =9.3430 and x 2 =14 ⇒ f(x 2 ) = -12.3910


1 1
x3 = (x 1 + x 2 ) = (6 + 14) = 10 ⇒ f(x 3 ) = -3.3878
2 2
1 1
f(x 1 ) f(x 3 ) < 0 ⇒ x 4 = (x 1 + x 3 ) = (6 + 10) = 8 ⇒ f(x 4 ) =2.4104
2 2
1 1
f(x 3 ) f(x 4 ) < 0 ⇒ x 5 = (x 3 + x 4 ) = (10 + 8) = 9 ⇒ f(x 4 ) = -0.6160
2 2

Numerical Analysis / M. Sc. - 27 - Written by Assoc. Prof.


Chap. 2 Dr. Zaidoon M. Shakoor
1 1
Since f(x 1 ) and f(x 2 ) bracket the root and x 3 = (x 1 + x 2 ) = (a + b), the error after
2 2
1
the first iteration is less than or equal to (b − a).
2
A Matlab program for solving example 2.1 using bisection method is listed in Table
2.2 where the function f(x) is an input to the program.

Table (2.2) Matlab code and results for solving example (2.1) using bisection method
fun=inline('600*(1-exp(-0.15*x)) /x-50')
x1=6;
f1= fun (x1);
x2=14;
f2= fun (x2);
tol=1e-5;
for i=1:100
x3=(x1+x2)/2;
f3= fun(x3);
Matlab
if f1*f3<0
Code
x2=x3;
f2=f3;
else
x1=x3;
f1=f3;
end
if abs(x2-x1)<tol; break;end
end
x3
x3 =
Results
8.7892
The statement Inline is used to define the function at a given value of x.

Example 2.2
Use the bisection method to find the root of the equation x-cos(x) = 0 with a percent
relative error |ε t |≤ 1%. (The exact value is 0.7391)
Solution
f(x)= x-cos(x)

Numerical Analysis / M. Sc. - 28 - Written by Assoc. Prof.


Chap. 2 Dr. Zaidoon M. Shakoor
We have seen before that there is a single root lies in the interval [0,1]. Therefore, we
start with x l =0 and x 2 =1, then iterate using the same procedure followed in example
2.1 to get the following tabulated results:
Iter Xl X2 X3 f(X l ) f(X 2 ) f(X 3 ) |ε t |
1 0.0 1.0 0.5 -1.0000 0.4597 -0.3776 32.35
2 0.5 1.0 0.75 -0.3776 0.4597 0.0183 1.48
3 0.5 0.75 0.625 -0.3776 0.0183 -0.1860 15.44
4 0.625 0.75 0.6875 -0.1860 0.0183 -0.0853 6.98
5 0.6875 0.75 0.7188 -0.0853 0.0183 -0.0339 2.75
6 0.7188 0.75 0.7344 -0.0339 0.0183 -0.0079 0.64
Then x=0.7344

Example 2.3
The friction factor f depends on the Reynolds number Re for turbulent flow in smooth
pipe according to the following relationship.
1
= −0.40 + 3 ln(Re f )
f
Use the bisection method to compute f for Re = 25200 that lies between [0.001 ,0.1 ].

Solution
Re-write the above equation in the form
1
E ( f ) = −0.40 + 3 ln(25200 f ) −
f
Iter fl f2 f3 E(f l ) E(f 2 ) E(f 3 )
1 0.0010 0.1000 0.0505 -20.4514 11.9973 10.1179
2 0.0010 0.0505 0.0258 -20.4514 10.1179 7.7528
3 0.0010 0.0258 0.0134 -20.4514 7.7528 4.7705
4 0.0010 0.0134 0.0072 -20.4514 4.7705 1.0841
5 0.0010 0.0072 0.0041 -20.4514 1.0841 -3.2373
6 0.0041 0.0072 0.0056 -3.2373 1.0841 -0.6453
7 0.0056 0.0072 0.0064 -0.6453 1.0841 0.2946
8 0.0056 0.0064 0.0060 -0.6453 0.2946 -0.1536
Then according to above table f = 0.006

Numerical Analysis / M. Sc. - 29 - Written by Assoc. Prof.


Chap. 2 Dr. Zaidoon M. Shakoor
2.1.3 Secant Method (Linear Interpolation Method)
The bisection method is generally inefficient, it requires more function evaluations in
comparison with the secant method which is linear interpolation using the latest two
points. Figure 2.4 shows graphically the root x 3 obtained from the intersection of the
line AB with the x-axis.
f(x)
f(x2) B

x1 x3 x4
x5 x2

A
f(x1)

Figure 2.4 Graphical depiction of the secant method.

The intersection of the straight line with the x-axis can be obtained by using similar
triangles x 3 x 1 A and x 3 x 2 A or by using linear interpolation with the following
points.
x x1 x3 x2
f(x) f(x 1 ) 0 f(x 2 )
x3 − x 2 0 − f ( x2 ) x2 − x1
= ⇒ x 3 = x 2 − f(x 2 )
x2 − x1 f ( x 2 ) − f ( x1 ) f ( x2 ) − f ( x1 )
The next guess is then obtained from the straight line through two points [x 2 , f(x 2 )]
and [x 3 , f(x 3 )]. In general, the guessed valued is calculated from the two previous
points [x n-1 , f(x n-1 )] and [x n , f(x n )] as

xn − xn −1
x n+1 = x n − f(x n )
f ( xn ) − f ( xn −1 )

The secant method always uses the latest two points without the requirement that they
bracket the root as shown in Figure 2.4 for points [x 3 , f(x 3 )] and [x 4 , f(x 4 )]. As a
consequence, the secant method can sometime diverge. A Matlab program for

Numerical Analysis / M. Sc. - 30 - Written by Assoc. Prof.


Chap. 2 Dr. Zaidoon M. Shakoor
solving example 2.1 using secant method is listed in Table 2.3
Table (2.3) Matlab code and results for solving example (2.1) using secant method
fun=inline('600*(1-exp(-0.15*x))/x-50')
tol=1e-5;
x(1)=1;
f(1)= fun(x(1));
x(2)=14;
f(2)= fun(x(2));
Matlab for i=2:20
Code x(i+1)=x(i)-f(i)*(x(i)-x(i-1))/(f(i)-f(i-1))
f(i+1)= fun(x(i+1));
if abs(x(i+1)-x(i))<tol;
break;
end
end
x
x=
1.0000
14.0000
10.4956
8.3724
Results
8.8209
8.7897
8.7892
8.7892
The last value of x vector is the solution of the equation

Example 2.4
Use secant method to estimate the root of f(x)=e-x-x. with the two initial guesses x 0 =
0. and x 1 = 1.
Solution:
Iter x f(x)
Starting 0 1.0000
Values 1.0 -0.6321
1 0.6127 -0.0708
2 0.563838 0.00518
3 0.56717 -0.0000418

Numerical Analysis / M. Sc. - 31 - Written by Assoc. Prof.


Chap. 2 Dr. Zaidoon M. Shakoor
Example 2.5
Repeat Examples 2.4 using the secant method with the two initial guesses x 0 = 2. and
x 1 = 3.
Solution:
Iter x f(x)
Starting 2 -1.8647
Values 3 -2.9502
1 0.2823 0.4718
2 0.6570 -0.1385
3 0.5719 -0.0075
4 0.5671
This method converges with the required accuracy after 5 iterations.

Example 2.6
Use secant method with initial guesses T = 300 and T = 350 to calculate the bubble
point of binary system (VCM 18 mol%, Water 82 mol%). The vapor pressure for this
components is calculated by:
VCM Po vcm =exp(14.9601-1803.84/(T-43.15))
Water Po w =exp(18.3036-3816.44/(T-46.13))
Where: K i = P o i /P t
P t =760
y i =K i ×x i
At Bubble point ∑y i =∑K i ×x i =1
Solution
1803.84 3816.44
(14.9601- ) 0.18 (18.3036 - T - 46.13 ) 0.82
f(T)= K i ×x i -1 = e T - 43.15
× +e × -1
760 760
Iter T f(T)
Initial 300.0000 -0.3086
Values 350.0000 1.4192
1 308.9299 -0.1132
2 311.9636 -0.0378
3 313.4866 0.0018
4 313.4157 -0.000028
Then at bubble point T=313.457 K

Numerical Analysis / M. Sc. - 32 - Written by Assoc. Prof.


Chap. 2 Dr. Zaidoon M. Shakoor
2.1.4 The Newton-Raphson Method
The Newton-Raphson method and its modification is probably the most widely used
of all root-finding methods. Starting with an initial guess x 1 at the root, the next guess
x 2 is the intersection of the tangent from the point [x 1 , f(x 1 )] to the x-axis. The next
guess x 3 is the intersection of the tangent from the point [x 2 , f(x 2 )] to the x-axis as
shown in Figure 2.5. The process can be repeated until the desired tolerance is
attained.

f(x)
f(x1) B

x3 x2 x1

Figure 2.5 Graphical depiction of the Newton-Raphson method.

The derivative or slope f(x n ) can be approximated numerically as


f ( xn + ∆x ) − f ( xn )
f’(x n ) =
∆x

The Newton-Raphson method can be derived from the definition of a slope


f ( x1 ) − 0 f ( x1 )
f’(x 1 ) = ⇒ x2 = x1 −
x1 − x2 f ' ( x1 )

In general, from the point [x n , f(x n )], the next guess is calculated as
f ( xn )
x n+1 = x n −
f ' ( xn )

A Matlab program for solving example 2.1 using Newton- Raphson method is listed
in Table 2.4 .

Numerical Analysis / M. Sc. - 33 - Written by Assoc. Prof.


Chap. 2 Dr. Zaidoon M. Shakoor
Table (2.4) Matlab code and results for solving example (2.1) using Newton-
Raphson method
f=inline('600*(1-exp(-0.15*x))/x-50');
df=inline('(90.0*exp(-0.15*x))/x + (600*exp(-0.15*x) - 600)/x^2');
tol=1e-5;
x(1)=1;
for i=2:20
Matlab
x(i+1)=x(i)-f(x(i))/df(x(i));
Code
if abs(x(i+1)-x(i))<tol;
break;
end
end
x'
x=
1.0000
0.0031
5.9278
Results 8.4473
8.7840
8.7892
8.7892
The last value of x vector is the solution of the equation

Example 2.7
Use Newton-Raphson method to estimate the root of f(x)=e-x-x. Show all details of
the iterations. Hint: the root is located between 0 and 1.
Solution:
Iter Xi f(X i ) f’(X i ) X i+1 |ε a (%)|
1 0.0 1.0 -2.0 0.5 100.00
2 0.5 0.1065 -1.6065 0.5663 11.71
3 0.5663 0.0013 -1.5676 0.5671 0.15
4 0.5671 0.0000 -1.5676 0.5671 0.00

Example 2.8
Repeat Example 2.7 starting with x o = 5.
Solution:
Iter Xi f(X i ) f’(X i ) X i+1 |ε a (%)|
1 5.0 -4.9933 -1.0067 0.04016 12351
2 0.04016 0.92048 -1.9606 0.5096 92.12
3 0.5096 0.0911 -1.6007 0.5665 10.04
Numerical Analysis / M. Sc. - 34 - Written by Assoc. Prof.
Chap. 2 Dr. Zaidoon M. Shakoor
4 0.5665 0.0010 -1.5675 0.5671 0.000

Example 2.9
Apply Newton-Raphson method to solve Redlich-Kwong equation which used to
estimate the molar volume of saturated vapor of methyl chloride at 333.15 K and
13.76 bar
RT a
P= − 0.5
V − b T V (V + b)
If you know that:-
A=1.5651×108 cm6 bar mol-2 K1/2
b=44.891 cm3 mol-1
R=83.14 cm 3.bar.K-1.mol-1

Solution
RT a
f (V ) = − 0.5 −P
V − b T V (V + b)
83.14 × 333.15 1.5651 × 108
f (V ) = − − 13.76
V − 44.891 333.150.5V (V + 44.891)
27698 8574764.131
f (V ) = − − 13.76
V − 44.891 V (V + 44.891)
- 27698 8574764.131 8574764.131
f ′(V) = + + 2
(V − 44.891) 2
V(V + 44.891) 2
V (V + 44.891)
It’s better to start with ideal molar volume as initial value of V
RT 83.14 × 333.15
V= = = 2012.943
P 13.76
Iter Vi f(V i ) f’(V i ) V i+1
1 2012.943 -1.75623 -0.00512 1669.717
2 1669.717 0.291628 -0.00695 1711.673
3 1711.673 0.005731 -0.00668 1712.531
4 1712.531 0.00000231 -0.00667 1712.531
The molar volume equal to 1712.531 cm3 mol-1

Numerical Analysis / M. Sc. - 35 - Written by Assoc. Prof.


Chap. 2 Dr. Zaidoon M. Shakoor
2.2 Solving System of Non-Linear Systems of Equations
A nonlinear system of equations has at least one equation which is not first degree.

Examples: x2 + y2 = 25 y = 3x2 – 4x + 2 xy = 9
2x + 3y = 7 x2 + y = 8 3x2 – y2 = 12
The solutions of a nonlinear system are the points of intersection of the graphs of the
equations. Some systems have one point of intersection; some have more than one
point of intersection; and some have no points of intersection.

2.2.1 Analytical Methods for Solving Systems of Equations


Solutions of nonlinear systems of equations can be found using the substitution or the
elimination method. The substitution method is preferable for a system with one
linear equation. The elimination method is preferable in most, but not all, cases when
both equations are nonlinear.

2.2.1.1. The Substitution Method


Solve one of the equations for a first degree variable. Substitute the resulting
expression in for that variable in the other equation. Solve for the remaining variable.
Back substitute to find the value(s) of the first variable. Write your solutions as
ordered pairs.

Example 2.10
Solve x2 + 2x = y + 6
x + y = –2
Solution
x + y = –2 → y = –2 – x → x2 + 2x = –2 – x + 6 →
x2 + 3x – 4 = 0 → (x + 4) (x – 1) = 0 → x = –4 & x = 1
y = –2 – (–4) = 2 & y = –2 – 1 = –3 →

Solution: (–4, 2) (1, –3)

Numerical Analysis / M. Sc. - 36 - Written by Assoc. Prof.


Chap. 2 Dr. Zaidoon M. Shakoor
To check graphically, enter y 1 = x2 + 2x – 6 & y2 = –2 – x
Find the points of intersection.

2.2.1.2. The Elimination Method

Line the equations up vertically so like terms are underneath each other. If needed,
multiply each equation by a number so that when the equations are added together one
of the variables is eliminated. Solve for the remaining variable. Back substitute to find
the value(s) of the eliminated variable. Write your solutions as ordered pairs.

Example 2.11
Solve 3x2 + 5y2 = 17
2x2 – 3y2 = 5
Solution:
3x2 + 5y2 = 17 3R 1 9x2 + 15y2 = 51

2x2 – 3y2 = 5 5R 2 → 10x2 – 15y2 = 25


19x2 = 76
x2 = 4 → x = –2 & x = 2
2 2
3(–2) + 5y = 17 → 12 + 5y2 = 17 →

5y2 = 5 → y2 = 1 → y = –1 & y=1


3(2)2 + 5y2 = 17 → 12 + 5y2 = 17 →

5y2 = 5 → y2 = 1 → y = –1 & y=1


Solutions: (–2, –1) (–2, 1) (2, –1) (2, 1)

2.2.2 Numerical Methods for Solving Systems of Equations


2.2.2.1 Fixed point iteration for systems of non-linear equations
Using an initial guess, solve for each variable in the system and use fixed-point
iteration to estimate the solution.
One of the most important drawbacks of the fixed iteration method is that the
convergence of the method is dependent on how the equations are formulated. May
diverge quickly, in that case try solving for the variables in a different way.

It can be shown that sufficient convergence criteria for two equations are:
∂f1 ∂f
+ 1 <1
∂x1 ∂x2

Numerical Analysis / M. Sc. - 37 - Written by Assoc. Prof.


Chap. 2 Dr. Zaidoon M. Shakoor
And
∂f 2 ∂f 2
+ <1
∂x1 ∂x2
This represents a very restrictive criteria and that’s why fixed point iteration method
is not used to solve systems of non-linear equations.

Example 2.12
Solve using 5 iteration of successive substitution where x = y = 1.5 initially:
 x 2 + y 2 = 5

 y − x 2 = −1

Solution

Using x = 5 − y 2 and y = x2 – 1 it is easy to show the following iterations:

Iteration xn yn
0 1.5 1.5
1 1.658 1.75
2 1.392 0.9375
3 2.030 3.121
4 Non-real Non-real
5
So it is apparent that successive substitution this will not work using these formulas.

Using y = 5 − x 2 and x = 1 + y we can just as easily show the following iterations:

Iteration xn yn
0 1.5 1.5
1 1.5811 1.5811
2 1.607 1.555
3 1.599 1.564
4 1.601 1.561
5 1.600 1.562
 1 + 17 − 1 + 17 
Which is rapidly converging on the true solution of  , 
 2 2 
 
As with the Jacobi iterative process, convergence is assured only for a system of
Numerical Analysis / M. Sc. - 38 - Written by Assoc. Prof.
Chap. 2 Dr. Zaidoon M. Shakoor
diagonally dominant linear equations. For systems that are neither linear nor
diagonally dominant, convergence is a function of the equations themselves as well
as the values of x’s chosen to start the iterations.

Example 2.13
As an example of applying the Jacobi method to a system of non-linear equations,
consider the following system:
4 x1 − x 2 + x 3 =7
4x1 −8( x 2 ) + x 3 =−21
2

−2x1 + x 2 +5( x 3 ) =15


2

Solving the equations for each of the unknowns (x’s), we have the following:
 7+ x 2 − x 3
2

x1 = 
 4 
−21−4x1 − x 3
x2 =
−8
15+2x1 − x 2
x3 =
5
Using these relationships in a Jacobi algorithm with starting values of x 1 = x 2 = x 3 =
0, we can show the convergence of the algorithm over ten iterations in the following
table:
Itr x1 x2 x3
0 0 0 0
1 3.0625 1.620185 1.732051
2 2.9654 2.091114 1.975086
3 3.164866 2.086764 1.941118
4 3.191267 2.109519 1.961783
5 3.193133 2.113257 1.963314
6 3.195105 2.113523 1.963314
7 3.195343 2.113757 1.963501
8 3.195385 2.113790 1.963514
9 3.195403 2.113796 1.963516
10 3.195406 2.113798 1.963518

Numerical Analysis / M. Sc. - 39 - Written by Assoc. Prof.


Chap. 2 Dr. Zaidoon M. Shakoor
2.2.2.2 Newton-Raphson for solving systems of non-linear equations
The Newton-Raphson formula is the following:
f ( xi )
xi +1 = xi −
f ' ( xi )
This formula can be obtained using Taylor series expansion. We can do the same
approach for a system of equations, but considering a Taylor series that account for
the presence of both variables:
∂f1(i ) ∂f1(i )
f1(i +1) = f1(i ) + (x1(i +1) − x1(i ) ) + (x2(i +1) − x2(i ) ) + ...
∂x1 ∂x2
and
∂f 2(i ) ∂f 2(i )
f 2(i +1) = f 2(i ) + (x1(i +1) − x1(i ) ) + (x2(i +1) − x2(i ) ) + ...
∂x1 ∂x2

For the root estimate f1(i+1) and f 2(i+1) must be equal zero.

Therefore:
∂f1(i ) ∂f ∂f ∂f
x1(i+1) + 1(i ) x2 (i+1) = − f1(i ) + x1(i ) 1(i ) + x2(i ) 1(i )
∂x1 ∂x2 ∂x1 ∂x2
and
∂f 2(i ) ∂f ∂f ∂f
x1(i+1) + 2(i ) x2(i+1) = − f 2(i ) + x1(i ) 2(i ) + x2(i ) 2(i )
∂x1 ∂x2 ∂x1 ∂x2

Finally:
∂f 2 (i ) ∂f1(i )
f1(i ) − f 2 (i )
∂x 2 ∂x 2
x 1(i +1) = x 1(i ) −
∂f1(i ) ∂f 2 (i ) ∂f1(i ) ∂f 2 (i )

∂x 1 ∂x 2 ∂x 2 ∂x 1
∂f1(i ) ∂f 2 (i )
f 2 (i ) − f1(i )
∂x 1 ∂x 1
x 2 (i +1) = x 2 (i ) −
∂f1(i ) ∂f 2 (i ) ∂f1(i ) ∂f 2 (i )

∂x 1 ∂x 2 ∂x 2 ∂x 1

Which is an iterative method to solve the system of nonlinear equations.

Note also that the Newton-Raphson method can be generalized to solve N


simultaneous equations.

Numerical Analysis / M. Sc. - 40 - Written by Assoc. Prof.


Chap. 2 Dr. Zaidoon M. Shakoor
Example 2.14
Solve the following system using Newton-Raphson method:
x 2 + y 2 − 8x − 4 y + 11 = 0
x 2 + y 2 − 20 x + 75 = 0

By tacking a starting point as (x=2; y=4) and ε = 10−5 .


Solution
∂f 2 (i ) ∂f1(i )
f 1(i ) − f 2 (i )
∂y ∂y
x ( i +1 ) = x ( i ) −
∂f1(i ) ∂f 2 (i ) ∂f1(i ) ∂f 2 (i )

∂x ∂y ∂y ∂x
∂f1(i ) ∂f 2 (i )
f 2 (i ) − f1(i )
y ( i +1 ) = y ( i ) − ∂x ∂x
∂f1(i ) ∂f 2 (i ) ∂f1(i ) ∂f 2 (i )

∂x ∂y ∂y ∂x

Let f1 = x 2 + y 2 − 8x − 4 y + 11 and f 2 = x 2 + y 2 − 20x + 75


∂f1,i ∂f1,i ∂f 2,i ∂f 2,i
Thus: = 2 x − 8, = 2 y − 4, = 2 x − 20, and = 2y
∂x ∂y ∂x ∂y
Hence when x =2 and y = 4 we find that:
∂f1, 0 ∂f1, 0 ∂f 2, 0
= 2(2) − 8 = −4, = 2(4) − 4 = 4, = 2(2) − 20 = −16, and
∂x ∂y ∂x
∂f 2, 0
= 2(4) = 8
∂y

Also f1, 0 = 22 + 42 − 8(2) − 4(4) + 11 = −1 and f 2, 0 = 22 + 42 − 20(2) + 75 = 55


∂f1, 0 ∂f 2, 0 ∂f1, 0 ∂f 2, 0
− = (−4)(8) − (4)(−16) = 32
∂x ∂y ∂y ∂x
So for the first iteration we see that:
(−1)(8) − (55)(4)
x1 = 2 − ≈ 9.1250
32
(55)(−4) − (−1)(−16)
y1 = 4 − ≈ 11.3750
32

x 0 − x1 2 − 9.125
ε= = = 3.5625
x0 2

Now we find that iteration 2 produces:

Numerical Analysis / M. Sc. - 41 - Written by Assoc. Prof.


Chap. 2 Dr. Zaidoon M. Shakoor
∂f1,1 ∂f1,1
= 2(9.1250) − 8 = 10.25, = 2(11.375) − 4 = 18.75,
∂x ∂y
∂f 2,1 ∂f 2,1
= 2(9.1250) − 20 = -1.75, and = 2(11.375) = 22.75
∂x ∂y

Also f1,1 = 9.1252 + 11.375 2 − 8(9.125) − 4(11.375) + 11 = 105.1563 and

f 2,1 = 9.1252 + 11.375 2 − 20(9.125) + 75 = 105.1563

∂f1,1 ∂f 2,1 ∂f1,1 ∂f 2,1


− = (10.25)(22.75) − (18.75)(−1.75) = 266
∂x ∂y ∂y ∂x
So for the second iteration we see that:
(105.1563)(22.75) − (105.1563)(18.75)
x 2 = 9.125 − ≈ 7.543703
266
(105.1563)(10.25) − (105.1563)(-1.75)
y 2 = 11.375 − ≈ 6.631109
266

x 0 − x1 9.125 − 7.543703
ε = = = 0.173293
x0 9.125

∂f1 ∂f1 ∂f 2 ∂f 2
Itr f1 f2 x y εs
∂x ∂y ∂x ∂y
2 4
1 -1 55 -4 4 -16 8 9.125 11.375 3.5625
2 105.1563 105.1563 10.25 18.75 -1.75 22.75 7.543703 6.631109 0.173293
3 25.005 25.005 7.087406 9.262218 -4.91259 13.26222 6.826694 4.480083 0.095047
4 5.141014 5.141014 5.653389 4.960166 -6.34661 8.960166 6.576327 3.728981 0.036675
5 0.626838 0.626838 5.152654 3.457963 -6.84735 7.457963 6.535955 3.607865 0.006139
6 0.016299 0.016299 5.07191 3.215731 -6.92809 7.215731 6.534848 3.604543 0.000169
7 1.23E-05 1.23E-05 5.069696 3.209087 -6.9303 7.209087 6.534847 3.604541 1.28×10-7

Numerical Analysis / M. Sc. - 42 - Written by Assoc. Prof.


Chap. 2 Dr. Zaidoon M. Shakoor
Chapter 3: Solution of Ordinary Differential Equations

3.1 Solution of First-Order Ordinary Differential Equations


An equation that consists of derivatives is called a differential equation. Differential
equations have applications in all areas of science and engineering. Mathematical
formulation of most of the physical and engineering problems lead to differential
equations. So, it is important for engineers and scientists to know how to set up
differential equations and solve them. Differential equations are of two types
1) Ordinary differential equation (ODE).
2) Partial differential equations (PDE).
An ordinary differential equation is that in which all the derivatives are with respect
to a single independent variable. Examples of ordinary differential equation include:
dy
1) + y = sin(x) , y (0) = 1 ,
dx
d2y dy dy
2) 2
+2 + y=0 , (0) = 2 , y (0) = 4
dx dx dx
d3y d2y dy d2y dy
3) 3
+ 3 2
+ 5 + y = sin x , 2
(0) = 12 , (0) = 2 , y (0) = 4
dx dx dx dx dx
First order ordinary differential equations are of the form:
dx
= f=( x, t ) with x(0) x0
dt
On the left hand side is the derivative of the dependent variable x with respect to
the independent variable t. On the right hand side, there is a function that may depend
on both x and t.
Many differential equations cannot be solved exactly. Numerical methods have
been developed to approximate solutions. Numerical analysis is a field in mathematics
that is concerned with developing approximate numerical methods and assessing their
accuracy, for instance for solving differential equations. We will discuss the most basic
method such Taylor, Euler and Runge-Kutta methods.

3.1.1 Taylor Series Method


Function y (x) can be expanded over a small interval x using the Taylor series from a
start or reference point x
1 2 1 1
y( x + h ) = y( x ) + hy′( x ) + h y′′( x ) + h 3 y′′′( x ) + h 4 y ( 4 ) ( x ) +  (1)
2! 3! 4!
Numerical Analysis / M. Sc. - 43 - Written by Assoc. Prof.
Chap. 3 Dr. Zaidoon M. Shakoor
Where hi = xi +1 − xi = h, a constant.
Example 3.1
Solve the following ordinary differential equation (ODE) using Taylor’s method of
order 2 with h= 0.2
dy
= y − x2 + 1 , for 0 ≤ x ≤ 2 , with y(0) = 0.5
dx
Solution
d2y dy
2
= − 2x = y − x2 + 1 − 2x
dx dx
1
y n+1 = y n + (y n − x n 2 + 1)h + ( yn − x n 2 − 2x n + 1)h2
2
i x y
1 0 0.5000
2 0.2000 0.8300
3 0.4000 1.2158
4 0.6000 1.6521
5 0.8000 2.1323
6 1.0000 2.6486
7 1.2000 3.1913
8 1.4000 3.7486
9 1.6000 4.3061
10 1.8000 4.8463
11 2.0000 5.3477

3.1.2. Euler’s Method


Euler’s method is the simplest and least useful of these three methods. If we are
dy
solving a first-order differential equation of the form = f ( t , y ) with the initial
dt
condition y(0)=A, Euler’s method begins by approximating the first derivative as
dy y (t + ∆t ) − y (t )

dt ∆t

Setting this equal to f(t,y) and solving for y (t + ∆t ) yields the following algorithm for

advancing the numerical solution of an ordinary differential equation:

Numerical Analysis / M. Sc. - 44 - Written by Assoc. Prof.


Chap. 3 Dr. Zaidoon M. Shakoor
dy
= f ( x, y ) y( 0 ) = y0
dx
yn +1 = yn + h × f ( x, y )

Using Euler’s method we have the following consideration:


x1 = x0 + h y1 = y 0 + h ⋅ f ( x0 , y 0 )
x 2 = x1 + h y 2 = y1 + h ⋅ f ( x1 , y1 )
x3 = x 2 + h y3 = y 2 + h ⋅ f ( x2 , y 2 )
 
 
 

Example 3.2:
Apply Euler’s method to approximate the solution of the initial value problem
dy
= 2 y with y (0) = 5 (2)
dx
Solution:
We know that the analytical solution of equation (2) is , y = 5 exp(2 x) . We numerically
solve equation (2) using Euler’s method with h=0.1 in the time interval [0, 0.5], and
then check how well this method performs. We have f ( y ) = 2 y . Then
x0 = 0
x1 = x0 + h = 0 + 0.1 = 0.1
x 2 = x1 + h = 0.1 + 0.1 = 0.2
x3 = x 2 + h = 0.2 + 0.1 = 0.3
x 4 = x3 + h = 0.3 + 0.1 = 0.4
x5 = x 4 + h = 0.4 + 0.1 = 0.5
And
y0 = 5
y1 = y 0 + hf ( y 0 ) = 5 + (
0.1)( 2)(5) = 6

h f ( y0 )

y 2 = y1 + hf ( y1 ) = 6 + (0.1)(2)(6) = 7.2
y 3 = y 2 + hf ( y 2 ) = 7.2 + (0.1)(2)(7.2) = 8.64
y 4 = y 3 + hf ( y 3 ) = 8.64 + (0.1)(2)(8.64) = 10.368
y 5 = y 4 + hf ( y 4 ) = 10.368 + (0.1)(2)(10.368) = 12.4416

We summarize this in the following table. If h=0.1, then

Numerical Analysis / M. Sc. - 45 - Written by Assoc. Prof.


Chap. 3 Dr. Zaidoon M. Shakoor
x y Exact Difference
0 5 5 0
0.1 6 6.107014 0.107014
0.2 7.2 7.459123 0.259123
0.3 8.64 9.110594 0.470594
0.4 10.368 11.1277 0.759705
0.5 12.4416 13.59141 1.149809
The third column contains the exact values, y = 5 exp(2 x) . The last column contains
the absolute error after each step, computed as |y-yExact |. We see that when h=0.1, the
numerical approximation is not very good after five steps. If we repeat the same
approximation with a smaller value for h, say h=0.01, the following table results for
the first five steps:
x y Exact Difference
0 5 5 0
0.01 5.1 5.101007 0.001007
0.02 5.202 5.204054 0.002054
0.03 5.30604 5.309183 0.003143
0.04 5.412161 5.416435 0.004275
0.05 5.520404 5.525855 0.005451

Doing five steps only gets us to x=0.05. We can do more steps until we reach x=0.5.
We find that the final point will be:
x y Exact Difference
0.5 13.45794 13.59141 0.133469

Choosing a smaller value for h resulted in a better approximation at x=0.5 but also
required more steps. One source of error in the approximation comes from the
approximation itself.

Example 3.3
Consider the following initial value problem:
dx 2
=t +t with the initial condition: x (0) = 0.5 from t=0 to t=2
dt
Solution:

Numerical Analysis / M. Sc. - 46 - Written by Assoc. Prof.


Chap. 3 Dr. Zaidoon M. Shakoor
dt=0.2; t(1)=0; x(1)= 0.5;
for n=1:10
t(n+1)=t(n)+dt
x(n+1)=x(n)+dt*(t(n)^2+t(n))
end
plot (t,x)
xlabel('Time (t)')
ylabel('x(t)')

The result is in Figure (3.1)


5

4.5

3.5

3
x(t)

2.5

1.5

0.5
0 0.2 0.4 0.6 0.8 1 1.2 1.4 1.6 1.8 2
Time (t)

Figure 3.1: Euler method used to solve exercise1


This ODE can be analytically integrated to get the true solution:
t3 t2
x( t ) = + + 0.5
3 2
Applying the explicit Euler's scheme with a step size h = 0.2 we get:
x(0.2) = x(0) + 0.2f (0)
That is
x(0.2) = 0.5 + 0 = 0.5

The true solution from is x(0.2) = 0.52267

The relative error (e r ) expressed in percent is


True - approximation
er = × 100
True
0.5 - 0.52267
er = × 100 = 4.5%
0.5
The calculations results are plotted in Figure 3.2 showing the true and the approximate
value for h=0.2. Although the general trend of the true and the approximate values is
Numerical Analysis / M. Sc. - 47 - Written by Assoc. Prof.
Chap. 3 Dr. Zaidoon M. Shakoor
the same, the error is large. One way to reduce this error is by choosing a smaller step
size. Figure 3.2 shows the solution when the step size is halved i.e. h=0.1. Since the
Euler's method is first order, the global error is halved O (h /2) while the local error is
quartered O (h2/4). To get acceptable levels of errors the step size has to be further
reduced to very low values. This will however considerably increase the computational
time since it will take a larger number of iterations for each step. Nevertheless the
Euler's method because of its simplicity and easiness for implementation is still
attractive for many engineering problems.

5.5

4.5

3.5
x(t)

2.5

1.5 Euler (h = 0.2)


Euler (h = 0.1)
1 True

0.5
0 0.5 1 1.5 2 2.5
Time (t)

Figure 3.2: Euler method used to solve exercise 1

3.1.3 Fourth order Runge-Kutta Method


dy
To find numerical solution to the initial value problem = f ( x, y ), y (0) = y 0 using
dx
Runge-Kutta method we have the following consideration:

yi +1 = yi +
1
(k1 + 2k2 + 2k3 + k4 )h
6

k1 = f (xi , yi )

 1 1 
k2 = f  xi + h, yi + k1h 
 2 2 
 1 1 
k3 = f  xi + h, yi + k2 h 
 2 2 
Numerical Analysis / M. Sc. - 48 - Written by Assoc. Prof.
Chap. 3 Dr. Zaidoon M. Shakoor
k4 = f (xi + h, yi + k3h )

This method gives more accurate result compared to Euler’s method

Example 3.4:
Solve the following ordinary differential equation (ODE) using fourth order
Runge-Kutta method to calculate y(x=0.2)
dy
= x + y ; y(0) = 1 , h = 0.1
dx
Solution:
k 1 = 0 + 1= 1

k 2 = (0+0.05)+(1+1×0.05)=1.10

k 3 = (0+0.05) +(1+1.1×0.05)= 1.1050

k 4 = (0+0.1)+(1+1.1050×0.1) = 1.2105
h
y1 = y o + ( k1 + 2k 2 + 2k 3 + k 4 )
6
0.1
y(0.1) = 1 + × (1 + 2 × 1.1 + 2 × 1.105 + 1.2105) = 1.11034
6

k 1 = 0.1 + 1.11034 = 1.21034

k 2 = (0.1+0.05)+(1.11034+1.21034×0.05) = 1.3209

k 3 = (0.1+0.05) +(1.11034+1.3209×0.05)= 1.3264

k 4 = (0.1+0.1)+( 1.11034+1.3264×0.1) = 1.4430


0.1
y(0.2) = 1.11034 + × (1.21034 + 2 × 1.3209 + 2 × 1.3264 + 1.4430) = 1.2428
6
at x=0.2 y=1.2428

Example 3.5:
A ball at 1200 K is allowed to cool down in air at an ambient temperature of 300 K.
Assuming heat is lost only due to radiation, the differential equation for the
temperature of the ball is given by

Numerical Analysis / M. Sc. - 49 - Written by Assoc. Prof.


Chap. 3 Dr. Zaidoon M. Shakoor
= −2.2067 × 10 −12 (T 4 − 81 × 108 ) , T(0 ) = 1200 K
dT
dt
where T is in K and t in seconds. Find the temperature at t = 480 seconds using
Runge-Kutta 4th order method. Assume a step size of h = 240 seconds.

Solution
= −2.2067 × 10 −12 (T 4 − 81 × 108 )
dT
dt

f (t , T ) = −2.2067 × 10 −12 (T 4 − 81 × 108 )

Ti+1 = Ti +
h
(k1 + 2k 2 + 2k 3 + k 4 )
6

For i = 0 , t 0 = 0 , T0 = 1200 K

k1 = f (t 0 , T0 ) = f (0,1200 ) = −2.2067 × 10 −12 (1200 4 − 81 × 108 ) = −4.5579

   
k 2 = f  t 0 + h , T0 + k1h  = f  0 + (240 ),1200 + (− 4.5579 ) × 240 
1 1 1 1
 2 2   2 2 
(
= f (120,653.05) = −2.2067 × 10 −12 653.054 − 81 × 108 )
= −0.38347
   
k 3 = f  t 0 + h , T0 + k 2 h  = f  0 + (240 ),1200 + (− 0.38347 ) × 240 
1 1 1 1
 2 2   2 2 
( )
= f (120,1154 .0 ) = −2.2067 × 10 −12 1154 .0 4 − 81 × 108 = −3.8954

k 4 = f (t 0 + h , T0 + k 3h ) = f (0 + 240,1200 + (− 3.894 ) × 240 ) = f (240,265.10 )

= −2.2067 × 10 −12 (265.10 4 − 81 × 108 ) = 0.069750

h
T1 = T0 + ( k1 + 2k 2 + 2k 3 + k 4 )
6
= 1200 +
240
(− 4.5579 + 2(− 0.38347 ) + 2(− 3.8954) + (0.069750))
6
= 675.65 K

T1 is the approximate temperature at t=t 1


t = t 0 + h = 0 + 240 = 240
For i = 1, t1 = 240, T1 = 675.65 K

k1 = f (t1 , T1 ) = f (240,675.65) = −2.2067 × 10 −12 (675.654 − 81 × 108 )

= −0.44199
Numerical Analysis / M. Sc. - 50 - Written by Assoc. Prof.
Chap. 3 Dr. Zaidoon M. Shakoor
   
k 2 = f  t1 + h , T1 + k1h  = f  240 + (240 ),675.65 + (− 0.44199 )240 
1 1 1 1
 2 2   2 2 
= f (360,622.61) = −2.2067 × 10 −12 (622.614 − 81 × 108 )

= −0.31372
   
k 3 = f  t1 + h , T1 + k 2 h  = f  240 + (240 ),675.65 + (− 0.31372 ) × 240 
1 1 1 1
 2 2   2 2 
= f (360,638.00 )
(
= −2.2067 × 10 −12 638.00 4 − 81 × 10 8 )
= −0.34775
k 4 = f (t1 + h , T1 + k 3h ) = f (240 + 240,675.65 + (− 0.34775) × 240 ) = f (480,592.19 )

= 2.2067 × 10 −12 (592.19 4 − 81 × 108 )

= −0.25351
h
T2 = T1 + (k1 + 2k 2 + 2k 3 + k 4 )
6
= 675.65 +
240
(− 0.44199 + 2(− 0.31372) + 2(− 0.34775) + (− 0.25351))
6
= 594.91 K

T2 is the approximate temperature at time t 2


t 2 = t1 + h = 240 + 240 = 480

Table 1 and Figure 2 show the effect of step size on the value of the calculated
temperature at t = 480 seconds.

Table: Value of temperature at time, t = 480 s for different step sizes


Step size, h T(480) E t | εt | %
480 -90.278 737.85 113.94
240 594.91 52.660 8.1319
120 646.16 1.4122 0.21807
60 647.54 0.033626 0.0051926
30 647.57 0.00086900 0.00013419

Example 3.6
Using Matlab Commands solve the following equation using both Eular and
Runge-Kutta method and to approximate the solution of the initial value problem
dy
= x + y, y (0) = 1 with step size h = 0.1.
dx
Numerical Analysis / M. Sc. - 51 - Written by Assoc. Prof.
Chap. 3 Dr. Zaidoon M. Shakoor
Solution:
Eular Runge-Kutta
clear all, clc,format short clear all, clc,format short
x(1)=0; x(1)=0;y(1)=1;h=0.1;
y(1)=1; f=inline('x+y');
h=0.5 % f(x,y) = x+y
for i=1:5 for i=1:5
x(i+1)=x(i)+h; x(i+1)=x(i)+h;
dy=x(i)+y(i); k1 = f(x(i),y(i));
y(i+1)=y(1)+h*dy; k2 = f(x(i)+h/2,y(i)+k1*h/2);
end k3 = f( x(i) + h/2,y(i)+k2*h/2);
y_exact= -1-x+2*exp(x); k4 = f( x(i) + h,y(i)+k3*h);
error=y_exact-y y(i+1)=y(i)+(1/6)*h*(k1 +2*k2 + 2*k3 +k4);
table=[x',y',y_exact',error'] end
y_exact= -1-x+2*exp(x); error=y_exact-y
table=[x',y',y_exact',error']
table = table =
0 1.0000 1.0000 0 0 1.0000 1.0000 0
0.1000 1.1000 1.1103 0.0103 0.1000 1.1103 1.1103 0.0000
0.2000 1.1200 1.2428 0.1228 0.2000 1.2428 1.2428 0.0000
0.3000 1.1320 1.3997 0.2677 0.3000 1.3997 1.3997 0.0000
0.4000 1.1432 1.5836 0.4404 0.4000 1.5836 1.5836 0.0000
0.5000 1.1543 1.7974 0.6431 0.5000 1.7974 1.7974 0.0000

3.3.1 MATLAB Built-In Routines for solving ODES


MATLAB have several sub programs (Routines) to solve ODES;
• ode113:Variable order solution to non-stiff system
• ode15s:Variable order, multistep method for solution of stiff system
• ode23: Lower order adaptive step size routine for non-stiff systems
• ode23s:Lower order adaptive step size routine for stiff systems
• ode45: Higher order adaptive step size routine for non-stiff system

To solve the example 3.3 using MATLAB Routine, We just have to write a
function which returns the rate of change of the vector x.

function dx = example(t,x)
dx(1,1) = t^2+t;

The above file named example.m must be saved in default MATLAB folder then
Numerical Analysis / M. Sc. - 52 - Written by Assoc. Prof.
Chap. 3 Dr. Zaidoon M. Shakoor
the following code must run.

Vspan = 0:0.2:2;
x0=0.5;
[t,x] = ode45('example',Vspan,x0);
plot (t,x,'k*:')
xlabel('Time (t)')
ylabel('x(t)')
legend('Runge Kutta integration')

The numerical solution, computed using ODE45 is given below in Figure (3.3)

5 Runge Kutta integration

4
x(t)

0 0.5 1 1.5 2
Time (t)
Figure (3.3): Solution produced by ODE45.

3.2 Solving Simultaneous First-Order Ordinary Differential Equations


3.2.1 Integration two simultaneous first-order ordinary differential equations
Consider the following system of first-order ODE’s describing the dependence of two
dependent variables y and z on one independent variable x:
dy
= f ( x, y , z )
dx
dz
= g ( x, y , z )
dx
These two differential equations are coupled and must be integrated simultaneously
because both equations involve both dependent variables.
Initial conditions are required giving the values of y and z at the initial value of x. The
algorithm for 4th-order Runge-Kutta integration of two coupled ODEs is:

Numerical Analysis / M. Sc. - 53 - Written by Assoc. Prof.


Chap. 3 Dr. Zaidoon M. Shakoor
h
yi + 1 = yi + ( k11 + 2k 21 + 2k31 + k41 )
6
h
zi + 1 = zi + ( k12 + 2k 22 + 2k32 + k42 )
6

k11 = f ( xi , yi , zi )
k12 = g( xi , yi , zi )
k 21 = f ( xi + 0.5 h , yi + 0.5 hk11 , zi + 0.5 hk12 )
k 22 = g ( xi + 0.5 h , yi + 0.5 hk11 , zi + 0.5 hk12 )
k32 = f ( xi + 0.5 h , yi + 0.5 hk 21 , zi + 0.5 hk 22 )
k32 = g( xi + 0.5 h , yi + 0.5 hk 21 , zi + 0.5 hk 22 )
k41 = f ( xi + h , yi + hk31 , zi + hk32 )
k42 = g( xi + h , yi + hk31 , zi + hk32 )
As example an exothermic reaction in unsteady-state continuous stirred tank reactor
and exothermic reaction in a plug flow reactor with heat exchange through the reactor
wall.
From the one and two ODE examples, you can extend the method to integration of
three coupled ODE’s. Three coupled ODE’s would be encountered, for example, for
reaction of gases in a steady-state non-isothermal plug flow reactor with significant
pressure drop (dC/dx =, dT/dx =, and dP/dx=).

3.2.2 Integration of a system of first-order ordinary differential equations


dy1
= f1(x, y1 , y2 ,…, ym )
dx
dy2
= f 2 (x, y1 , y2 ,…, ym )
dx
.....................
......................
dym
= f m (x, y1 , y2 ,…, ym )
dx
The solution of the above equations is:

h
yin + 1 = yin + (k1,i + 2k 2,i + 2k3,i + k4,i ) Where i = 1, 2, ..., m and
6

k1,i = f i (x n , y1n , y2n …, ymn )

h n hk1,1 n hk1,2 hk
k 2,i = f i (x n + , y1 + , y2 + …, ymn + 1,m )
2 2 2 2

Numerical Analysis / M. Sc. - 54 - Written by Assoc. Prof.


Chap. 3 Dr. Zaidoon M. Shakoor
h n hk 2 ,1 n hk 2 ,2 hk
k3,i = f i (x n + , y1 + , y2 + …, ymn + 2 ,m )
2 2 2 2

k4,i = f i (x n + h , y1n + hk3 ,1 , y2n + hk3 ,2 …, ymn + hk3 ,m )

The idea of the solution to a system of differential equations is similar to a solution of a


single differential equation.

Example 3.7:
Using fourth order Runge-Kutta method with step size h = 0.1 solve
dy1
= y1 y2 + x , y 1 (0) = 1
dx
dy 2
= xy 2 + y 1 , y 2 (0) = -1
dx

To calculate y 1 (0.1) and y 2 (0.1)

Solution
At x = 0, y 1 = 1, y2 = -1
k 1,1 = y1 y2 + x = (1)(-1) + 0 = -1
k 1,2 = xy 2 + y 1 = (0)(-1) + 1= 1
k 2,1 = (y 1 +0.5hk 1,1 )(y 2 +0.5hk 1,2 ) + (x+0.5h) =(0.95)(-0.95) + 0.05 = -0.8525
k 2,2 = (x+0.5h)(y 2 +0.5hk 1,2 ) +(y 1 +0.5hk 1,1 ) =(0.05)(-0.95) + 0.95 = 0.9025
k 3,1 = (y 1 +0.5hk 2,1 )(y 2 +0.5hk 2,2 ) + (x+0.5h) =(0.9574)(-0.9549) + 0.05= -0.8642
k 3,2 = (x+0.5h)(y 2 +0.5hk 2,2 ) +(y 1 +0.5hk 2,1 ) =(0.05)(-0.9549) + 0.9574 = 0.9096
k 4,1 = (y 1 +hk 3,1 )(y 2 +hk 3,2 ) + (x+h) =(0.9136)(-0.9091) + 0.1 = -0.7305
k 4,2 = (x+h)(y 2 +hk 3,2 ) +(y1 +hk 3,1 ) =(0.1)(-0.9091) + 0.9136= 0.8227
at x=0.1
y1 (0.1) = y 1 (0) + (h/6)(k 1,1 + 2k 2,1 + 2k 3,1 + k 4,1 )

y1 (0.1) = 1 +(0.1/6) [(-1) + 2(-0.8525) + 2(-0.864) + (-0.730)]= 0.9139

y2 (0.1) = y 2 (0) +(h/6) (k 1,2 + 2k 2,2 + 2k 3,2 + k 4,2 )

y2 (0.1) = -1 + [(1) + 2(0.9025) + 2(0.909) + (0.823)]= -0.9092

Example 3.8
Use ode45 Matlab Command to solve the following first order system for y1 and y2 at
0 ≤ x ≤ 1.

Numerical Analysis / M. Sc. - 55 - Written by Assoc. Prof.


Chap. 3 Dr. Zaidoon M. Shakoor
dy1
= y1 y2 + x , y1 (0) = 1
dx
dy2
= xy 2 + y1 , y2 (0) = -1
dx
Using fourth order Runge-Kutta method with step size h = 0.1
Solution:
The Matlab routines ode45 can be used to solve the system. A Matlab function must
be created to evaluate the slopes as a column vector. The function name in this
example is exode(x, y) which must be saved first in the hard drive with the same
name exode.m.
function dydx = exode(x,y)
dydx (1,1)=y(1)*y(2)+x;
dydx (2,1)=x*y(2)+y(1);

The command ode45 is then evaluated from the command windows. Matlab will set
the step size to achieve a preset accuracy that can be changed by user.
The independent variable can also be specified at certain locations between the initial
and final values and Matlab will provide the dependent value at these locations.
xspan=0:0.1:1;
[x,y]=ode45('exode',xspan,[1 , -1])
x= y=
0 1.0000 -1.0000
0.1000 0.9139 -0.9092
0.2000 0.8522 -0.8341
0.3000 0.8106 -0.7711
0.4000 0.7863 -0.7174
0.5000 0.7772 -0.6705
0.6000 0.7817 -0.6283
0.7000 0.7987 -0.5889
0.8000 0.8274 -0.5504
0.9000 0.8675 -0.5108
1.0000 0.9188 -0.4681

Exercise 3.9:
Let’s consider a simple example of a model of a plug flow reactor that is described by a
system of ordinary differential equations. A plug flow reactor is operated as shown in
Figure (3.4) below.

Numerical Analysis / M. Sc. - 56 - Written by Assoc. Prof.


Chap. 3 Dr. Zaidoon M. Shakoor
A →
k1
B →
k2
C

Figure (3.4) Isothermal plug flow reactor

The plug flow initially has only reactant A, the components A react to form component
B. The mole balance for each component is given by the following differential
equations
dC A
u = − k 1C A
dz
dC
u B = k 1C A − k 2 C B
dz
dC
u C = k 2CB
dz
With the following initial values
C A (z=0) =1 kmol/m3 C B (z=0) =0 C C (z=0) =0 and k 1 =2 k 2 =3
If u=0.5 m/s and reactor length z=3 m. Solve the differential equations and plot the
concentration of each species along the reactor length
Solution:
We’ll start by writing the function defining the right hand side (RHS) of the ODEs. The
following function file ‘Example’ is used to set up the ode solver.

function dC= Example( z, C)


u = 0.5;
k1=2; k2=3;
dC(1,1) = -k1 *C(1) / u;
dC(2,1) = (k1 *C(1)-k2 *C(2)) / u;
dC(3,1) = k2 *C(2)/ u;

Now we’ll write a main script file to call ode45. CA, CB and CC must be defined
within the same matrix, and so by calling CA as C(1), CB as C(2) and CC as c(3), they
are listed as common to matrix C.
The following run file is created to obtain the solution:

Numerical Analysis / M. Sc. - 57 - Written by Assoc. Prof.


Chap. 3 Dr. Zaidoon M. Shakoor
clear all, clc
[z , C] = ode45(' Example', [0:0.1:3], [1 0 0])
plot (z,C(:,1),'k+-',z,C(:,2),'k*:',z,C(:,3),'kd-.')
xlabel ('Length (m)');
ylabel ('Concentrations (kmol/m^3) ');
legend ('A', 'B', 'C')

The produced plot is as in Figure (3.5)

1
A
Concentrations (kmol/m3)

B
0.8
C

0.6

0.4

0.2

0
0 1 2 3
Length (m)
Figure (3.5): A, B and C concentrations along plug flow reactor

3.2.3 Solving Higher Order Ordinary Differential Equations


We have learned Euler’s and Runge-Kutta methods to solve first order ordinary
differential equations of the form
= f ( x, y ), y (0) = y0
dy
dx
What do we do to solve differential equations that are higher than first order? For
example an n th order differential equation of the form
dny d n −1 y
+ ao y = f (x )
dy
an n
+ a n −1 n −1
+  + a1
dx dx dx
with n − 1 initial conditions can be solved by assuming
y = z1 (1)

dy dz1
= = z2 (2)
dx dx
d 2 y dz 2
= = z3 (3)
dx 2 dx

Numerical Analysis / M. Sc. - 58 - Written by Assoc. Prof.


Chap. 3 Dr. Zaidoon M. Shakoor

d n −1 y dz n −1
= = zn (n)
dx n −1 dx
d n y dzn
=
dx n dx
1  d n −1 y 
− a0 y + f ( x )
dy
=  − an −1 n −1  − a1
an  dx dx 

=
1
(− a n−1 z n  − a1 z 2 − a0 z1 + f (x )) (n+1)
an
The above Equations from (2) to (n+1) represent n first order differential equations as
follows
dz1
= z 2 = f 1 ( z1 , z 2 , , x )
dx
dz 2
= z 3 = f 2 ( z1 , z 2 , , x )
dx

dz n
=
1
(− a n−1 z n  − a1 z 2 − a0 z1 + f (x ))
dx an
Each of the n first order ordinary differential equations is accompanied by one initial
condition. These first order ordinary differential equations are simultaneous in nature
but can be solved by the methods used for solving first order ordinary differential
equations that we have already learned.

Higher Order Ordinary Differential System of first Order Ordinary


Equations Differential Equations
d2y dy dy
2
− 2 + 2 y = e 2 x sin x =z , y (0) = −0.4
dx dx dx
with dz
= e 2 x sin x − 2 y + 2 z , z (0) = −0.6
dy dx
y(0)= -0.4 , (0) = −0.6
dx
d3y d2y dy dy
3
+2 2 −3 = x 2 + y, = z, y (0) = 4
dx dx dx dx
with dz
2
=u, z (0) = 2
d y dy dx
2
(0) = 1, (0) = 2 , y (0) = 4
dx dx du
= x 2 + y − 2u + 3z , u (0) = 1
dx
d4y dy y dy
4
+ − + 3y = , = z, y ( 0) = 1
dx dx x dx

Numerical Analysis / M. Sc. - 59 - Written by Assoc. Prof.


Chap. 3 Dr. Zaidoon M. Shakoor
with dz
=u, z (0) = 1
d3y d2y dy dx
3
( 0 ) = 0 . 5, 2
(0) = 0.25, (0) = 1 , y (0) = 1
dx dx dx du
=v, u (0) = 0.25
dx
dv y
= − 3y + z , v(0) = 0.5
dx x

Example 3.10
Re-write the following differential equation as a set of first order differential equations.
d2y
+ 2 + 5 y = e − x , y (0) = 5, y ′(0) = 7
dy
3 2
dx dx
Solution
The ordinary differential equation would be rewritten as follows. Assume
dy d 2 y dz
= z, Then =
dx dx 2 dx
Substituting this in the given second order ordinary differential equation gives
dz
3 + 2z + 5 y = e −x
dx
dz 1 − x
(
= e − 2z − 5 y
dx 3
)
The set of two simultaneous first order ordinary differential equations complete with
the initial conditions then is
= z, y (0) = 5
dy
dx

= (e − 2 z − 5 y ), z (0) = 7 .
dz 1 − x
dx 3
Now one can apply any of the numerical methods used for solving first order ordinary
differential equations.

Example 3.11
Given the third-order ordinary differential equation and associated initial conditions
d3y d2y dy y (0) = 4, dy d2y
3
+ 3 2 + 5 + y = x2 , = 0.6 , 2 = 0.22
dx dx dx dx x =0 dx x =0

a. Write this differential equation as a system of first-order ordinary differential


equations
b. Using fourth order Runge-Kutaa method to estimate y(0.1) and y(0.2) taking
Δx=0.1
Numerical Analysis / M. Sc. - 60 - Written by Assoc. Prof.
Chap. 3 Dr. Zaidoon M. Shakoor
a)
dy
= z , y(0) = 4
dx
dz
= u , z(0) = 0.6
dx
du
= x 2 − 3u − 5z − y , u (0) = 0.22
dx
b) Solution (1)
First step of integration x=0 , y=4 , z=0.6 , u=0.22 , Δx=0.1
k11= z =0.6000
k21= u= 0.2200
k31=x2-3×u-5×z-y= 02-3×0.22-5×0.6-4=-7.6600
k12=z+0.5×Δx×k21=0.6+0.5×0.1×0.22= 0.6110
k22=(u+0.5×Δx×k31) =(0.22+0.5×0.1×(-7.6600)) = -0.1630
k32=(x+0.5×Δx)2 -3×(u+0.5×Δx×k31)-5×(z+0.5×Δx×k21)-(y+0.5×Δx×k11)
=(0+0.5×0.1) 2-3×(0.22+0.5×0.1×(-7.66))-5×(0.6+0.5×0.1×0.22)-(4+0.5×0.1×0.6)
= -6.5935
k13=z+0.5×Δx×k22=0.6+0.5×0.1×(-0.1630)= 0.5918
k23=u+0.5×Δx×k32 =0.22+0.5×0.1×(-6.5935) = -0.1097
k33=(x+0.5×Δx) 2 -3×(u+0.5×Δx×k32)-5×(z+0.5×Δx×k22)-(y+0.5×Δx×k12)
=(0+0.5×0.1)2-3×(0.22+0.5×0.1×(-6.5935))-5×(0.6+0.5×0.1×(-0.1630))-(4+0.5×0.1×
0.6110) = -6.6583
k14=z+Δx×k23=0.6+0.1×(-0.1097)= 0.5890
k24=u+Δx×k33=0.22+0.1×(-6.6583) = -0.4458
k34=(x+Δx) 2 -3×(u+Δx×k33)-5×(z+Δx×k23)-(y+Δx×k13)
=(0+0.1)2-3×(0.22+0.1×(-6.6583))-5×(0.6+0.1×(-0.1097))-(4+0.1×0.5918) = -5.6569
x=x+Δx=0+0.1= 0.1000
y=y+Δx/6×(k11+2×k12+2×k13+k14)=4+0.1/6×(0.6+2×0.6110+2×0.5918+0.5890)
= 4.0599
z=z+Δx/6*(k21+2*k22+2*k23+k24)=0.6+0.1/6 ×(0.2200+2×(-0.1630)+2×(-0.1097)+
( -0.4458))= 0.5871
u=u+Δx/6*(k31+2*k32+2*k33+k34)=u+0.1/6 ×(-7.6600+2×(-6.5935)+2×(-6.6583)+
(-5.6569))=-0.4437
dy dz d 2 y
Then at x=0.1 , y=4.0599, z = = 0.5871 , u= = = -0.4437
dx dx dx 2
Numerical Analysis / M. Sc. - 61 - Written by Assoc. Prof.
Chap. 3 Dr. Zaidoon M. Shakoor
Second step of integration x=0.1 , z=0.5871 , u=-0.4437 , Δx=0.1
k11= z = 0.5871
k21= u= -0.4437
k31=x2-3×u-5×z-y= 0.12-3×(-0.4437)-5×(0.5871)- 4.0599=-5.6546
k12=z+0.5×Δx×k21=0.5871+0.5×0.1×(-0.4437)= 0.5650
k22=(u+0.5×Δx×k31) =((-0.4437)+0.5×0.1×(-5.6546)) = -0.7264
k32=(x+0.5×Δx)2-3×(u+0.5×Δx×k31)-5×(z+0.5×Δx×k21)-(y+0.5×Δx×k11)
=(0.1+0.5×0.1)2-3×(-0.4437+0.5×0.1×(-5.6546))-5×(0.5871+0.5×0.1×(-0.4437))-(
4.0599+0.5×0.1×0.5871) = -4.7124
k13=z+0.5×Δx×k22=0.5871+0.5×0.1×(-0.7264)= 0.5508
k23=u+0.5×Δx×k32 =-0.4437+0.5×0.1×(-4.7124) = -0.6793
k33=(x+0.5×Δx)2 -3×(u+0.5×Δx×k32)-5×(z+0.5×Δx×k22)-(y+0.5×Δx×k12)
=(0.1+0.5×0.1)2-3×(-0.4437+0.5×0.1×(-4.7124))-5×(0.5871+0.5×0.1×(-0.7264))-(
4.0599+0.5×0.1×0.5650) = -4.7819
k14=z+Δx×k23=0.5871+0.1×(-0.6793)= 0.5192
k24=u+Δx×k33=-0.4437+0.1×(-4.7819) = -0.9219
k34=(x+Δx)2-3×(u+Δx×k33)-5×(z+Δx×k23)-(y+Δx×k13)
=(0.1+0.1)2-3×(-0.4437+0.1×(-4.7819))-5×(0.5871+0.1×(-0.6793))-(4.0599+0.1×0.5
508) = -3.9055
x=x+Δx=0.1+0.1= 0.2
y=y+Δx/6×(k11+2×k12+2×k13+k14)=
4.0599+0.1/6×(0.5871+2×0.5650+2×0.5508+0.5192)= 4.1155
z=z+Δx/6*(k21+2*k22+2*k23+k24)=
0.5871+0.1/6×((-0.4437)+2×(-0.7264)+2×(-0.6793)+( -0.9219))= 0.5175
u=u+Δx/6*(k31+2*k32+2*k33+k34) =
-0.4437 +0.1/6×((-5.6546) +2×( -4.7124 )+2×(-4.7819)+ (-3.9055 ))= -0.9195
dy dz d 2 y
Then at x=0.2 , y=4.1155, z = = 0.5175 , u= = = -0.9195
dx dx dx 2

Solution 2 : Using Matlab commands:


We’ll start by writing the function defining the right hand side (RHS) of the ODEs. The
following function file ‘Ex’ is used to set up the ode solver.

Numerical Analysis / M. Sc. - 62 - Written by Assoc. Prof.


Chap. 3 Dr. Zaidoon M. Shakoor
function dq = Ex(x,q)
y=q(1);z=q(2);u=q(3);
dq(1,1)=z;
dq(2,1)=u;
dq(3,1)=x^2-3*u-5*z-y;

The following run file is created to obtain the solution:


clear all,clc,format compact
[x,q]=ode45('Ex',[0:0.1:0.2],[4,0.6,0.22])
y=q(:,1)

The produced results will be


x=
0
0.1000
0.2000
q=
4.0000 0.6000 0.2200
4.0599 0.5871 -0.4437
4.1155 0.5175 -0.9195
y=
4.0000
4.0599
4.1155

Numerical Analysis / M. Sc. - 63 - Written by Assoc. Prof.


Chap. 3 Dr. Zaidoon M. Shakoor
Chapter 4: Solution of Partial Differential Equations

Classification of second-order linear equations

Examples of different PDEs are:

(a) Elliptic
∂ 2u ∂ 2u
+ 2 =0 (Laplace’s equation)
∂x 2 ∂y
∂ 2u ∂ 2u
+ 2 +e=0 (Poisson’s equation)
∂x 2 ∂y
(b) Parabolic
∂u ∂ 2u
=α 2 ; where α is a positive constant
∂y ∂x

∂u ∂ 2u ∂ 2u
= α ( 2 + 2 ) + e; (Parabolic in time t and elliptic in spatial dimensions x
∂t ∂x ∂y
and y)
(c) Hyperbolic
∂ 2u 2∂ u
2
− c =0 (wave’s equation)
∂t 2 ∂x 2

The unsteady state heat conduction equation is a parabolic differential equation


ρCp ∂T = ∇⋅(k∇T) + q′′′
∂t

For constant physical properties and no heat generation, the heat conduction equation
becomes
ρCp ∂T = k∇2T or ∂T = α∇2T
∂t ∂t
k
Where: α = is the thermal diffusivity of the materials.
ρc p
We must consider changes in time as well as in space for parabolic equations. The
domain of elliptic equations is a closed region whereas the domain of parabolic
equations is an open region in time.

Numerical Analysis / M. Sc. 64 Written by Assoc. Prof.


Chap. 4 Dr. Zaidoon M. Shakoor
4.1 Numerical Differentiation
Numerical Differentiation is a method used to approximate the value of a derivative
over a continuous region [a,b].
Let f(x) is a continuous function with step size h. There are forward, backward and
centered difference methods to approximate the derivatives of f(x) at a point x i .

4.1.1 Forward Difference Approximation of the First Derivative


We know
f ( x + ∆x ) − f ( x )
f ′( x ) = lim
∆x → 0 ∆x
f ( x + ∆x ) − f ( x )
For a finite ' ∆x' . f ′(x ) ≅
∆x
f (x)

x x + ∆x x

Figure 4.1: Graphical representation of forward


difference approximation of first derivative

So if you want to find the value of f ′(x ) at x = xi , we may choose another point ' Δx'
ahead as x = xi +1 . This gives
f ( xi +1 ) − f ( xi )
f ′( xi ) ≅
∆x
f ( xi +1 ) − f ( xi )
= Where Δx = xi +1 − xi
xi +1 − xi

4.1.2 Backward Difference Approximation of the First Derivative


We know
f ( x + ∆x ) − f ( x )
f ′( x ) = lim
∆x →0 ∆x
For a finite ' Δx' ,

Numerical Analysis / M. Sc. 65 Written by Assoc. Prof.


Chap. 4 Dr. Zaidoon M. Shakoor
f ( x + ∆x ) − f ( x )
f ′( x ) ≅
∆x
If ' Δx' is chosen as a negative number,
f (x + ∆x ) − f (x )
f ′(x ) ≅
∆x
f (x ) − f (x − ∆x )
f ′(x ) =
∆x
This is a backward difference approximation as you are taking a point backward
from x . To find the value of f ′(x ) at x = xi , we may choose another point ' Δx' behind
as x = xi −1 . This gives
f ( xi ) − f (xi −1 )
f ′( xi ) ≅
∆x
f ( xi ) − f ( xi −1 )
= where ∆x = xi − xi −1
xi − xi −1

f (x)

x
x − ∆x x
Figure 4.2 Graphical representation of backward
difference approximation of first derivative

4.1.3 Central Difference Approximation of the First Derivative


As shown above, both forward and backward divided difference approximation of the
first derivative are accurate on the order of 0(Δx ) . Can we get better approximations?
Yes, another method to approximate the first derivative is called the Central
difference approximation of the first derivative.
From Taylor series
f ′′( xi ) ′′′( )
f ( xi +1 ) = f ( xi ) + f ′( xi )Δx + (Δx )2 + f xi (Δx )3 +  (1)
2! 3!
f ′′( xi )
(∆x )2 − f (xi ) (∆x )3 + 
′′′
f ( xi −1 ) = f ( xi ) − f ′( xi )∆x + (2)
2! 3!
Subtracting equation (2) from equation (1)

Numerical Analysis / M. Sc. 66 Written by Assoc. Prof.


Chap. 4 Dr. Zaidoon M. Shakoor
2 f ′′′( xi )
f ( xi +1 ) − f ( xi −1 ) = f ′( xi )(2∆x ) + (∆x )3 + 
3!
f (xi +1 ) − f ( xi −1 ) f ′′′( xi )
f ′( xi ) = − (∆x )2 + 
2∆x 3!
f ( xi +1 ) − f (xi −1 )
f ′( xi ) = + 0(∆x )
2
2∆x
f (xi +1 ) − f (xi −1 )
f ′( xi ) =
2∆x
Hence showing that we have obtained a more accurate formula as the error is of the
order of 0(∆x )2 .
f (x)

x − ∆x x x + ∆x

Figure 4.3 Graphical Representation of central


difference approximation of first derivative.

4.1.4 Higher Order Derivatives

Example: Second order derivative:


Note that for the centered formulation, it is a derivation of a derivative:
f (xi +1 ) − f ( xi ) f (xi ) − f ( xi −1 )

∆x ∆x f ( xi +1 ) − 2 f ( xi ) + f ( xi −1 )
f ' ' (x ) ≅ =
∆x (∆x) 2
f (xi + 2 ) − 2 f ( xi +1 ) + f ( xi )
f ' ' (x ) ≅
Forward
(∆x) 2
f (xi ) − 2 f ( xi −1 ) + f ( xi − 2 )
f ' ' (x ) ≅
Backward
(∆x) 2
f ( xi +1 ) − 2 f ( xi ) + f ( xi −1 )
f ' ' (x ) ≅
Centered
(∆x) 2

I) Forward Difference Methods


First Derivative
Numerical Analysis / M. Sc. 67 Written by Assoc. Prof.
Chap. 4 Dr. Zaidoon M. Shakoor
f (xi +1 ) − f (xi )
f ' ( xi ) =
∆x
Second Derivative
f ( xi + 2 ) − 2 f ( xi +1 ) + f ( xi )
f ' ' ( xi ) =
(∆x) 2
Third Derivative
f ( xi + 3 ) − 3 f ( xi + 2 ) + 3 f ( xi +1 ) − f ( xi )
f (3) ( xi ) =
(∆x)3
Fourth Derivative
f (xi + 4 ) − 4 f ( xi + 3 ) + 6 f (xi + 2 ) − 4 f ( xi +1 ) + f (xi )
f (4 ) ( xi ) =
(∆x) 4
II) Backward Difference Methods
First Derivative
f ( xi ) − f ( xi −1 )
f ' ( xi ) =
∆x
Second Derivative
f ( xi ) − 2 f ( xi −1 ) + f ( xi − 2 )
f ' ' ( xi ) =
(∆x) 2
Third Derivative
f ( xi ) − 3 f ( xi −1 ) + 3 f ( xi − 2 ) − f ( xi −3 )
f (3) ( xi ) =
(∆x)3
Fourth Derivative
f ( xi ) − 4 f ( xi −1 ) + 6 f ( xi − 2 ) − 4 f ( xi − 3 ) + f (xi − 4 )
f (4 ) ( xi ) =
(∆x) 4
III) Central Difference Methods
First Derivative
f ( xi +1 ) − f ( xi −1 )
f ' ( xi ) =
2∆x
Second Derivative
f (xi +1 ) − 2 f (xi ) + f (xi −1 )
f ' ' ( xi ) =
(∆x) 2
Third Derivative
f ( xi + 2 ) − 2 f ( xi +1 ) + 2 f ( xi −1 ) − f ( xi − 2 )
f (3) ( xi ) =
2(∆x)3
Fourth Derivative
f (xi + 2 ) − 4 f ( xi +1 ) + 6 f ( xi ) − 4 f ( xi −1 ) + f ( xi − 2 )
f (4 ) ( xi ) =
(∆x) 4

Numerical Analysis / M. Sc. 68 Written by Assoc. Prof.


Chap. 4 Dr. Zaidoon M. Shakoor
4.2 Solution of Partial Differential Equations
4.2.1 Explicit Method
Let consider the one dimensional unsteady heat conduction equation
∂T = α ∂ T
2

∂t ∂x 2
We have an explicit method if the time derivative is approximated by a forward
difference and the second spatial derivative is approximated by a central difference
n +1
∂T = Ti − Ti
n

∂t ∆t
∂ 2T Ti +n1 − 2Ti n + Ti −n1
=
∂x 2 ∆x 2
In finite difference form, the heat conduction equation becomes
Ti n +1 − Ti n Ti +n1 − 2Ti n + Ti −n1

∆t ∆x 2
The explicit scheme is depicted in Figure 4.4 where three nodes in the previous time
tn are required to compute the node at the present time tn+1.
xi-1 xi xi+1

Grid point involved in space difference


n
t

Grid point involved in time difference

n+1
t

Figure (4.4): A computational diagram for the explicit method.

The temperature at node i can be obtained from the three previous time nodes as
α∆t  α∆t 
Ti n +1 = ( Ti +n1 + Ti −n1 ) + 1 − 2  Ti n
( ∆x ) 2
 ( ∆x ) 
2

A solution is stable when the errors at any stage of the computation are not amplified
but are attenuated as the computation progresses. For stable solution
 α∆t  α∆t 1
1 − 2  ≥0⇒
2 

 ( ∆x )  ( ∆x ) 2
2

Example 4.1:
∂ T 2

Solve the partial differential equation ∂T = 0.02 2 with the following initial and
∂t ∂x
boundary conditions Initial conditions: T(x, 0) = 100x for 0 ≤ x ≤ 0.5; T(x, 0) =
100(1− x) for 0.5 ≤ x ≤ 1 Boundary conditions: T(0, t) = 0; T(1, t) = 0

Numerical Analysis / M. Sc. 69 Written by Assoc. Prof.


Chap. 4 Dr. Zaidoon M. Shakoor
Solution:
α∆t  α∆t 
Ti n +1 = ( Ti +n1 + Ti −n1 ) + 1 − 2  Ti n
( ∆x ) 2
 ( ∆x ) 
2

Let ∆x = 0.2, ∆t = 0.5, we have


α∆t (0.02)(0.5) 1
= = 0.25 ≤
( ∆x ) 2
(0.2) 2
2
Ti n +1 = 0.25( Ti +n1 + Ti −n1 ) + 0.5 Ti n
The problem is symmetric about the value x = 1. Therefore at the node i
corresponding to x = 1, we have Ti +n1 = Ti −n1
Ti n +1 = 0.25( Ti −n1 + Ti −n1 ) + 0.5 Ti n = 0.5( Ti −n1 + Ti n )
Solution

x=0:0.1:1;
T=[0 20 40 60 80 100 80 60 40 20 0];
k=1;
plot(x,T,'k-*')
for t=0.5 :0.5:5
k=k+1;
T(k,1)=0;
T(k,11)=0;
for i=2 :10
T(k,i)=0.25*(T(k-1,i+1)+T(k-1,i-1))+0.5*T(k-1,i);
end
hold on
plot(x,T(k,:),'k-*')
hold off
end
xlabel('x'),ylabel ('Temperature')

The result of the MATLAB program is in Figure (4.5).

Numerical Analysis / M. Sc. 70 Written by Assoc. Prof.


Chap. 4 Dr. Zaidoon M. Shakoor
100

90

80

70

Temperature
60

50

40

30

20

10
Time
0
0 0.1 0.2 0.3 0.4 0.5 0.6 0.7 0.8 0.9 1
x
Figure (4.5): Temperature distribution using explicit method.

If you need to plot your results in three dimension plot, try the code after the above
program:
[X,Time]=meshgrid(x,[0:0.5:5])
surf(X,Time,T)
xlabel('x')
ylabel ('Time')
zlabel ('Temperature')

The result of the MATLAB program is in Figure (4.6).

100
Temperature

80

60

40

20

0
6
1
4
0.8
0.6
2 0.4
0.2
0 0
Time x
Figure (4.6): Three dimensional temperature distribution using explicit method.

Numerical Analysis / M. Sc. 71 Written by Assoc. Prof.


Chap. 4 Dr. Zaidoon M. Shakoor
Example 4.2:
Solve the heat diffusion problem of a steel bar of length 2 m with one end held at 100
o
C the other end at 0 oC and initially the bar is at 0 oC.
dT d 2T
=D 2 Where for steel D = 0.02 m2/hr. The boundary and initial conditions can
dt dx
be written T(0, t) = 100, T(2, t) = 0, T(x,0) = 0
Find the temperature distribution at times t = 4,8,12. . .20 hours, then plot your results
Solution:

L=2,dt=1,D=0.02;dx=sqrt(dt*D/.25);x=0:dx:L;
nx=(L+dx)/dx;T(1,1:nx)=0;T(1,1)=0;
time(1)=0;k=1;
for t=dt:dt:20
k=k+1
T(k,1)=100
for i=2:nx-1
T(k,i)=.25*(T(k-1,i+1)+T(k-1,i-1))+.5*T(k-1,i)
end
end
plot(x,T(4,:),'k-+'x,T(8,:),'k-^',x,T(12,:),'k-*',x,T(20,:),'k-o')
xlabel('length'),ylabel('temperature')
legend('T4','T8','T12','T20')

The result of the MATLAB program is in Figure (4.7)

100
T4
90 T8
T12
80
T20
70

60
temperature

50

40

30

20

10

0
0 0.2 0.4 0.6 0.8 1 1.2 1.4 1.6 1.8 2
length

Figure (4.7): Temperature distribution.

Numerical Analysis / M. Sc. 72 Written by Assoc. Prof.


Chap. 4 Dr. Zaidoon M. Shakoor
4.2.2 Gauss-Seidel Method
The Gauss-Seidel method may be used to solve a set of linear or nonlinear algebraic
equations. We will illustrate the method by solving a heat transfer problem. For
steady state, no heat generation, and constant k, the heat conduction equation is
simplified to Laplace equation
∂ 2T ∂ 2T
+ =0
∂x 2 ∂y 2
The above equation can be put in the finite difference form. We divide the medium of
interest into a number of small regions and apply the heat equation to these regions.
Each sub-region is assigned a reference point called a node or a nodal point. The
average temperature of a nodal point is then calculated by solving the resulting
equations from the energy balance. Accurate solutions can be obtained by choosing a
fine mesh with a large number of nodes.

Example 4.3
A long column with thermal conductivity k = 1 W/m⋅oK is maintained at 500oK on
three surfaces while the remaining surface is exposed to a convective environment
with h = 10 W/m2⋅oK and fluid temperature T ∞ . The cross sectional area of the
column is 1 m by 1 m. Using a grid spacing ∆x = ∆y = 0.25 m, determine the steady-
state temperature distribution in the column and the heat flow to the fluid per unit
length of the column.
Solution
The cross sectional area of the column is divided into many sub-areas called a grid or
nodal network with 25 nodes as shown in Figure 4.8. There are 12 nodal points with
unknown temperature, however only 8 unknowns need to be solved due to symmetry
so that the nodes to the left of the centerline are the same as those to the right.

∆x
500 K
m, n+1
∆y
1 2 1

500 K 3 4 3
m-1, n m, n m+1, n
5 6 5 500 K

7 8 7
h= 300 K 2
m, n-1
h = 10 W/m K
Figure (4.8): The grid for the column cross sectional area.

Numerical Analysis / M. Sc. 73 Written by Assoc. Prof.


Chap. 4 Dr. Zaidoon M. Shakoor
The energy balance is now applied to the control volume ∆x×∆y×1 belongs to node 1
which is an interior node. To make the derivation general, node 1 can be considered
as a node with index (m, n) in a two-dimensional grid as shown in Figure 1. The
directions of conduction heat flow are assumed to be the positive x and y directions.
For steady state with no heat generation
q (m-1, n)→(m, n) + q (m, n-1)→(m, n) = q (m, n)→(m+1, n) + q (m, n)→(m, n+1) (1)
Where: q (m-1, n)→(m, n) is the conduction heat flow between nodes (m-1, n) and (m, n).
Fourier’s law can be used to obtain
Tm−1,n − Tm ,n
q (m-1, n)→(m, n) = k(∆y×1)
∆x
Tm−1,n − Tm ,n
Where: (∆y×1) is the heat transfer area with a unit depth and is the finite-
∆x
difference approximation to the temperature gradient at the boundary between the
two nodes. Appling Fourier’s law to each term in Equation (1) yields
Tm−1,n − Tm ,n T −T T −T T −T
k(∆y×1) + k(∆x×1) m,n−1 m,n = k(∆y×1) m,n m+1,n + k(∆x×1) m,n m,n+1
∆x ∆y ∆x ∆y
The equation is divided by k(∆x×∆y×1) and simplified to
Tm+1,n − 2Tm ,n + Tm−1,n T − 2Tm ,n + Tm ,n −1
+ m,n+1 =0 (2)
∆x 2
∆y 2
For ∆x = ∆y, Eq. (2) becomes
Tm+1,n + Tm ,n +1 + Tm−1,n + Tm ,n −1
T m,n = (3)
4

The above result shows that the temperature of an interior node is just the average of
the temperatures of the four adjoining nodal points. Using this formula, the
temperatures for the first six nodes are
1
T1 = (T 2 + T 3 + 500 + 500)
4
1
T2 = (T 1 + T 4 + T 1 + 500)
4
1
T3 = (T 1 + T 4 + T 5 + 500)
4
1
T4 = (T 2 + T3 + T6 + T3 )
4
1
T5 = (T 3 + T 6 + T 7 + 500)
4
1
T6 = (T 4 + T5 + T8 + T5 )
4
Nodes 7 and 8 are not interior points; therefore Eq. (3) is not applicable.

Numerical Analysis / M. Sc. 74 Written by Assoc. Prof.


Chap. 4 Dr. Zaidoon M. Shakoor
∆x
500 K

∆y
1 2 1

500 K 3 4 3 5 6

5 6 5 500 K

7 8 7
500 K 7 8 7
h = 300 K
h = 10 W/m2K
Figure (4.9): Directions of heat flow for nodes 7 and 8.

Making energy balance for node 7 yields


∆y 500 − T7 T −T ∆y T −T
k( ×1) + k(∆x×1) 5 7 = k( ×1) 7 8 + h(∆x×1)(T 7 − 300)
2 ∆x ∆y 2 ∆x
2
Multiplying the above equation by we obtain
k
2h∆x
500 − T 7 + 2T 5 − 2T 7 = T 7 − T 8 + (T 7 − 300)
k
2h∆x 2 × 10 × 0.25
= = 5.0
k 1
1
T 7 = (2T 5 + T 8 + 2000)
9
Similarly an energy balance for node 8 yields
∆y T −T T −T ∆y T −T
k( ×1) 7 8 + k(∆x×1) 6 8 = k( ×1) 8 7 + h(∆x×1)(T 8 − 300)
2 ∆x ∆y 2 ∆x
2
Multiplying the above equation by we obtain
k
2h∆x
T 7 − T 8 + 2T 6 − 2T 8 = T 8 − T 7 + (T 8 − 300)
k
1
T 8 = (2T 6 + 2T 7 + 1500)
9

We have 8 linear equations with 8 unknowns that can be solved by matrix method or
iterations. As below the MATLAB program using Gauss-Seidel iteration to solve for
the temperatures.

Numerical Analysis / M. Sc. 75 Written by Assoc. Prof.


Chap. 4 Dr. Zaidoon M. Shakoor
T(1:8)=400;
for i=1:100
Tsave=T;
T(1)=0.25*(T(2)+T(3)+1000);
T(2)=0.25*(2*T(1)+T(4)+500);
T(3)=0.25*(T(1)+T(4)+T(5)+500);
T(4)=0.25*(T(2)+2*T(3)+T(6));
T(5)=0.25*(T(3)+T(6)+T(7)+500);
T(6)=0.25*(T(4)+2*T(5)+T(8));
T(7)=(2*T(5)+T(8)+2000)/9;
T(8)=(2*T(6)+2*T(7)+1500)/9;
eT=abs(T-Tsave);
if max(eT)<0.01; break; end
end
T

The program results will be


T=
489.2984
485.1473
472.0580
461.9985
436.9452
418.7346
356.9933
339.0507

4.2.3 Gaussian elimination Method


These 8 equations of example 4.3 can also be solve by matrix method.
1
T1 = (T 2 + T 3 + 500 + 500)
4
1
T2 = (T 1 + T 4 + T 1 + 500)
4
1
T3 = (T 1 + T 4 + T 5 + 500)
4
1
T4 = (T 2 + T 3 + T 6 + T 3 )
4
1
T5 = (T 3 + T 6 + T 7 + 500)
4
1
T6 = (T 4 + T 5 + T 8 + T 5 )
4
1
T7 = (2T 5 + T 8 + 2000)
9
1
T8 = (2T 6 + 2T 7 + 1500)
9

Numerical Analysis / M. Sc. 76 Written by Assoc. Prof.


Chap. 4 Dr. Zaidoon M. Shakoor
A=[1 , -1/4 , -1/4 , 0 , 0 , 0 , 0 , 0
-2/4 , 1 , 0 , -1/4 , 0 , 0 , 0 , 0
-1/4 , 0 , 1 , -1/4 , -1/4 , 0 , 0 , 0
0 , -1/4 , -2/4 , 1 , 0 , -1/4 , 0 , 0
0 , 0 , -1/4 , 0 , 1 , -1/4 , -1/4 , 0
0 , 0 , 0 , -1/4 , -2/4 , 1 , 0 , -1/4
0 , 0 , 0 , 0 , -2/9 , 0 , 1 , -1/9
0 , 0 , 0 , 0 , 0 , -2/9 , -2/9 , 1];
b=[1000/4;500/4;500/4;0;500/4;0;2000/9;1500/9]
T=A\b
The result will be:
T=
489.3047
485.1538
472.0651
462.0058
436.9498
418.7393
356.9946
339.052

Example 4.4:
Solve for the steady-state temperature in a rectangular slab that is 20 cm wide and 10
cm high. All edges are kept at 0oC except the right edge, which is at 100oC. No heat
gain or loss from the top and bottom surface of the slab as shown in Figure 4.10. Let
∆x = ∆y = 2.5 cm.

top surface

y
b

bottom surface

0 a x

Figure (4.10): A thin rectangular plate with insulated top and bottom surfaces

Solution
The two-dimensional heat conduction equation for steady state, no heat generation,
and k independent of T is given
∂ 2T ∂ 2T
+ =0
∂x 2 ∂y 2
Let i be the index in the x direction and j be the index in the y direction as shown in
Figure 3, the finite difference form of the two-dimensional heat conduction is
Numerical Analysis / M. Sc. 77 Written by Assoc. Prof.
Chap. 4 Dr. Zaidoon M. Shakoor
Ti +1,n − 2Ti , j + Ti −1, j Ti , j +1 − 2Ti , j + Ti , j −1
+ =0
∆x 2
∆y 2
For ∆x = ∆y, the temperature at the node (i, j) can be obtained as
Ti +1, j + Ti , j +1 + Ti −1, j + Ti , j −1
T i,j =
4
y
5

j3

1
1 2 3 4 5 6 7 8 9 x
i
Figure (4.11): The grid of the rectangular plate with insulated top and bottom
surfaces

From the boundary conditions


T(i, j = 1) = 0; T(i, j = 5) = 0
T(i = 1, j) = 0; T(i = 9, j) = 100

The temperature distribution can be solved by the successive-over-relaxation method


for which
T(i, j) = 0.25[T(i, j−1) + T(i, j+1) + T(i−1, j) + T(i+1, j)]
As below the MATLAB program to iterate for the temperature distribution using the
initial guesses of 5 for the interior nodes. The iteration is stopped when the maximum
difference between two successive iterations is less than 0.01.
T(1:9,1:5)=5;
T(:,1)=0;T(:,5)=0; T(1,:)=0;T(9,:)=100;
T_initial=T;
for n=1:100
Tsave=T;
for i=2:8
for j=2:4
T(i,j)=0.25*(T(i,j-1) + T(i,j+1) + T(i-1,j) + T(i+1,j));
end
end
eT=abs(T-Tsave);
if max(eT)<0.01,
break
end
end
T_final=T
Numerical Analysis / M. Sc. 78 Written by Assoc. Prof.
Chap. 4 Dr. Zaidoon M. Shakoor
The result will be as below
T_final =
0 0 0 0 0
0 0.3444 0.4888 0.3472 0
0 0.8999 1.2740 0.9043 0
0 1.9960 2.8158 2.0007 0
0 4.2829 6.0046 4.2872 0
0 9.1434 12.6425 9.1467 0
0 19.6570 26.2823 19.6591 0
0 43.2074 53.1743 43.2083 0
100.0000 100.0000 100.0000 100.0000 100.0000

Example 4.5:
The steady state concentration profile in 0.1 m radius spherical particle can be
described by the following equation:
∂ 2 C A 2 ∂C A
D Ae ( + ) − k1C A = 0
∂r 2 r ∂r
The boundary conditions: C A = C AS at r = R and dC A /dr = 0 at r = 0
With the aid of finite difference approximation and Gaussian elimination method,
solve system of linear equations and plot the concentration profile taken 20 points
radialy in the sphere if you know that:
Effective diffusivity; D Ae =2×10-7 m2/s
Reaction rate constant: K 1 =1×10-3
Concentration in sphere surface: C As =500 mol/m3

Figure (4.12) Spherical catalyst particle

Solution
Gaussian elimination Method

Numerical Analysis / M. Sc. 79 Written by Assoc. Prof.


Chap. 4 Dr. Zaidoon M. Shakoor
∂ 2 C A 2 ∂C A
D Ae ( + ) − k1C A = 0
∂r 2 r ∂r . . . . (1)
CA j +1 − 2CA j + CA j −1 2 CA j +1 − CA j −1 k
+ − 1 CA j = 0 . . . . (2)
(∆r ) 2
rj 2∆r D AE
1 1 2 k 1 1
( − )CA j +1 − ( + 1 )CA j + ( + )CA j −1 = 0 . . . . (3)
(∆r ) r j ∆r
2
(∆r ) 2
D AB (∆r ) r j ∆r
2

Solving eq. (3) using Gaussian elimination Method


DAE=2e-7;% m2/s
R=0.1;% m
K1=1e-3; CAs=500;% mol/m3
np=20; dr=R/(np-1);
r=0:dr:R;
A(1,1)=1;B(1,1)=0;
for j=2:np-1
A(j,j-1)=(1/dr^2)-(1/(r(j)*dr));
A(j,j)=-2*(1/dr^2)-K1/DAE;
A(j,j+1)=(1/dr^2)+(1/(r(j)*dr));
B(j,1)=0;
end
A(np,np)=1;B(np,1)=CAs;
CA=A\B
plot(r,CA)
xlabel('Radius (m)')
ylabel('Concentration (mol/m3)')
The results will be such in figure (4.13)

500

400
Concentration (mol/m3)

300

200

100

0
0 0.02 0.04 0.06 0.08 0.1
Radius (m)

Figure (4.13) Concentration distribution with respect to radius

Numerical Analysis / M. Sc. 80 Written by Assoc. Prof.


Chap. 4 Dr. Zaidoon M. Shakoor
Example 4.6: Un-Steady State concentration profile for a spherical catalyst
Re-solve example 4.5 assuming that the system is unsteady state
Solution:
dCA d 2 CA 2 dCA
= D AE ( + ) − k1CA . . . . (4)
dt dr r dr
CA mj+1 − CA mj CA mj+1 − 2CA mj + CA mj−1 2 CA mj+1 − CA mj−1
= D AE ( + ) − k1CA mj . . . . (5)
∆t (∆r ) 2
rj 2∆r
m +1
CA mj+1 − 2CA mj + CA mj−1 1 CA j +1 − CA j −1
m m

CA = ∆t D AE ( + ) − ∆t k1CA mj + CA mj . . . . (6)
(∆r ) 2 ∆r
j
rj

Solution 1: Using crack Nicolson method

DAE=2e-7;% m2/s
R=.1;% m
K1=20;
CAs=500;% mol/m3
np=20;
dr=R/(np-1);
r=0:dr:R;
CA(1:np,1)=0;CA(np,1)=CAs;
m=0;
for time=0:10:3600;
m=m+1;
for j=2:np-1
CA(j,m+1)=CA(j,m)+dt*DAE*((CA(j+1,m)-2*CA(j,m)+…
CA(j-1,m))/(dr^2)+(2/(r(j)))*(CA(j+1,m)-CA(j,m))/dr);
end
CA(np,m+1)=CAs;
CA(1,m+1)=CA(2,m+1);
end
plot(r,CA(:,1:100:end))
xlabel('Radius m')
zlabel('Time (sec)')
ylabel('Concentration (mol/m3)')

The results will be such in figure (4.14)

Numerical Analysis / M. Sc. 81 Written by Assoc. Prof.


Chap. 4 Dr. Zaidoon M. Shakoor
500

Concentration (mol/m3) 400

300 Time Increasing

200

100

Time =0
0
0 0.02 0.04 0.06 0.08 0.1
Radius m
Figure (4.14) Concentration distribution at different time periods

Solution 2
Using method of line integration using fourth order Runge-Kutta Method
Eq. (4) Re-written in the form
dCA j CA mj+1 − 2CA mj + CA mj−1 2 CA j +1 − CA j −1
m m

= D AE ( + ) − k1CA mj . . . . (7)
dt (∆r ) 2 rj 2∆r
Eq. (7) represents the concentration change of each point with respect to time and
other points. This equation can be written for each point except the terminal points (at
r=0 and at r=R). Therefore these equations can be solved using Fourth order Runge-
Kutta integration method
R=0.1;% m
CAs=500;% mol/m3
np=20;
r=0:R/(np-1):R;
CAo(1:np)=0;CAo(np)=CAs;
Time=0:100:3600;
[Ti,CA]=ode45('Code',Time,CAo);
[RX,TX]=meshgrid(r,Time);
surf(RX,TX,CA);
xlabel('Radius m')
zlabel('Time (sec)')
ylabel('Concentration (mol/m3)')

Numerical Analysis / M. Sc. 82 Written by Assoc. Prof.


Chap. 4 Dr. Zaidoon M. Shakoor
function dCAdt=Code(t,CA)
DAE=2e-7;% m2/s
R=0.1;% m
K1=20;
CAs=500;% mol/m3
np=20;
r=0:R/(np-1):R;
dCAdt(np,1)=0;
for j=2:np-1
dCAdt(j,1)=DAE*((CA(j+1)-2*CA(j)+CA(j-1))/((r(j+1)-r(j))^2)...
+(1/(2*r(j)))*(CA(j+1)-CA(j))/(r(j+1)-r(j)));
end
dCAdt(1,1)=dCAdt(2,1);

The results will be such in figure (4.15)

500

400
Time (sec)

300

200

100

0
4000
0.1
2000
0.05

Concentr ation (mol/sec) 0 0 Radius m

Figure (4.15) Un-steady state concentration distribution

Numerical Analysis / M. Sc. 83 Written by Assoc. Prof.


Chap. 4 Dr. Zaidoon M. Shakoor

You might also like